NU335 ABSN Maternity Exam 1
22) The nurse is teaching a class on reading a fetal monitor to nursing students. The nurse explains that bradycardia is a fetal heart rate baseline below 110 and can be caused by which of the following? Select all that apply. 1. Maternal hypotension 2. Prolonged umbilical cord compression 3. Fetal dysrhythmia 4. Central nervous system malformation 5. Late fetal asphyxia
Answer: 1, 2, 3, 5
8) The nurse has completed the physical assessment of a client in early labor, and proceeds with the social assessment. A social history of the client would include which of the following? Select all that apply. 1. Use of drugs and alcohol 2. Family violence or sexual assault 3. Current living situation 4. Type of insurance 5. Availability of resources
Answer: 1, 2, 3, 5
30) The nurse is admitting a client who was diagnosed with hydramnios. The client asks why she has developed this condition. The nurse should explain that hydramnios is sometimes associated with which of the following? Select all that apply. 1. Rh sensitization 2. Postmaturity syndrome 3. Renal malformation or dysfunction 4. Maternal diabetes 5. Large-for-gestational-age infants
Answer: 1, 4
3) The female and male reproductive organs are homologous, which means what? 1. They are believed to cause vasoconstriction and muscular contraction 2. They are fundamentally similar in function and structure 3. They are rich in sebaceous glands 4. They are target organs for estrogenic hormones
Answer: 2 Explanation: 2. The female and male reproductive organs are homologous; that is, they are fundamentally similar in function and structure.
33) The nurse is presenting a prenatal class to a group of women pregnant for the first time who are all over 35 years of age. The nurse knows that the advantage of waiting until later to start a family is which of the following? 1. That the woman will have an easier labor and delivery. 2. That the baby will be at less of a risk for congenital anomalies. 3. That the woman is more likely to be financially secure. 4. That the woman will be more fertile than a younger woman would.
Answer: 3
21) A woman pregnant with twins asks the nurse about differences between identical and fraternal twins. The nurse explains that since it has been determined that she is having a boy and a girl, they are fraternal, and will have which of the following? 1. One placenta, two amnions, and two chorions 2. Two placentas, two amnions, and two chorions 3. Two placentas, one amnion, and two chorions 4. Two placentas, two amnions, one chorion
Answer 2 Explanation: 2. This is the correct answer. Fraternal twins have two placentas, two amnions, and two chorions, however, the placentas sometimes fuse and look as if they are one.
1) The nurse is caring for a pregnant woman who admits to using cocaine and ecstasy on a regular basis. The client states, "Everybody knows that alcohol is bad during pregnancy, but what's the big deal about ecstasy?" What is the nurse's best response? 1. "Ecstasy can cause a high fever in you and therefore cause the baby harm." 2. "Ecstasy leads to deficiencies of thiamine and folic acid, which help the baby develop." 3. "Ecstasy produces babies with small heads and short bodies with brain function alterations." 4. "Ecstasy produces intrauterine growth restriction and meconium aspiration."
Answer: 1
1) The nurse is presenting a class to women who are currently pregnant or are planning pregnancy in the near future. Which client statement indicates that additional teaching is required? 1. "The older a woman is when she conceives, the safer the pregnancy is." 2. "Pregnant teens can have additional nutritional needs." 3. "A woman whose sisters all had hypertension will be watched carefully." 4. "Pregnancy may be more difficult to achieve in my 40s."
Answer: 1
8) Which of the following tests has become a widely accepted method of evaluating fetal status? 1. Contraction stress test (CST) 2. MSAFP test 3. Non-stress test (NST) 4. Nuchal translucency test
Answer: 3
33) A pregnant client has a hemoglobin of 10 g/dL and a Hct of 30%. The clinic nurse recognizes the fetus is at risk for which of the following? Select all that apply. 1. Macrosomia 2. Respiratory distress syndrome 3. Low birth weight 4. Prematurity 5. Fetal death
Answer: 3, 4, 5 Explanation: 3. Anemia places the fetus at risk for a low birth weight. 4. Anemia places the fetus at risk for premature birth. 5. Anemia places the fetus at risk for fetal death.
15) A pregnant client who was of normal prepregnancy weight is now 30 weeks pregnant. She asks the nurse what appropriate weight gain for her should be. What is the nurse's best response? 1. "25-35 pounds" 2. "30-40 pounds" 3. "17-18 pounds" 4. "Less than 15 pounds"
Answer: 1
17) The prenatal clinic nurse is caring for a 15-year-old client who is at 8 weeks' gestation. The client asks the nurse why she is supposed to gain so much weight. What is the best response by the nurse? 1. "Gaining 25-35 pounds is recommended for healthy fetal growth." 2. "It's what your certified nurse-midwife recommended for you." 3. "Inadequate weight gain delays lactation after delivery." 4. "Weight gain is important to ensure that you get enough vitamins."
Answer: 1
17) What is the most significant maternal risk factor for preterm birth? 1. Previous preterm birth 2. Smoking 3. Stress 4. Substance abuse
Answer: 1
18) A pregnant client is admitted to the emergency department with bleeding. The nurse realizes that the client might have placenta previa. Which signs would be suggestive of placenta previa? 1. Bright red vaginal bleeding 2. Sudden onset of vaginal bleeding 3. Firm and hard uterus 4. Change in the size of abdomen
Answer: 1
2) The nurse is admitting a client for a cerclage procedure. The client asks for information about the procedure. What is the nurse's most accurate response? 1. "A stitch is placed in the cervix to prevent a spontaneous abortion or premature birth." 2. "The procedure is done during the third trimester." 3. "Cerclage is always placed after the cervix has dilated and effaced." 4. "An uncomplicated elective cerclage may is done on inpatient basis."
Answer: 1
20) The laboring client is complaining of tingling and numbness in her fingers and toes, dizziness, and spots before her eyes. The nurse recognizes that these are clinical manifestations of which of the following? 1. Hyperventilation 2. Seizure auras 3. Imminent birth 4. Anxiety
Answer: 1
31) The prenatal clinic nurse has received four phone calls. Which client should the nurse call back first? 1. Pregnant woman at 28 weeks with history of asthma who is reporting difficulty breathing and shortness of breath 2. Pregnant woman at 6 weeks with a seizure disorder who is inquiring which foods are good folic acid sources for her 3. Pregnant woman at 35 weeks with a positive HBsAG who is wondering what treatment her baby will receive after birth 4. Pregnant woman at 11 weeks with untreated hyperthyroidism who is describing the onset of vaginal bleeding
Answer: 1
8) The client with insulin-dependent type 2 diabetes and an HbA1c of 5.0% is planning to become pregnant soon. What anticipatory guidance should the nurse provide this client? 1. Insulin needs decrease in the first trimester and usually begin to rise late in the first trimester as glucose use and glycogen storage by the woman and fetus increase. 2. The risk of ketoacidosis decreases during the length of the pregnancy. 3. Vascular disease that accompanies diabetes slows progression. 4. The baby is likely to have a congenital abnormality because of the diabetes.
Answer: 1
9) To identify the duration of a contraction, the nurse would do which of the following? 1. Start timing from the beginning of one contraction to the completion of the same contraction. 2. Time between the beginning of one contraction and the beginning of the next contraction. 3. Palpate for the strength of the contraction at its peak. 4. Time from the beginning of the contraction to the peak of the same contraction.
Answer: 1
23) The nurse administered oxytocin 20 units at the time of placental delivery. Why was this primarily done? 1. To contract the uterus and minimize bleeding 2. To decrease breast milk production 3. To decrease maternal blood pressure 4. To increase maternal blood pressure
Answer: 1 Explanation: 1. Oxytocin is given to contract the uterus and minimize bleeding.
34) The nurse knows that a baby born to a mother who had oligohydramnios could show signs of which of the following? 1. Respiratory difficulty 2. Hypertension 3. Heart murmur 4. Decreased temperature
Answer: 1 Explanation: 1. Because there is less fluid available for the fetus to use during fetal breathing movements, pulmonary hypoplasia may develop.
18) Upon assessing the FHR tracing, the nurse determines that there is fetal tachycardia. The fetal tachycardia would be caused by which of the following? Select all that apply. 1. Early fetal hypoxia 2. Prolonged fetal stimulation 3. Fetal anemia 4. Fetal sleep cycle 5. Infection
Answer: 1, 2, 3, 5
2) During home care of a low-risk pregnant client, the nurse provides care by assessing which of the following? Select all that apply. 1. Urine 2. Weight 3. Diet 4. Pelvic measurements 5. Physical activity
Answer: 1, 2, 3, 5
9) In evaluating information taught about conception and fetal development, the client verbalizes understanding about transportation time of the zygote through the fallopian tube and into the cavity of the uterus with which statement? 1. "It will take at least 3 days for the egg to reach the uterus." 2. "It will take 8 days for the egg to reach the uterus." 3. "It will only take 12 hours for the egg to go through the fallopian tube." 4. "It will take 18 hours for the fertilized egg to implant in the uterus."
Answer: 1 Explanation: 1. "It will take at least 3 days for the egg to reach the uterus" is the correct statement.
24) The home health nurse is admitting a client at 18 weeks who is pregnant with twins. Which nursing action is most important? 1. Teach the client about foods that are good sources of protein. 2. Assess the client's blood pressure in her upper right arm. 3. Determine whether the pregnancy is the result of infertility treatment. 4. Collect a cervicovaginal fetal fibronectin (fFN) specimen.
Answer: 1 Explanation: 1. A daily intake of 4000 kcal (minimum) and 135 g protein is recommended for a woman with normal-weight twins.
9) The prenatal clinic nurse is explaining test results to a client who has had an assessment for fetal well-being. Which statement indicates that the client understands the test result? 1. "The normal Doppler velocimetry wave result indicates my placenta is getting enough blood to the baby." 2. "The reactive non-stress test means that my baby is not growing because of a lack of oxygen." 3. "Because my contraction stress test was positive, we know that my baby will tolerate labor well." 4. "My biophysical profile score of 6 points to everything being normal and healthy for my baby."
Answer: 1 Explanation: 1. A decrease in fetal cardiac output or an increase in resistance of placental vessels will reduce umbilical artery blood flow. Doppler velocimetry is best used when intrauterine growth restriction is diagnosed; therefore, the baby is getting an adequate blood supply.
17) The nurse auscultates the FHR and determines a rate of 112 beats/min. Which action is appropriate? 1. Inform the maternal client that the rate is normal. 2. Reassess the FHR in 5 minutes because the rate is low. 3. Report the FHR to the doctor immediately. 4. Turn the maternal client on her side and administer oxygen.
Answer: 1 Explanation: 1. A fetal heart rate of 112 beats/min. falls within the normal range of 110-160 beats/min., so there is no need to inform the doctor.
18) A 27-year-old married woman is 16 weeks pregnant and has an abnormally low maternal serum alpha-fetoprotein test. Which statement indicates that the couple understands the implications of this test result? 1. "We have decided to have an abortion if this baby has Down syndrome." 2. "If we hadn't had this test, we wouldn't have to worry about this baby." 3. "I'll eat plenty of dark green leafy vegetables until I have the ultrasound." 4. "The ultrasound should be normal because I'm under the age of 35."
Answer: 1 Explanation: 1. A low maternal serum alpha-fetoprotein test can indicate trisomy 18 or trisomy 21 (Down syndrome). Many couples abort a fetus that has a genetic abnormality that significantly affects quality of life or has multiple medical problems. Down syndrome is more likely to occur in the fetuses of women over the age of 35 at delivery, but is not limited to this age group.
27) A cesarean section is ordered for a pregnant client. Because the client is to receive general anesthesia, what is the primary danger with which the nurse is concerned? 1. Fetal depression 2. Vomiting 3. Maternal depression 4. Uterine relaxation
Answer: 1 Explanation: 1. A primary danger of general anesthesia is fetal depression. The depression in the fetus is directly proportional to the depth and duration of the anesthesia.
18) The nurse is planning an educational session for pregnant vegans. What information should the nurse include? 1. Eating beans and rice provides complete protein needs. 2. Soy is not a good source of protein for vegans. 3. Rice contains a high level of vitamin B12. 4. Vegan diets are excessively high in iron.
Answer: 1 Explanation: 1. Adequate dietary protein can be obtained by consuming a varied diet with adequate caloric intake and plant-based proteins. Consuming an assortment of plant proteins throughout the day such as beans and rice, peanut butter on whole-grain bread, and whole-grain cereal with soy milk ensures that the expectant mother obtains all essential amino acids.
22) The pregnant client and her partner are both 40 years old. The nurse is explaining the options of chorionic villus sampling (CVS) and amniocentesis for genetic testing. The nurse should correct the client if she makes which statement? 1. "Amniocentesis results are available sooner than CVS results are." 2. "CVS carries a higher risk of limb abnormalities." 3. "Amniocentesis cannot detect a neural tube defect." 4. "CVS is performed through my belly or my cervix."
Answer: 1 Explanation: 1. Amniocentesis results take longer to process than do CVS results.
15) The nurse has completed a community presentation about the changes of pregnancy, and knows that the lesson was successful when a community member states that which of the following is one probable or objective change of pregnancy? 1. "Enlargement of the uterus" 2. "Hearing the baby's heart rate" 3. "Increased urinary frequency" 4. "Nausea and vomiting"
Answer: 1 Explanation: 1. An examiner can perceive the objective (probable) changes that occur in pregnancy. Enlargement of the uterus is a probable change.
32) To reduce possible side effects from a cesarean section under general anesthesia, clients are routinely given which type of medication? 1. Antacids 2. Tranquilizers 3. Antihypertensives 4. Anticonvulsants
Answer: 1 Explanation: 1. Antacids are routinely administered before surgery for a cesarean section.
9) The nurse has received end-of-shift reports in the high-risk maternity unit. Which client should the nurse see first? 1. The client at 26 weeks' gestation with placenta previa experiencing blood on toilet tissue after a bowel movement 2. The client at 30 weeks' gestation with placenta previa whose fetal monitor strip shows late decelerations 3. The client at 35 weeks' gestation with grade I abruptio placentae in labor who has a strong urge to push 4. The client at 37 weeks' gestation with pregnancy-induced hypertension whose membranes ruptured spontaneously
Answer: 1 Explanation: 1. Assessment of the woman with placenta previa must be ongoing to prevent or treat complications that are potentially lethal to the mother and fetus. Painless, bright red vaginal bleeding is the best diagnostic sign of placenta previa. This client is the highest priority.
28) A woman has a hydatidiform mole (molar pregnancy) evacuated, and is prepared for discharge. The nurse should make certain that the client understands that what is essential? 1. That she not become pregnant until after the follow-up program is completed 2. That she receive RhoGAM with her next pregnancy and birth 3. That she has her blood pressure checked weekly for the next 30 days 4. That she seek genetic counseling with her partner before the next pregnancy
Answer: 1 Explanation: 1. Because of the risk of choriocarcinoma, the woman treated for hydatidiform mole should receive extensive follow-up therapy. Follow-up care includes a baseline chest X-ray to detect lung metastasis and a physical examination including a pelvic examination. The woman should avoid pregnancy during this time because the elevated hCG levels associated with pregnancy would cause confusion as to whether cancer had developed.
19) The nurse is explaining to a new prenatal client that the certified nurse-midwife will perform clinical pelvimetry as a part of the pelvic exam. The nurse knows that teaching has been successful when the client makes which statement about the reason for the exam? 1. "It will help us know how big a baby I can deliver vaginally." 2. "Doing this exam is a part of prenatal care at this clinic." 3. "My sister had both of her babies by cesarean." 4. "I am pregnant with my first child."
Answer: 1 Explanation: 1. By performing a series of assessments and measurements, the examiner assesses the pelvis vaginally to determine whether the size and shape are adequate for a vaginal birth; this procedure is called clinical pelvimetry.
26) At her first prenatal visit, a woman is discussing fetal development with the nurse. The client asks, "When will my baby actually have a heartbeat?" The nurse should say the heartbeat of an embryo is distinguishable by what time? 1. "The fourth week" 2. "The sixth week" 3. "The eighth week" 4. "The twelfth week"
Answer: 1 Explanation: 1. By the end of the fourth week, embryonic blood is circulating between the embryo and the chorionic villi.
20) A nurse needs to evaluate the progress of a woman's labor. The nurse obtains the following data: cervical dilatation 6 cm; contractions mild in intensity, occurring every 5 minutes, with a duration of 30-40 seconds. Which clue in this data does not fit the pattern suggested by the rest of the clues? 1. Cervical dilatation 6 cm 2. Mild contraction intensity 3. Contraction frequency every 5 minutes 4. Contraction duration 30-40 seconds
Answer: 1 Explanation: 1. Cervical dilatation of 6 cm indicates the active phase of labor. During this phase the cervix dilates from about 4 to 7 cm and contractions and pain intensify.
31) During a prenatal exam, a client describes several psychosomatic symptoms and has several vague complaints. What could these behaviors indicate? 1. Abuse 2. Mental illness 3. Depression 4. Nothing, they are normal
Answer: 1 Explanation: 1. Chronic psychosomatic symptoms and vague complaints can be indicators of abuse.
5) The nurse is presenting a preconception counseling class. The nurse instructs the participants that niacin intake should increase during pregnancy to promote metabolic coenzyme activity. The nurse will know that teaching has been effective if a client suggests which food as a source of niacin? 1. Fish 2. Apples 3. Broccoli 4. Milk
Answer: 1 Explanation: 1. Dietary sources of niacin include meats, fish, and whole grains.
15) Doppler flow studies (umbilical velocimetry) help to assess which of the following? 1. Placental function and sufficiency 2. Fetal heart rate 3. Fetal growth and fluid levels 4. Maturity of the fetal lungs
Answer: 1 Explanation: 1. Doppler flow studies (umbilical velocimetry) help to assess placental function and sufficiency. Uteroplacental insufficiency is a risk for a woman with preeclampsia. If fetal growth restriction is present, Doppler velocimetry of the umbilical artery is useful for fetal surveillance.
23) The nurse is working with a client who has experienced a fetal death in utero at 20 weeks. The client asks what her baby will look like when it is delivered. Which statement by the nurse is best? 1. "Your baby will be covered in fine hair called lanugo." 2. "Your child will have arm and leg buds, not fully formed limbs." 3. "A white, cheesy substance called vernix caseosa will be on the skin." 4. "The genitals of the baby will be ambiguous."
Answer: 1 Explanation: 1. Downy fine hair called lanugo covers the body of a 20-week fetus.
10) The charge nurse is looking at the charts of laboring clients. Which client is in greatest need of further intervention? 1. Woman at 7 cm, fetal heart tones auscultated every 90 minutes 2. Woman at 10 cm and pushing, external fetal monitor applied 3. Woman with meconium-stained fluid, internal fetal scalp electrode in use 4. Woman in preterm labor, external monitor in place
Answer: 1 Explanation: 1. During active labor, the fetal heart tones should be auscultated every 30 minutes; every 90 minutes is not frequent enough.
7) The client has been pushing for 3 hours, and the fetus is making a slow descent. The partner asks the nurse whether pushing for this long is normal. How should the nurse respond? 1. "Your baby is taking a little longer than average, but is making progress." 2. "First babies take a long time to be born. The next baby will be easier." 3. "The birth would go faster if you had taken prenatal classes and practiced." 4. "Every baby is different; there really are no norms for labor and birth."
Answer: 1 Explanation: 1. Establishing rapport and a trusting relationship and providing information that is true is best response.
19) A 38-year-old client in her second trimester states a desire to begin an exercise program to decrease her fatigue. What is the most appropriate nursing response? 1. "Fatigue should resolve in the second trimester, but walking daily might help." 2. "Avoid a strenuous exercise regimen at your age. Drink coffee to combat fatigue." 3. "Avoid an exercise regimen due to your pregnancy. Try to nap daily." 4. "Fatigue will increase as pregnancy progresses, but running daily might help."
Answer: 1 Explanation: 1. Even mild to moderate exercise is beneficial during pregnancy. Regular exercise-at least 30 minutes of moderate exercise daily or at least most days of the week-is preferred.
34) The nurse is teaching a woman about her menstrual cycle. Which is the most important change that happens during the follicular phase of the menstrual cycle? 1. Maturation of the primordial follicle 2. Multiplication of the fimbriae 3. Secretion of human chorionic gonadotropin 4. Growth of the endometrium
Answer: 1 Explanation: 1. Follicle-stimulating hormone is elevated during the follicular phase, and the primordial follicle matures.
9) A nurse is checking the postpartum orders. The doctor has prescribed bed rest for 6-12 hours. The nurse knows this is an appropriate order if the client had which type of anesthesia? 1. Spinal 2. Pudendal 3. General 4. Epidural
Answer: 1 Explanation: 1. Following the birth, the woman may be kept flat. Although the effectiveness of the supine position to avoid headache following a spinal is controversial, the physician's orders may include lying flat for 6 to 12 hours.
29) The nurse at the prenatal clinic has four calls to return. Which phone call should the nurse return first? 1. Client at 32 weeks, reports headache and blurred vision. 2. Client at 18 weeks, reports no fetal movement in this pregnancy. 3. Client at 16 weeks, reports increased urinary frequency. 4. Client at 40 weeks, reports sudden gush of fluid and contractions.
Answer: 1 Explanation: 1. Headache and blurred vision are signs of preeclampsia, which is potentially life-threatening for both mother and fetus. This client has top priority.
33) A nurse is discussing diet with a pregnant woman. Which food should the nurse advise the client to avoid during her pregnancy? 1. Bologna 2. Cantaloupe 3. Spinach 4. Cornbread
Answer: 1 Explanation: 1. Hot dogs and other luncheon meats should not be eaten during pregnancy unless they are fully cooked.
5) An analgesic medication has been administered IM to a client in labor. How would the nurse evaluate if the medication was effective? 1. The client dozes between contractions. 2. The client is moaning during contractions. 3. The contractions decrease in intensity. 4. The contractions decrease in frequency.
Answer: 1 Explanation: 1. If the client dozes between contractions, the analgesic is effective. Analgesics decrease discomfort and increase relaxation.
24) A client delivered 30 minutes ago. Which postpartal assessment finding would require close nursing attention? 1. A soaked perineal pad since the last 15-minute check 2. An edematous perineum 3. The client experiencing tremors 4. A fundus located at the umbilicus
Answer: 1 Explanation: 1. If the perineal pad becomes soaked in a 15-minute period or if blood pools under the buttocks, continuous observation is necessary. As long as the woman remains in bed during the first hour, bleeding should not exceed saturation of one pad.
27) A 28-year-old woman has been an insulin-dependent diabetic for 10 years. At 36 weeks' gestation, she has an amniocentesis. A lecithin/sphingomyelin (L/S) ratio test is performed on the sample of her amniotic fluid. Because she is a diabetic, what would an obtained 2:1 ratio indicate for the fetus? 1. The fetus may or may not have immature lungs. 2. The amniotic fluid is contaminated. 3. The fetus has a neural tube defect. 4. There is blood in the amniotic fluid.
Answer: 1 Explanation: 1. Infants of diabetic mothers (IDMs) have a high incidence of false-positive results (i.e., the L/S ratio is thought to indicate lung maturity, but after birth the baby develops RDS).
18) Infants of women with preeclampsia during pregnancy tend to be small for gestational age (SGA) because of which condition? 1. Intrauterine growth restriction 2. Oliguria 3. Proteinuria 4. Hypertension
Answer: 1 Explanation: 1. Infants of women with preeclampsia during pregnancy tend to be small for gestational age (SGA) because of intrauterine growth restriction. The cause is related specifically to maternal vasospasm and hypovolemia, which result in fetal hypoxia and malnutrition.
12) The nurse is responding to phone calls. Whose call should the nurse return first? 1. A client at 37 weeks' gestation reports no fetal movement for 24 hours. 2. A client at 29 weeks' gestation reports increased fetal movement. 3. A client at 32 weeks' gestation reports decreased fetal movement X 2 days. 4. A client at 35 weeks' gestation reports decreased fetal movement X 4 hours.
Answer: 1 Explanation: 1. Lack of fetal movement can be an indication of nonreassuring fetal status or even fetal death. This client is the highest priority.
11) The primiparous client has told the nurse that she is afraid she will develop hemorrhoids during pregnancy because her mother did. Which statement would be best for the nurse to make? 1. "It is not unusual for women to develop hemorrhoids during pregnancy." 2. "Most women don't have any problem until after they've delivered." 3. "If your mother had hemorrhoids, you will get them, too." 4. "If you get hemorrhoids, you probably will need surgery to get rid of them."
Answer: 1 Explanation: 1. Many pregnant women will develop hemorrhoids. Hemorrhoids are varicosities of the veins in the lower end of the rectum and anus. During pregnancy, the gravid uterus presses on the veins and interferes with venous circulation. As the pregnancy progresses, the straining that accompanies constipation can contribute to the development of hemorrhoids.
29) The client at 20 weeks' gestation thinks she might have been exposed to a toxin at work that could affect fetal development. The client asks the nurse what organs might be affected at this point in pregnancy. What is the nurse's best response? 1. "The brain is developing now, and could be affected." 2. "Because you are in the second trimester, there is no danger." 3. "The internal organs like the heart and lungs could be impacted." 4. "It's best to not worry about possible problems with your baby."
Answer: 1 Explanation: 1. Maximum brain growth and myelination are occurring at this point in fetal development.
34) The nurse teaching the expectant parents about the placenta also talks about the circulation and how the fetus gets its oxygen. She will include in this teaching which important fact? 1. The placenta functions as the lungs for the fetus. 2. The fetus obtains its oxygen from the amniotic fluid. 3. The fetus receives its oxygen by osmosis from the mother's bloodstream. 4. Fetal circulation delivers the highest amount of oxygen to the abdomen and lower body of the fetus.
Answer: 1 Explanation: 1. Most of the blood supply bypasses the fetal lungs because they do not carry out respiratory gas exchange. The placenta assumes the function of the fetal lungs by supplying oxygen and allowing the fetus to excrete carbon dioxide into the maternal bloodstream.
7) A Navajo client who is 36 weeks pregnant meets with a traditional healer as well as her physician. What does the nurse understand this to mean? 1. The client is seeking spiritual direction. 2. The client does not trust her physician. 3. The client will not adapt to mothering well. 4. The client is experiencing complications of pregnancy.
Answer: 1 Explanation: 1. Navajo clients are aware of the mind-soul connection, and might try to follow certain practices to have a healthy pregnancy and birth. Practices could include focus on peace and positive thoughts as well as certain types of prayers and ceremonies. A traditional healer may assist them.
15) The nurse is preparing to assess the fetus of a laboring client. Which assessment should the nurse perform first? 1. Perform Leopold maneuvers to determine fetal position. 2. Count the fetal heart rate between, during, and for 30 seconds following a uterine contraction (UC). 3. Dry the maternal abdomen before using the Doppler. 4. The diaphragm should be cooled before using the Doppler.
Answer: 1 Explanation: 1. Performing Leopold maneuvers is the first step.
30) The nurse is inducing the labor of a client with severe preeclampsia. As labor progresses, fetal intolerance of labor develops. The induction medication is turned off, and the client is prepared for cesarean birth. Which statement should the nurse include in her preoperative teaching? 1. "Because of your preeclampsia you are at higher risk for hypotension after an epidural anesthesia." 2. "Because of your preeclampsia you might develop hypertension after a spinal anesthesia." 3. "Because of your preeclampsia your baby might have decreased blood pressure after birth." 4. "Because of your preeclampsia your husband will not be allowed into the operating room."
Answer: 1 Explanation: 1. Pregnancies complicated by preeclampsia are high-risk situations. The woman with mild preeclampsia usually may have the analgesia or anesthesia of choice, although the incidence of hypotension with epidural anesthesia is increased. If hypotension occurs with the epidural block, it provides further stress on an already compromised cardiovascular system.
8) A client is admitted to the birth setting in early labor. She is 3 cm dilated, -2 station, with intact membranes, and FHR of 150 bpm. Her membranes rupture spontaneously, and the FHR drops to 90 bpm with variable decelerations. What would the nurse's initial response be? 1. Perform a vaginal exam 2. Notify the physician 3. Place the client in a left lateral position 4. Administer oxygen at 2 L per nasal cannula
Answer: 1 Explanation: 1. Prolapsed umbilical cord can occur when the membranes rupture. The fetus is more likely to experience variable decelerations because the amniotic fluid is insufficient to keep pressure off the umbilical cord. A vaginal exam is the best way to confirm.
30) A pregnant client asks the nurse, "What is this "knuckle test" that is supposed to tell whether my baby has a genetic problem?" What does the nurse correctly explain? 1. "In the first trimester, the nuchal translucency measurement is added to improve the detection rate for Down syndrome and trisomy 18." 2. "You will need to ask the physician for an explanation." 3. "It tests for hemophilia A or B." 4. "It tests for Duchenne muscular dystrophy."
Answer: 1 Explanation: 1. Screening tests, such as nuchal translucency ultrasound are designed to gather information about the risk that the pregnancy could have chromosome abnormalities or open spina bifida.
36) The introduction of a new baby into the family is often the beginning of which of the following? 1. Sibling rivalry 2. Inconsistent childrearing 3. Toilet training 4. Weaning
Answer: 1 Explanation: 1. Sibling rivalry results from children's fear of change in the security of their relationships with their parents.
24) After teaching a pregnant client about the effects of smoking on pregnancy, the nurse knows that the client needs further education when she makes which statement? 1. "I am at increased risk for preeclampsia." 2. "I am at increased risk for preterm birth." 3. "I am at increased risk for placenta previa." 4. "I am at increased risk for abruptio placentae."
Answer: 1 Explanation: 1. Smoking is not associated with increased risk for preeclampsia.
18) A client tells you that her mother was a twin, two of her sisters have twins, and several cousins either are twins or gave birth to twins. The client, too, is expecting twins. Because there is a genetic predisposition to twins in her family, there is a good chance that the client will have what type of twins? 1. Dizygotic twins 2. Monozygotic twins 3. Identical twins 4. Nonzygotic twins
Answer: 1 Explanation: 1. Studies indicate that dizygotic twins tend to occur in certain families, perhaps because of genetic factors that result in elevated serum gonadotropin levels leading to double ovulation.
1) While completing the medical and surgical history during the initial prenatal visit, the 16-year-old primigravida interrupts with "Why are you asking me all these questions? What difference does it make?" Which statement would best answer the client's questions?" 1. "We ask these questions to detect anything that happened in your past that might affect the pregnancy." 2. "We ask these questions to see whether you can have prenatal visits less often than most clients do." 3. "We ask these questions to make sure that our paperwork and records are complete and up to date." 4. "We ask these questions to look for any health problems in the past that might affect your parenting."
Answer: 1 Explanation: 1. The course of a pregnancy depends on a number of factors, including the past pregnancy history (if this is not a first pregnancy), prepregnancy health of the woman, presence of disease/illness states, family history, emotional status, and past health care. Page Ref: 243
20) The laboring client's fetal heart rate baseline is 120 beats per minute. Accelerations are present to 135 beats/min. During contractions, the fetal heart rate gradually slows to 110, and is at 120 by the end of the contraction. What nursing action is best? 1. Document the fetal heart rate. 2. Apply oxygen via mask at 10 liters. 3. Prepare for imminent delivery. 4. Assist the client into Fowler's position.
Answer: 1 Explanation: 1. The described fetal heart rate has a normal baseline; the presence of accelerations indicates adequate fetal oxygenation, and early decelerations are normal. No intervention is necessary.
25) The pregnant client who is at 14 weeks' gestation asks the nurse why the doctor used to call her baby an embryo, and now calls it a fetus. What is the best answer to this question? 1. "A fetus is the term used from the ninth week of gestation and onward." 2. "We call a baby a fetus when it is larger than an embryo." 3. "An embryo is a baby from conception until the eighth week." 4. "The official term for a baby in utero is really zygote."
Answer: 1 Explanation: 1. The fetal stage begins in the ninth week.
7) After explaining how meiotic division occurs within the ovum, the nurse knows that the pregnant client understands when she makes what statement? 1. "The second meiotic division is arrested until and unless the oocyte is fertilized." 2. "Meiosis in the oocyte begins at puberty." 3. "The first meiotic division continues when the female infant is born." 4. "Fertilization does not take place in the secondary oocyte."
Answer: 1 Explanation: 1. This is true. The secondary oocyte moves into the metaphase stage of cell division, where its meiotic division is arrested until and unless the oocyte is fertilized.
26) The fetal heart rate baseline is 140 beats/min. When contractions begin, the fetal heart rate drops suddenly to 120, and rapidly returns to 140 before the end of the contraction. Which nursing intervention is best? 1. Assist the client to change position. 2. Apply oxygen to the client at 2 liters per nasal cannula. 3. Notify the operating room of the need for a cesarean birth. 4. Determine the color of the leaking amniotic fluid.
Answer: 1 Explanation: 1. The fetus is exhibiting variable decelerations, which are caused by cord compression. Sometimes late or variable decelerations are due to the supine position of the laboring woman. In this case, the decrease in uterine blood flow to the fetus may be alleviated by raising the woman's upper trunk or turning her to the side to displace pressure of the gravid uterus on the inferior vena cava.
21) The kosher diet followed by many Jewish people forbids the eating of what foods? 1. Pig products and shellfish 2. Dairy products 3. All animal products 4. Dairy products and eggs
Answer: 1 Explanation: 1. The kosher diet followed by many Jewish people forbids the eating of pig products and shellfish. Certain cuts of meat from sheep and cattle are allowed, as are fish with fins and scales. In addition, many Jews believe that meat and dairy products should not be mixed or eaten at the same meal.
14) What is the major adverse side effect of epidural anesthesia? 1. Maternal hypotension 2. Decrease in variability of the FHR 3. Vertigo 4. Decreased or absent respiratory movements
Answer: 1 Explanation: 1. The major adverse effect of epidural anesthesia is maternal hypotension caused by a spinal blockade, which lowers peripheral resistance, decreases venous return to the heart, and subsequently lessens cardiac output and lowers blood pressure.
17) After administration of an epidural anesthetic to a client in active labor, it is most important to assess the mother immediately for which of the following? 1. Hypotension 2. Headache 3. Urinary retention 4. Bradycardia
Answer: 1 Explanation: 1. The most common complication of an epidural is maternal hypotension.
19) The OB-GYN nurse knows that the most common shape for the female pelvis is which of the following? 1. Gynecoid type 2. Android type 3. Anthropoid type 4. Platypelloid type
Answer: 1 Explanation: 1. The most common female pelvis is the gynecoid type. The inlet is rounded with the anteroposterior diameter a little shorter than the transverse diameter.
10) A woman at 7 weeks' gestation is diagnosed with hyperemesis gravidarum. Which nursing diagnosis would receive priority? 1. Fluid Volume: Deficient 2. Cardiac Output, Decreased 3. Injury, Risk for 4. Nutrition, Imbalanced: Less than Body Requirements
Answer: 1 Explanation: 1. The newly admitted client with hyperemesis gravidarum has been experiencing excessive vomiting, and is in a fluid volume-deficit state.
6) It is 1 week before a pregnant client's due date. The nurse notes on the chart that the client's pulse rate was 74-80 before pregnancy. Today, the client's pulse rate at rest is 90. What action should the nurse should take? 1. Chart the findings. 2. Notify the physician of tachycardia. 3. Prepare the client for an electrocardiogram (EKG). 4. Prepare the client for transport to the hospital.
Answer: 1 Explanation: 1. The pulse rate frequently increases during pregnancy, although the amount varies from almost no increase to an increase of 10 to 15 beats per minute. This is a normal response, and does not indicate a need for emergency measures or treatment.
10) At 32 weeks' gestation, a woman is scheduled for a second non-stress test (following one she had at 28 weeks' gestation). Which statement by the client would indicate an adequate understanding of this procedure? 1. "I can't get up and walk around during the test." 2. "I'll have an IV started before the test." 3. "I can still smoke before the test." 4. "I need to have a full bladder for this test."
Answer: 1 Explanation: 1. The purpose of the non-stress test is to determine the results of movement on fetal heart rate. The NST is typically performed with the woman in the semi-Fowler's position with a small pillow or blanket under the right hip to displace the uterus to the left.
4) A woman gave birth last week to a fetus at 18 weeks' gestation after her first pregnancy. She is in the clinic for follow-up, and notices that her chart states she has had one abortion. The client is upset over the use of this word. How can the nurse best explain this terminology to the client? 1. "Abortion is the obstetric term for all pregnancies that end before 20 weeks." 2. "Abortion is the word we use when someone has miscarried." 3. "Abortion is how we label babies born in the second trimester." 4. "Abortion is what we call all babies who are born dead."
Answer: 1 Explanation: 1. The term abortion means a birth that occurs before 20 weeks' gestation or the birth of a fetus-newborn who weighs less than 500 g. An abortion may occur spontaneously or it may be induced by medical or surgical means.
6) The nurse teaching a class on reproductive anatomy knows that no further instruction is needed when a student shows an understanding of the pelvic cavity divisions by making which statement? 1. "The true pelvis is made up of the sacrum, coccyx, and innominate bones." 2. "The false pelvis consists of the inlet, the pelvic cavity, and the outlet." 3. "The true pelvis is the portion above the pelvic brim." 4. "The relationship between the false pelvis and the fetal head is of paramount importance."
Answer: 1 Explanation: 1. The true pelvis is made up of the sacrum, the coccyx, and the two innominate bones.
31) The nurse asks a woman how her husband is dealing with the pregnancy. The nurse concludes that counseling is needed when the woman makes which statement? 1. "My husband is ready for the pregnancy to end so that we can have sex again." 2. "My husband is much more attentive to me now that I am pregnant." 3. "My husband seems more worried about our finances now than he was before the pregnancy." 4. "My husband plays his favorite music for my belly so the baby will learn to like it."
Answer: 1 Explanation: 1. This is implying that the woman and her husband are not having sex, which indicates the need for counseling. Sex is fine with a normal pregnancy.
30) The nurse educator is presenting a program to college students about factors that can cause congenital malformations. What should the nurse tell them? 1. The growing embryo is considered most vulnerable to hazardous agents during the first months of pregnancy. 2. Spontaneous abortion always occurs if the fetus is affected by a teratogen. 3. Potential teratogens can cause malformations of the heart, limbs, eyes, and other organ systems only in the second trimester. 4. Teratogen agents are primarily drugs.
Answer: 1 Explanation: 1. This is true. Because organs are formed primarily during embryonic development, the growing embryo is considered most vulnerable to hazardous agents during the first months of pregnancy.
11) The nurse is preparing for a postpartum home visit. The client has been home for a week, is breastfeeding, and experienced a third-degree perineal tear after vaginal delivery. The nurse should assess the client for which of the following? 1. Dietary intake of fiber and fluids 2. Dietary intake of folic acid and prenatal vitamins 3. Return of hemoglobin and hematocrit levels to baseline 4. Return of protein and albumin to predelivery levels
Answer: 1 Explanation: 1. This mother needs to avoid the risk of constipation. She might be hesitant to have a bowel movement due to anticipated pain from the perineal tear, and constipation will decrease the healing of the laceration.
20) The labor nurse is caring for a client at 38 weeks' gestation who has been diagnosed with symptomatic placenta previa. Which physician order should the nurse question? 1. Begin oxytocin drip rate at 0.5 milliunits/min. 2. Assess fetal heart rate every 10 minutes. 3. Weigh all vaginal pads. 4. Assess hematocrit and hemoglobin.
Answer: 1 Explanation: 1. This order should be questioned, as this client is not a good candidate for labor induction.
25) The client at 14 weeks' gestation has undergone a transvaginal ultrasound to assess cervical length. The ultrasound revealed cervical funneling. How should the nurse explain these findings to the client? 1. "Your cervix has become cone-shaped and more open at the end near the baby." 2. "Your cervix is lengthened, and you will deliver your baby prematurely." 3. "Your cervix is short, and has become wider at the end that extends into the vagina." 4. "Your cervix was beginning to open but now is starting to close up again."
Answer: 1 Explanation: 1. Transvaginal ultrasound can most accurately identify shortened cervical length and cervical funneling, which is a cone-shaped indentation in the cervical os indicating cervical insufficiency or risk of preterm labor.
30) A pregnant client is lactose intolerant. Which alternative food could this client eat to get sufficient calcium? 1. Turnip greens 2. Green beans 3. Cantaloupe 4. Nectarines
Answer: 1 Explanation: 1. Turnip greens are rich in calcium.
12) By inquiring about the expectations and plans that a laboring woman and her partner have for the labor and birth, the nurse is primarily doing which of the following? 1. Recognizing the client as an active participant in her own care. 2. Attempting to correct any misinformation the client might have received. 3. Acting as an advocate for the client. 4. Establishing rapport with the client.
Answer: 1 Explanation: 1. Understanding the couple's expectations and plans helps the nurse provide optimal nursing care and facilitate the best possible birth experience.
25) A client is concerned because she has been told her blood type and her baby's are incompatible. What is the nurse's best response? 1. "This is called ABO incompatibility. It is somewhat common but rarely causes significant hemolysis." 2. "This is a serious condition, and additional blood studies are currently in process to determine whether you need a medication to prevent it from occurring with a future pregnancy." 3. "This is a condition caused by a blood incompatibility between you and your husband, but does not affect the baby." 4. "This type of condition is very common, and the baby can receive a medication to prevent jaundice from occurring."
Answer: 1 Explanation: 1. When blood types, not Rh, are incompatible, it is called ABO incompatibility. The incompatibility occurs as a result of the maternal antibodies present in her serum and interaction between the antigen sites on the fetal RBCs.
14) The laboring client presses the call light and reports that her water has just broken. What would the nurse's first action be? 1. Check fetal heart tones. 2. Encourage the mother to go for a walk. 3. Change bed linens. 4. Call the physician.
Answer: 1 Explanation: 1. When the membranes rupture, the nurse notes the color and odor of the amniotic fluid and the time of rupture and immediately auscultates the FHR.
7) The client has asked the nurse why her cervix has only changed from 1 to 2 cm in 3 hours of contractions occurring every 5 minutes. What is the nurse's best response to the client? 1. "Your cervix has also effaced, or thinned out, and that change in the cervix is also labor progress." 2. "When your perineal body thins out, your cervix will begin to dilate much faster than it is now." 3. "What did you expect? You've only had contractions for a few hours. Labor takes time." 4. "The hormones that cause labor to begin are just getting to be at levels that will change your cervix."
Answer: 1 Explanation: 1. With each contraction, the muscles of the upper uterine segment shorten and exert a longitudinal traction on the cervix, causing effacement. Effacement is the taking up (or drawing up) of the internal os and the cervical canal into the uterine side walls.
12) A client with diabetes is receiving preconception counseling. The nurse will emphasize that during the first trimester, the woman should be prepared for which of the following? 1. The need for less insulin than she normally uses 2. Blood testing for anemia 3. Assessment for respiratory complications 4. Assessment for contagious conditions
Answer: 1 Explanation: 1. Women with diabetes often require less insulin during the first trimester.
19) The nurse notes the following findings in a client at 12-weeks' gestation. Which of the findings would enable the nurse to tell the client that she is diagnostically pregnant? 1. Fetal heart rate by Doppler 2. Positive pregnancy test 3. Positive Chadwicks sign 4. Montgomery gland enlargement
Answer: 1 Explanation: 1. A fetal heart rate by Doppler is a diagnostic (positive) change of pregnancy.
13) A primary herpes simplex infection in the first trimester can increase the risk of which of the following? 1. Spontaneous abortion 2. Preterm labor 3. Intrauterine growth restriction 4. Neonatal infection
Answer: 1 Explanation: 1. A primary herpes simplex infection can increase the risk of spontaneous abortion when infection occurs in the first trimester.
1) How would the nurse best analyze the results from a client's sonogram that shows the fetal shoulder as the presenting part? 1. Breech, transverse 2. Breech, longitudinal 3. Breech, frank 4. Vertex, transverse
Answer: 1 Explanation: 1. A shoulder presentation is one type of breech presentation, and is also called a transverse lie.
19) The nurse is preparing a client for amniocentesis. Which statement would indicate that the client clearly understands the risks of an amniocentesis? 1. "I might go into labor early." 2. "It could produce a congenital defect in my baby." 3. "Actually, there are no real risks to this procedure." 4. "The test could stunt my baby's growth."
Answer: 1 Explanation: 1. Amniocentesis has the potential for causing a spontaneous abortion.
35) A 43-year-old client has just had a positive pregnancy test. She cries, and states, "I just don't know what I'll do. I can't be pregnant." Which nursing diagnosis would be the most appropriate? 1. Decisional Conflict related to unexpected pregnancy 2. Knowledge, Deficient related to advanced maternal age 3. Depression related to unexpected pregnancy 4. Health Maintenance, Ineffective related to advanced maternal age
Answer: 1 Explanation: 1. Decisional Conflict related to unexpected pregnancy is the most appropriate nursing diagnosis.
14) A woman is being treated for preterm labor with magnesium sulfate. The nurse is concerned that the client is experiencing early drug toxicity. What assessment finding by the nurse indicates early magnesium sulfate toxicity? 1. Patellar reflexes weak or absent 2. Increased appetite 3. Respiratory rate of 16 4. Fetal heart rate of 120
Answer: 1 Explanation: 1. Early signs of magnesium sulfate toxicity are related to a decrease in deep tendon reflexes.
33) The nurse is caring for a laboring client with thrombocytopenia. During labor, it is determined that the client requires a cesarean delivery. The nurse is preparing the client for surgery, and should instruct the client that the recommended method of anesthesia is which of the following? 1. General anesthesia 2. Epidural anesthesia 3. Spinal anesthesia 4. Regional anesthesia
Answer: 1 Explanation: 1. General anesthesia will be recommended. Women with thrombocytopenia should avoid regional blocks.
28) The nurse is preparing a prenatal class about infant feeding methods. The maternal nutritional requirements for breastfeeding and formula-feeding will be discussed. What statement should the nurse include? 1. "Breastfeeding requires a continued high intake of protein and calcium." 2. "Formula-feeding mothers should protect their health with a lot of calcium." 3. "Producing breast milk requires calories, but any source of food is fine." 4. "Formula-feeding mothers need a high protein intake to avoid fatigue."
Answer: 1 Explanation: 1. Lactation requires calories, along with increased protein and calcium intake.
4) A woman in active labor is given nalbuphine hydrochloride (Nubain) 14 mg IV for pain relief. Half an hour later, her respirations are at 8 per minute. The physician would likely order which medication for this client? 1. Narcan 2. Reglan 3. Benadryl 4. Vistaril
Answer: 1 Explanation: 1. Narcan is useful for respiratory depression caused by nalbuphine (Nubain).
23) The client with blood type A, Rh-negative, delivered yesterday. Her infant is blood type AB, Rh-positive. Which statement indicates that teaching has been effective? 1. "I need to get RhoGAM so I don't have problems with my next pregnancy." 2. "Because my baby is Rh-positive, I don't need RhoGAM." 3. "If my baby had the same blood type I do, it might cause complications." 4. "Before my next pregnancy, I will need to have a RhoGAM shot."
Answer: 1 Explanation: 1. Rh-negative mothers who give birth to Rh-positive infants should receive Rh immune globulin (RhoGAM) to prevent alloimmunization.
13) The nurse is seeing prenatal clients in the clinic. Which client is exhibiting expected findings? 1. 12 weeks' gestation, with fetal heart tones heard by Doppler fetoscope 2. 22 weeks' gestation, client reports no fetal movement felt yet 3. 16 weeks' gestation, fundus three finger breadths above umbilicus 4. Marked edema
Answer: 1 Explanation: 1. This is an expected finding because fetal heart tones should be heard by 12 weeks using a Doppler fetoscope.
18) Two hours after delivery, a client's fundus is boggy and has risen to above the umbilicus. What is the first action the nurse would take? 1. Massage the fundus until firm 2. Express retained clots 3. Increase the intravenous solution 4. Call the physician
Answer: 1 Explanation: 1. When the uterus becomes boggy, pooling of blood occurs within it, resulting in the formation of clots. Anything left in the uterus prevents it from contracting effectively. Thus if it becomes boggy or appears to rise in the abdomen, the fundus should be massaged until firm.
3) A couple in their late 30s are pregnant for the first time. In evaluating the care delivered, the nurse assesses the mother for which of the following? Select all that apply. 1. Makes appropriate healthcare choices 2. Receives effective healthcare through the pregnancy 3. Has a partner who is not interested in child care 4. Cannot cope with her life change 5. Wishes to have amniocentesis done
Answer: 1, 2 Explanation: 1. Expected outcomes of nursing care for a pregnant couple over 35 include that the client and her partner are knowledgeable about the pregnancy and make appropriate healthcare choices. 2. Expected outcomes of nursing care for a pregnant couple over 35 include the client and her partner receive effective health care throughout the pregnancy, birth, and postpartum period.
21) The nurse is presenting a class for nursing students on multiple-gestation pregnancy. Which statements about multiple-gestation pregnancies are accurate? Select all that apply. 1. Hypertension is a major maternal complication. 2. Gestational diabetes occurs more often. 3. Maternal anemia occurs. 4. Pulmonary embolism is 12 times more likely to develop during pregnancy with multiple gestations. 5. Multiple gestations are more likely to acquire HELLP.
Answer: 1, 2, 3, 5
12) A client at 32 weeks' gestation comes to the clinic with urinary burning and frequency. The nurse explains that urinary tract infections are common in pregnancy due to which of the following? Select all that apply. 1. Ureteral atonia 2. Stasis of urine 3. Increased glomerular filtration rate 4. Increased renal plasma flow 5. Increased clearance of urea
Answer: 1, 2 Explanation: 1. The presence of amino acids and glucose in the urine in conjunction with the tendency toward ureteral atonia and stasis of urine in the ureters may increase the risk of urinary tract infection. 2. The presence of amino acids and glucose in the urine in conjunction with the tendency toward ureteral atonia and stasis of urine in the ureters may increase the risk of urinary tract infection.
15) A client is admitted to the labor and delivery unit in active labor. What nursing diagnoses might apply to the client with suspected abruptio placentae? Select all that apply. 1. Fluid Volume, Deficient, Risk for, related to hypovolemia secondary to excessive blood loss 2. Tissue Perfusion: Peripheral, Ineffective, related to blood loss secondary to uterine atony following birth 3. Anxiety related to concern for own personal status and the baby's safety 4. Knowledge, Deficient related to lack of information about inherited genetic defects 5. Alteration in Respiratory Function related to blood loss
Answer: 1, 2, 3
33) A pregnant asthmatic client is being seen for her initial prenatal visit. The nurse knows that the fetal implications of maternal asthma include which of the following? Select all that apply. 1. Prematurity 2. Low birth weight 3. Hypoxia with maternal exacerbation 4. Congenital anomalies from the medications 5. Perinatal transfer of the asthma
Answer: 1, 2, 3
36) What self-care measures would a nurse recommend for a client in her first trimester to reduce the discomfort of nausea and vomiting? Select all that apply. 1. Avoid odors or causative factors. 2. Have small but frequent meals. 3. Drink carbonated beverages. 4. Drink milk before arising in the morning. 5. Eat highly seasoned food.
Answer: 1, 2, 3
28) A cesarean section is ordered for the laboring client with whom the nurse has worked all shift. The client will receive general anesthesia. The nurse knows that potential complications of general anesthesia include which of the following? Select all that apply. 1. Fetal depression that is directly proportional to the depth and duration of the anesthesia 2. Poor fetal metabolism of anesthesia, which inhibits use with preterm infants 3. Uterine relaxation 4. Increased gastric motility 5. Itching of the face and neck
Answer: 1, 2, 3 Explanation: 1. A primary danger of general anesthesia is fetal depression, because the medication reaches the fetus in about 2 minutes. The depression is directly proportional to the depth and duration of anesthesia. 2. The poor fetal metabolism of general anesthetic agents is similar to that of analgesic agents administered during labor. General anesthesia is not advocated when the fetus is considered to be at high risk, particularly in preterm birth. 3. Most general anesthetic agents cause some degree of uterine relaxation.
3) During the initial prenatal visit, the nurse obtains a weight of 42 kg (92.4 lb). The nurse must further assess the client for information about which of the following? Select all that apply. 1. Eating habits 2. Foods regularly eaten 3. Income limitations 4. Blood pressure and pulse rate 5. Weight loss during pregnancy
Answer: 1, 2, 3 Explanation: 1. For a client whose weight is less than 100 lb, the nurse would obtain information on eating habits. 2. For a client whose weight is less than 100 lb, the nurse would obtain information on foods regularly eaten. 3. For a client whose weight is less than 100 lb, the nurse would obtain information on income limitations.
22) The labor nurse would not encourage a mother to bear down until the cervix is completely dilated, to prevent which of the following? Select all that apply. 1. Maternal exhaustion 2. Cervical edema 3. Tearing and bruising of the cervix 4. Enhanced perineal thinning 5. Having to perform an episiotomy
Answer: 1, 2, 3 Explanation: 1. If the cervix is not completely dilated, maternal exhaustion can occur. 2. If the cervix is not completely dilated, cervical edema can occur. 3. If the cervix is not completely dilated, tearing and bruising of the cervix can occur.
4) The nurse is teaching about reproduction, and explains that which of the following are the purposes of meiosis? Select all that apply. 1. Produce gametes 2. Reduce the number of chromosomes 3. Introduce genetic variability 4. Produce cells for growth and development 5. Divide somatic cells into new cells with identical characteristics
Answer: 1, 2, 3 Explanation: 1. Meiosis is a special type of cell division by which diploid cells give rise to gametes (sperm and ova). 2. The cells contain half the genetic material of the parent cell-only 23 chromosomes-the haploid number of chromosomes. 3. During meiosis new combinations of cells are provided by the newly formed chromosomes; these combinations account for the wide variation of traits.
The OB-GYN nurse works in a clinic with a culturally diverse group of clients whose specific actions during pregnancy are often determined by cultural beliefs. The nurse recognizes that these beliefs about pregnancy and childbirth fall into which categories? Select all that apply. 1. Prescriptive beliefs 2. Restrictive beliefs 3. Taboos 4. Cultural humility 5. Folk treatment beliefs
Answer: 1, 2, 3 Explanation: 1. Prescriptive beliefs or requirements describe expected behaviors. 2. Restrictive beliefs are stated negatively and limit behaviors. 3. Taboo beliefs refer to specific supernatural consequences.
37) The clinic nurse is culturally sensitive when, while assessing the pregnant client, he asks about which of the following? Select all that apply. 1. The family's expectations of the healthcare system 2. Which cultural practices should be incorporated into care 3. Any alternative healer who should be consulted 4. Positive consequences of the client's healthcare beliefs 5. The client's giving up her practices and adopting the practices of the dominant culture
Answer: 1, 2, 3 Explanation: 1. The nurse needs to ask about the client's expectations of the healthcare system. 2. The nurse should ask about any cultural or spiritual practices that should be incorporated into care. 3. The culturally sensitive nurse will ask whether any alternative healer should be consulted about care.
32) The nurse is working with a pregnant woman who has systemic lupus erythematosus (SLE). What does the nurse anticipate the infant might be born with? Select all that apply. 1. A tendency to bleed excessively 2. An increased chance of developing infections 3. A hemoglobin less than optimal for good health 4. Problems with vision 5. Hearing loss
Answer: 1, 2, 3 Explanation: 1. This is true, as the infant might be born with thrombocytopenia. 2. This is true, as the infant might be born with neutropenia. 3. This is true, as the infant might be born with anemia.
6) The prenatal period should be used to expose the prospective parents to up-to-date, evidence-based information about which of the following topics? Select all that apply. 1. Breastfeeding 2. Pain relief 3. Obstetric complications and procedures 4. Toddler care 5. Antepartum adjustment
Answer: 1, 2, 3 Explanation: 1. The prenatal period should expose prospective parents to up-to-date, evidence-based information about breastfeeding. 2. The prenatal period should expose prospective parents to up-to-date, evidence-based information about pain relief. 3. The prenatal period should expose prospective parents to up-to-date, evidence-based information about obstetric complications and procedures.
10) The nurse is presenting a class on preterm labor, its causes, and treatments to a group of newly pregnant couples. Which statements regarding preterm labor are true? Select all that apply. 1. Antepartum hemorrhage can cause preterm labor. 2. Trauma can cause preterm labor. 3. Infection can cause preterm labor. 4. Magnesium sulfate is a drug used to stop contractions. 5. Sedatives and narcotics may be given to stop labor.
Answer: 1, 2, 3, 4
2) The nurse educator is presenting a class on the different kinds of miscarriages. Miscarriages, or spontaneous abortions, are classified clinically into which of the following different categories? Select all that apply. 1. Threatened abortion 2. Incomplete abortion 3. Complete abortion 4. Missed abortion 5. Acute abortion
Answer: 1, 2, 3, 4
21) The nurse is caring for a client who is having fetal tachycardia. The nurse knows that possible causes include which of the following? Select all that apply. 1. Maternal dehydration 2. Maternal hyperthyroidism 3. Fetal hypoxia 4. Prematurity 5. Anesthesia or regional analgesia
Answer: 1, 2, 3, 4
23) What fetal factors require a cesarean birth? 1. Severe intrauterine growth restriction (IUGR) 2. Fetal anomalies 3. Unfavorable fetal position or presentation 4. Preterm birth 5. Lack of maternal attachment
Answer: 1, 2, 3, 4
33) The partner of a pregnant client comes to the clinic with her. He complains to the nurse that he is experiencing different physical changes. The nurse determines he is experiencing couvade when he describes which symptoms? Select all that apply. 1. Fatigue 2. Increased appetite 3. Headache 4. Backache 5. High anxiety level
Answer: 1, 2, 3, 4
35) The nurse knows that a mother who has been treated for Beta streptococcus passes this risk on to her newborn. Risk factors for neonatal sepsis caused by Beta streptococcus include which of the following? Select all that apply. 1. Prematurity 2. Maternal intrapartum fever 3. Membranes ruptured for longer than 18 hours 4. A previously infected infant with GBS disease 5. An older mother having her first baby
Answer: 1, 2, 3, 4
5) The nurse is working with a woman who abuses stimulants. The nurse is aware that the fetus is at risk for which of the following? Select all that apply. 1. Withdrawal symptoms 2. Cardiac anomalies 3. Sudden infant death syndrome 4. Being small for gestational age 5. Fetal alcohol syndrome
Answer: 1, 2, 3, 4
12) The nurse is counseling a group of first-trimester clients on diet increases that are necessary during pregnancy. Which information would be necessary to tell the pregnant women? Select all that apply. 1. An increase of protein is necessary to provide amino acids necessary for fetal development. 2. Protein contributes to the body's overall energy metabolism. 3. The recommended protein during pregnancy is 70 grams each day. 4. The increased amount of protein that a pregnant woman needs is 15 grams a day. 5. The quality of protein is as important as the amount.
Answer: 1, 2, 3, 5
33) Childbirth preparation offers several advantages including which of the following? Select all that apply. 1. It helps a pregnant woman and her support person understand the choices in the birth setting. 2. It promotes awareness of available options. 3. It provides tools for a pregnant woman and her support person to use during labor and birth. 4. Women who receive continuous support during labor require more analgesia, and have more cesarean and instrument births. 5. Each method has been shown to shorten labor.
Answer: 1, 2, 3, 5 Explanation: 1. Childbirth preparation offers several advantages. It helps a pregnant woman and her support person understand the choices in the birth setting, promotes awareness of available options, and provides tools for them to use during labor and birth. 2. Childbirth preparation offers several advantages. It helps a pregnant woman and her support person understand the choices in the birth setting, promotes awareness of available options, and provides tools for them to use during labor and birth. 3. Childbirth preparation offers several advantages. It helps a pregnant woman and her support person understand the choices in the birth setting, promotes awareness of available options, and provides tools for them to use during labor and birth. 5. Childbirth preparation offers several advantages. Each method has been shown to shorten labor.
16) When blood pressure and other signs indicate that the preeclampsia is worsening, hospitalization is necessary to monitor the woman's condition closely. At that time, which of the following should be assessed? Select all that apply. 1. Fetal heart rate 2. Blood pressure 3. Temperature 4. Urine color 5. Pulse and respirations
Answer: 1, 2, 3, 5 Explanation: 1. Determine the fetal heart rate along with blood pressure, or monitor continuously with the electronic fetal monitor if the situation indicates. 2. Determine blood pressure every 1 to 4 hours, or more frequently if indicated by medication or other changes in the woman's status. 3. Determine temperature every 4 hours, or every 2 hours if elevated or if premature rupture of the membranes (PROM) has occurred. 5. Determine pulse rate and respirations along with blood pressure.
21) For what common side effects of epidural anesthesia should the nurse watch? Select all that apply. 1. Elevated maternal temperature 2. Urinary retention 3. Nausea 4. Long-term back pain 5. Local itching
Answer: 1, 2, 3, 5 Explanation: 1. Elevated maternal temperature is a potential side effect of epidural anesthesia. 2. Urinary retention is a potential side effect of epidural anesthesia. 3. Nausea is a potential side effect of epidural anesthesia. 5. Pruritus may occur at any time during the epidural infusion. It usually appears first on the face, neck, or torso and is generally the result of the agent used in the epidural infusion. Benadryl, an antihistamine, can be administered to manage pruritus.
29) Which of the following symptoms, if progressive, are indicative of CHF, the heart's signal of its decreased ability to meet the demands of pregnancy? Select all that apply. 1. Palpitations 2. Heart murmurs 3. Dyspnea 4. Frequent urination 5. Rales
Answer: 1, 2, 3, 5 Explanation: 1. Palpitations are indicative of CHF. 2. Heart murmurs are indicative of CHF. 3. Dyspnea is indicative of CHF. 5. Rales are indicative of CHF.
31) The nurse is teaching the significance of good nutrition on fetal development to a group of young women. What factors will the nurse discuss? Select all that apply. 1. Vitamins and folic acid are important before conception. 2. Maternal nutrition may predispose to the development of adult coronary heart disease, hypertension, and diabetes. 3. Glucose, amino acids, and fatty acids are of high importance. 4. Nutrition is important only during the first trimester. 5. Maternal nutrition can affect brain and neural tube development.
Answer: 1, 2, 3, 5 Explanation: 1. Vitamins and folic acid supplements taken before conception can reduce the incidence of neural tube defects. 2. This is true. Maternal nutrition may predispose to the development of adult coronary heart disease, hypertension, and diabetes. 3. Amino acids, glucose, and fatty acids are considered to be the primary dietary factors in brain growth. 5. Adequacy of the maternal environment is also important during the periods of rapid embryonic and fetal development. Maternal nutrition can affect brain and neural tube development.
22) A full-term infant has just been born. Which interventions should the nurse perform first? Select all that apply. 1. Placing the infant in a radiant-heated unit 2. Suctioning the infant with a bulb syringe 3. Wrapping the infant in a blanket 4. Evaluating the newborn using the Apgar system 5. Offering a feeding of 5% glucose water
Answer: 1, 2, 4
5) Premonitory signs of labor include which of the following? Select all that apply. 1. Braxton Hicks contractions 2. Cervical softening and effacement 3. Weight gain 4. Rupture of membranes 5. Sudden loss of energy
Answer: 1, 2, 4 Explanation: 1. A premonitory sign of labor includes Braxton Hicks contractions. 2. A premonitory sign of labor includes cervical softening and effacement. 4. A premonitory sign of labor includes rupture of membranes.
21) The nurse is caring for a client in labor. Which signs and symptoms would indicate the client is progressing into the second stage of labor? Select all that apply. 1. Bulging perineum 2. Increased bloody show 3. Spontaneous rupture of the membranes 4. Uncontrollable urge to push 5. Inability to breathe through contractions
Answer: 1, 2, 4 Explanation: 1. As the fetal head continues its descent, the perineum begins to bulge, flatten, and move anteriorly. 2. Bloody show increases as a woman enters the second stage of labor. 4. As the fetal head descends, the woman has the urge to push because of pressure of the fetal head on the sacral and obturator nerves.
35) The OB-GYN nurse knows that the proliferative phase of the menstrual cycle includes which changes? Select all that apply. 1. Thin, clear cervical mucus 2. Estrogen peaks just before ovulation 3. No ferning pattern of cervical mucus 4. A pH change to 7.5 5. Epithelium warps into folds
Answer: 1, 2, 4 Explanation: 1. Cervical mucus at ovulation becomes thin, clear, watery, and more alkaline, making it more favorable to spermatozoa. 2. Estrogen peaks just before ovulation in the proliferative phase of the menstrual cycle. 4. The cervical mucosa pH increases from below 7 to 7.5 at the time of ovulation.
3) The nurse is teaching an infertile couple about the causes of infertility. The nurse tells them that infertility can be caused by which of the following? Select all that apply. 1. Immunological responses 2. Congenital anomalies 3. Patent fallopian tubes 4. Hypothyroidism 5. Favorable cervical mucus
Answer: 1, 2, 4 Explanation: 1. Immunological responses, such as antisperm antibodies, can cause infertility. 2. Congenital anomalies, such as a septate uterus, can cause infertility. 4. Hypothyroidism is a cause of infertility.
13) The nurse has written the nursing diagnosis Injury, Risk for for a diabetic pregnant client. Interventions for this diagnosis include which of the following? Select all that apply. 1. Assessment of fetal heart tones 2. Perform oxytocin challenge test, if ordered 3. Refer the client to a diabetes support group 4. Assist with the biophysical profile assessment 5. Develop an appropriate teaching plan
Answer: 1, 2, 4 Explanation: 1. Reassuring fetal heart rate variability and accelerations are interpreted as adequate placental oxygenation. 2. The nurse would perform oxytocin challenge test (OCT)/contraction stress test (CST) and non-stress tests as determined by physician. 4. The nurse assists the physician in performing a biophysical profile assessment.
8) What are the three functions of the fallopian tubes? Select all that apply. 1. Provide transport for the ovum from the ovary to the uterus 2. Serve as a warm, moist, nourishing environment for the ovum or zygote 3. Secrete large amounts of estrogens 4. Provide a site for fertilization to occur 5. Support and protect the pelvic contents
Answer: 1, 2, 4 Explanation: 1. The fallopian tubes provide transport for the ovum from the ovary to the uterus. 2. The fallopian tubes serve as a warm, moist, nourishing environment for the ovum or zygote. 4. The fallopian tubes provide a site for fertilization to occur.
10) Which of the following are specific culturally sensitive nursing considerations the nurse integrates into care of the pregnant client? Select all that apply. 1. Counseling about home remedies 2. Discussing components of a balanced diet 3. Refusing to discuss birthing choices 4. Encouraging use of support systems 5. Instructing the client to use no home remedies
Answer: 1, 2, 4 Explanation: 1. The culturally sensitive nurse should find out what medications and home remedies the client is using, and counsel the client regarding overall effects. 2. The culturally sensitive nurse will discuss the importance of a well-balanced diet during pregnancy with consideration of the client's cultural beliefs and practices. 4. Encouraging the use of support systems and spiritual aids that provide comfort for the mother is important in culturally sensitive care.
16) Remedies for back pain in pregnancy that are supported by research evidence and may safely be taught to any pregnant woman by the nurse include which of the following? Select all that apply. 1. Pelvic tilt 2. Water aerobics 3. Sit-ups 4. Proper body mechanics 5. Maintaining good posture
Answer: 1, 2, 4, 5
27) The nurse is caring for a client who is showing a sinusoidal fetal heart rate pattern on the monitor. The nurse knows that possible causes for this pattern include which of the following? Select all that apply. 1. Fetal anemia 2. Chronic fetal bleeding 3. Maternal hypotension 4. Twin-to-twin transfusion 5. Umbilical cord occlusion
Answer: 1, 2, 4, 5
3) The nurse is presenting a class on the pathophysiology of the different abortions. Some of the causes are which of the following? Select all that apply. 1. Chromosomal abnormalities 2. Insufficient or excessive hormonal levels 3. Sexual intercourse in the first trimester 4. Infections in the first trimester 5. Cervical insufficiency
Answer: 1, 2, 4, 5
30) The nurse is evaluating the plan of care for a pregnant client with a heart disorder. The nurse concludes that the plan was successful when data indicate which of the following? Select all that apply. 1. The client gave birth to a healthy baby. 2. The client did not develop congestive heart failure. 3. The client developed thromboembolism. 4. The client identified manifestations of potential complications. 5. The client can identify her condition and its impact on her pregnancy, labor and birth, and postpartum period.
Answer: 1, 2, 4, 5
33) When caring for a laboring client with oligohydramnios, what should the nurse be aware of? Select all that apply. 1. Increased risk of cord compression 2. Decreased variability 3. Labor progress is often more rapid than average 4. Presence of periodic decelerations 5. During gestation, fetal skin and skeletal abnormalities can occur
Answer: 1, 2, 4, 5
4) The nurse is planning an early-pregnancy class session on nutrition. Which information should the nurse include? Select all that apply. 1. Protein is important for fetal development. 2. Iron helps both mother and baby maintain the oxygen-carrying capacity of the blood. 3. Calcium prevents constipation at the end of pregnancy. 4. Zinc facilitates synthesis of RNA and DNA. 5. Vitamin A promotes development of the baby's eyes.
Answer: 1, 2, 4, 5
25) Ovarian hormones include which of the following? Select all that apply. 1. Estrogens 2. Progesterone 3. Parathyroid hormone 4. Luteinizing hormone 5. Testosterone
Answer: 1, 2, 5 Explanation: 1. Ovarian hormones include the estrogens, progesterone, and testosterone. 2. Ovarian hormones include the estrogens, progesterone, and testosterone. 5. Ovarian hormones include the estrogens, progesterone, and testosterone.
22) The nurse educator is teaching student nurses what a fetus will look like at various weeks of development. Which descriptions would be typical of a fetus at 20 weeks' gestation? Select all that apply. 1. The fetus has a body weight of 435-465 g. 2. Nipples appear over the mammary glands. 3. The kidneys begin to produce urine. 4. Nails are present on fingers and toes. 5. Lanugo covers the entire body.
Answer: 1, 2, 4, 5 Explanation: 1. A fetus at 20 weeks' gestation has a body weight of 435-465 g. 2. A fetus at 20 weeks' gestation has nipples appear over the mammary glands. 4. A fetus at 20 weeks' gestation nails are present on fingers and toes. 5. A fetus at 20 weeks' gestation has lanugo that covers the entire body.
2) The nurse is assessing a client before administering an analgesic. What are some of the factors the nurse should consider? Select all that apply. 1. The client is willing to receive medication after being advised about it. 2. The client's vital signs are stable. 3. The partner agrees to use of the medication. 4. The client has no known allergies to the medication. 5. The client is aware of the contraindications of the medication.
Answer: 1, 2, 4, 5 Explanation: 1. Medication should be explained to the client before it is administered. 2. Vital signs need to be stable before any analgesic medication is administered. 4. Ask the client about allergies before administering any medications. 5. Clients should always be aware of the contraindications of the medication.
28) The nurse is holding a class for newly pregnant couples, and discussing good and bad influences on their developing babies. Prenatal influences on the intrauterine environment include which of the following? Select all that apply. 1. The use of saunas or hot tubs 2. The use of drugs 3. The quality of the sperm or ovum 4. Maternal nutrition 5. Vitamins and folic acid
Answer: 1, 2, 4, 5 Explanation: 1. The use of saunas or hot tubs is associated with maternal hyperthermia. Studies of the effects of maternal hyperthermia during the first trimester have raised concern about possible spontaneous abortion, central nervous system (CNS) defects, and failure of neural tube closure. 2. The use of drugs can have teratogenic effects. 4. Maternal nutrition affects brain and neural tube development. 5. Vitamins and folic acid supplements taken before conception can reduce the incidence of neural tube defects.
27) A pregnant client calls the clinic nurse to say she is worried about symptoms she is experiencing. The nurse advises the client to come immediately to the clinic because of which reported symptoms? Select all that apply. 1. Vaginal bleeding 2. Abdominal pain 3. Constipation 4. Epigastric pain 5. Blurring of vision
Answer: 1, 2, 4, 5 Explanation: 1. Vaginal bleeding can indicate abruptio placentae, placenta previa, or lesions of cervix or vagina, or it can be "bloody show," and requires that the client be seen. 2. Abdominal pain can signal premature labor or abruptio placentae, and requires that the client be seen. 4. Epigastric pain must be evaluated, as it can indicate preeclampsia or ischemia in a major abdominal vessel. 5. Dizziness, blurring of vision, double vision, or spots before the eyes can indicate either hypertension and/or preeclampsia and requires the client be seen.
28) Fetal factors that possibly indicate electronic fetal monitoring include which of the following? 1. Meconium passage 2. Multiple gestation 3. Preeclampsia 4. Grand multiparity 5. Decreased fetal movement
Answer: 1, 2, 5
6) The client presents to the labor and delivery unit stating that her water broke 2 hours ago. Indicators of normal labor include which of the following? Select all that apply. 1. Fetal heart rate of 130 with average variability 2. Blood pressure of 130/80 3. Maternal pulse of 160 4. Protein of +1 in urine 5. Odorless, clear fluid on underwear
Answer: 1, 2, 5
23) The client and her partner are carriers of sickle-cell disease. They are considering prenatal diagnosis with either amniocentesis or chorionic villus sampling (CVS). Which statements indicate that further teaching is needed on these two diagnostic procedures? Select all that apply. 1. "Chorionic villus sampling carries a lower risk of miscarriage." 2. "Amniocentesis can be done earlier in my pregnancy than CVS." 3. "Neither test will conclusively diagnose sickle-cell disease in our baby." 4. "The diagnosis comes sooner if we have CVS, not amniocentesis." 5. "Amniocentesis is more accurate in diagnosis than the CVS."
Answer: 1, 2, 5 Explanation: 1. CVS has a risk of spontaneous abortion of 0.3% in cases. This rate is higher than second trimester amniocentesis. 2. CVS is performed in some medical centers for first trimester diagnosis after 9 completed weeks. Amniocentesis is performed between 15 and 20 weeks' gestation. 5. Much like amniocentesis, chorionic villus sampling (CVS) is a procedure that is used to detect genetic, metabolic, and DNA abnormalities. CVS permits earlier diagnosis than can be obtained by amniocentesis.
3) A client in labor is requesting pain medication. The nurse assesses her labor status first, focusing on which of the following? Select all that apply. 1. Contraction pattern 2. Amount of cervical dilatation 3. When the labor began 4. Whether the membranes are intact or ruptured 5. Fetal presenting part
Answer: 1, 2, 5 Explanation: 1. The client should have a good contraction pattern before receiving an analgesic. 2. The nurse should evaluate the amount of cervical dilatation before analgesic medication is administered. 5. If normal parameters are absent or if nonreassuring maternal or fetal factors are present, the nurse may need to complete further assessments with the physician/CNM.
31) When caring for a 13-year-old client in labor, how would the nurse provide sensitive care? Select all that apply. 1. Using simple and concrete instructions 2. Providing soothing encouragement and comfort measures 3. Making all decisions for the client when she expresses a feeling of helplessness 4. Deciding whom the client should allow in the room 5. Providing encouragement and support of the client's decisions
Answer: 1, 2, 5 Explanation: 1. A client at this developmental stage will need concrete and simplified instructions. 2. Touch, soothing encouragement, and measures to promote her comfort help her maintain control and meet her needs for dependence. 5. Establishing rapport without recrimination will provide emotional support and encouragement.
33) The client at 34 weeks' gestation has been stabbed in the low abdomen by her boyfriend. She is brought to the emergency department for treatment. Which statements indicate that the client understands the treatment being administered? Select all that apply. 1. "The baby needs to be monitored to check the heart rate." 2. "My bowel has probably been lacerated by the knife." 3. "I might need an ultrasound to look at the baby." 4. "The catheter in my bladder will prevent urinary complications." 5. "The IV in my arm will replace the amniotic fluid if it is leaking."
Answer: 1, 3
21) During the history, the client admits to being HIV-positive and says she knows that she is about 16 weeks pregnant. Which statements made by the client indicate an understanding of the plan of care both during the pregnancy and postpartally? Select all that apply. 1. "During labor and delivery, I can expect the zidovudine (ZDV) to be given in my IV." 2. "After delivery, the dose of zidovudine (ZDV) will be doubled to prevent further infection." 3. "My baby will be started on zidovudine (ZDV) for six weeks following the birth." 4. "My baby's zidovudine (ZDV) will be given in a cream form." 5. "My baby will not need zidovudine (ZDV) if I take it during my pregnancy."
Answer: 1, 3 Explanation: 1. ART therapy generally it includes oral Zidovudine (ZDV) daily, IV ZDV during labor and until birth, and ZDV therapy for the infant for 6 weeks following birth. 3. ART therapy generally it includes oral Zidovudine (ZDV) daily, IV ZDV during labor and until birth, and ZDV therapy for the infant for 6 weeks following birth.
3) The pregnant client states she does not want "to take all these supplements." What recommendations could the nurse make for the client? Select all that apply. 1. "Folic acid has been found to be essential for minimizing the risk of neural tube defects." 2. "You do not have to take these supplements if you think you are healthy enough." 3. "Most women do not have adequate intake of iron pre-pregnancy, and iron needs increase with pregnancy." 4. "These medications do the same thing. I will call your physician to cancel one of your medications." 5. "You should take the folic acid, but the vitamins are not that important."
Answer: 1, 3 Explanation: 1. An inadequate intake of folic acid has been associated with neural tube defects (NTDs) (e.g., spina bifida, anencephaly). 3. Iron is essential because many pregnant women do not have adequate intake of iron before pregnancy.
20) A client had an epidural inserted 2 hours ago. It is functioning well, the client is stable, and labor is progressing. Which parts of the nurse's assessment have the highest priority? Select all that apply. 1. Assess blood pressure every hour. 2. Assess the pulse rate every hour. 3. Palpate the bladder. 4. Auscultate the lungs. 5. Assess the reflexes.
Answer: 1, 3 Explanation: 1. Blood pressure should be monitored every 1 to 2 minutes for the first 10 minutes and then every 5 to 15 minutes until the block wears off because hypotension is the most common side effect of epidural anesthesia. 3. Nursing care following an epidural block includes frequent assessment of the bladder to avoid bladder distention. Catheterization may be necessary, because most women are unable to void.
13) Before performing Leopold maneuvers, what would the nurse do? Select all that apply. 1. Have the client empty her bladder. 2. Place the client in Trendelenburg position. 3. Have the client lie on her back with her feet on the bed and knees bent. 4. Turn the client to her left side. 5. Have the client lie flat with her ankles crossed.
Answer: 1, 3 Explanation: 1. The woman should have recently emptied her bladder before performing Leopold maneuvers. 3. The woman should lie on her back with her abdomen uncovered. To aid in relaxation of the abdominal wall, the shoulders should be raised slightly on a pillow and the knees drawn up a little.
35) A pregnant client at 30 weeks' gestation has had a steady rise in blood pressure. She is now 20 mmHg above her systolic baseline. The nurse advises her to immediately report which symptoms? Select all that apply. 1. Dizziness 2. Even a small amount of dependent edema 3. Spots before her eyes 4. Persistent nausea and vomiting 5. Vaginal spotting
Answer: 1, 3 Explanation: 1. Dizziness can be a sign of hypertension or preeclampsia, and should be reported immediately. 3. Spots before the eyes can be a sign of hypertension or preeclampsia, and should be reported immediately.
20) The nurse is explaining clinical pelvimetry to a client. The nurse explains that the anteroposterior diameters consist of which of the following? Select all that apply. 1. Diagonal conjugate 2. Transverse diameter 3. Conjugata vera 4. Obstetric conjugate 5. Oblique diameter
Answer: 1, 3, 4
9) Postpartum nutritional status is determined primarily by assessing which of the following? Select all that apply. 1. Dietary history 2. Menstrual history 3. Mother's weight 4. Hemoglobin levels 5. Mother's height
Answer: 1, 3, 4
2) During a client's initial prenatal visit, the nurse must assess and document the client's current medical history, including which information? Select all that apply. 1. Body mass index 2. Infections before the last menstrual period 3. Homeopathic or herbal medication use 4. Blood type and Rh factor 5. History of previous pregnancies
Answer: 1, 3, 4 Explanation: 1. The body mass index is an important part of the current medical history to be assessed and documented. 3. Homeopathic and herbal medication use is important for the nurse to assess and document in the current medical history. 4. The blood type must be assessed and documented in the current medical history, as must the Rh factor.
9) The nurse is assessing a client in the third trimester of pregnancy. What physiologic changes in the client are expected? Select all that apply. 1. The client's chest circumference has increased by 6 cm during the pregnancy. 2. The client has a narrowed subcostal angle. 3. The client is using thoracic breathing. 4. The client may have epistaxis. 5. The client has a productive cough.
Answer: 1, 3, 4 Explanation: 1. The chest increase compensates for the elevated diaphragm. 3. Breathing changes from abdominal to thoracic as pregnancy progresses. 4. Epistaxis (nosebleeds) may occur and are primarily the result of estrogen-induced edema and vascular congestion of the nasal mucosa.
28) The nurse in the OB-GYN clinic counsels a couple that in autosomal dominant inheritance, which of the following occur? Select all that apply. 1. An affected individual might have an affected parent. 2. The affected individual has a 75% chance of passing on the abnormality. 3. Males and females are equally affected by the gene. 4. A father can pass the defective gene to a son. 5. There are no variances in the genetic pattern for autosomal dominant disorders.
Answer: 1, 3, 4 Explanation: 1. The family pedigree usually shows multiple generations having the disorder. 3. Males and females are equally affected in autosomal dominant disorders. 4. In autosomal dominant disorders, the father can pass the defective gene to a son.
1) The nurse has admitted a woman with cervical insufficiency. The nurse is aware that causes of this condition include which of the following? Select all that apply. 1. Congenital factors 2. Intercourse during pregnancy 3. Infection 4. Increased uterine volume 5. Past cervical surgeries
Answer: 1, 3, 4, 5
3) Usually, the family is advised to arrive at the birth setting at the beginning of the active phase of labor or when which of the following occur? Select all that apply. 1. Rupture of membranes (ROM) 2. Increased fetal movement 3. Decreased fetal movement 4. Any vaginal bleeding 5. Regular, frequent uterine contractions (UCs)
Answer: 1, 3, 4, 5
36) The nurse is planning care for a client with hydramnios. For which interventions might the nurse need to prepare the client? Select all that apply. 1. Artificial rupture of the membranes 2. Amnioinfusion 3. Amniocentesis 4. Administration of prostaglandin synthesis inhibitor 5. Administration of indomethacin
Answer: 1, 3, 4, 5
35) The OB-GYN nurse teaches a first-time pregnant client that functions of the amniotic fluid include which of the following? Select all that apply. 1. Allowing fetal freedom of movement 2. Releasing heat to control fetal temperature 3. Acting as an extension of fetal extracellular space 4. Providing a water source for the fetus to swallow 5. Acting as a wedge during labor
Answer: 1, 3, 5 Explanation: 1. Amniotic fluid permits fetal freedom of movement. 3. A primary function of amniotic fluid is to act as an extension of fetal extracellular space. 5. A primary function of amniotic fluid is to act as a wedge during labor.
24) When general anesthesia is necessary for a cesarean delivery, what should the nurse be prepared to do? Select all that apply. 1. Administer an antacid to the client. 2. Place a wedge under her thigh. 3. Apply cricoid pressure during anesthesia intubation. 4. Preoxygenate for 3-5 minutes before anesthesia. 5. Place a Foley catheter in the client's bladder.
Answer: 1, 3, 4, 5 Explanation: 1. Prophylactic antacid therapy is given to reduce the acidic content of the stomach before general anesthesia. 3. During the process of rapid induction of anesthesia, the nurse applies cricoid pressure. 4. The woman should be preoxygenated with 3 to 5 minutes of 100% oxygen. 5. Urinary retention can be treated with the placement of an indwelling Foley catheter.
2) The nurse understands that a client's pregnancy is progressing normally when what physiologic changes are documented on the prenatal record of a woman at 36 weeks' gestation? Select all that apply. 1. The joints of the pelvis have relaxed, causing a waddling gait. 2. The cervix is firm and blue-purple in color. 3. The uterus vasculature contains one sixth of the total maternal blood volume. 4. Gastric emptying time is delayed, and the client complains of constipation and bloating. 5. Supine hypotension occurs when the client lies on her back.
Answer: 1, 3, 4, 5 Explanation: 1. The sacroiliac, sacrococcygeal, and pubic joints of the pelvis relax in the later part of the pregnancy, presumably as a result of hormonal changes. This often causes a waddling gait. 3. By the end of pregnancy, one sixth of the total maternal blood volume is contained within the vascular system of the uterus. 4. Gastric emptying time and intestinal motility are delayed, leading to frequent complaints of bloating and constipation, which can be aggravated by the smooth muscle relaxation and increased electrolyte and water reabsorption in the large intestine. 5. The enlarging uterus may exert pressure on the vena cava when the woman lies supine, causing a drop in blood pressure. This is called the vena caval syndrome, or supine hypotension.
29) Women with eating disorders who become pregnant are at risk for a variety of complications including which of the following? Select all that apply. 1. Premature birth 2. Too many nutrients available for the fetus 3. Miscarriage 4. High birth weight 5. Perinatal mortality
Answer: 1, 3, 5
32) The clinic nurse is assessing how the prenatal client is meeting developmental tasks using Rubin's tasks, including which of the following? Select all that apply. 1. Ensuring safe passage through pregnancy, labor, and birth. 2. Turning in on oneself to focus on the child. 3. Seeking commitment and acceptance of self as mother to the infant. 4. Completing the tasks of nesting at the appropriate time. 5. Seeking acceptance of the child by others.
Answer: 1, 3, 5
A client at 18 weeks' gestation has been diagnosed with a hydatidiform mole. In addition to vaginal bleeding, which signs or symptoms would the nurse expect to see? Select all that apply. 1. Hyperemesis gravidarum 2. Diarrhea and hyperthermia 3. Uterine enlargement greater than expected 4. Polydipsia 5. Vaginal bleeding
Answer: 1, 3, 5
9) A first-time 22-year-old single labor client, accompanied by her boyfriend, is admitted to the labor unit with ruptured membranes and mild to moderate contractions. She is determined to be 2 centimeters dilated. Which nursing diagnoses might apply during the current stage of labor? Select all that apply. 1. Fear/Anxiety related to discomfort of labor and unknown labor outcome 2. Knowledge, Deficient, related to lack of information about pushing methods 3. Pain, Acute, related to uterine contractions, cervical dilatation, and fetal descent 4. Pain, Acute, related to perineal trauma 5. Coping: Family, Compromised, related to labor process
Answer: 1, 3, 5 Explanation: 1. A Fear/Anxiety diagnosis would apply to the first stage of labor for a first-time labor client. 3. Contractions become more regular in frequency and duration, increasing discomfort and pain. 5. The woman and her boyfriend are about to undergo one of the most meaningful and stressful events in life together. Physical and psychologic resources, coping mechanisms, and support systems will all be challenged.
15) Student nurses in their obstetrical rotation are learning about fertilization and implantation. The process of implantation is characterized by which statements? Select all that apply. 1. The trophoblast attaches itself to the surface of the endometrium. 2. The most frequent site of attachment is the lower part of the anterior uterine wall. 3. Between days 7 and 10 after fertilization, the zona pellucida disappears, and the blastocyst implants itself by burrowing into the uterine lining. 4. The lining of the uterus thins below the implanted blastocyst. 5. The cells of the trophoblast grow down into the uterine lining, forming the chorionic villi.
Answer: 1, 3, 5 Explanation: 1. During implantation, the trophoblast attaches itself to the surface of the endometrium for further nourishment. 3. Between days 7 and 10 after fertilization, the zona pellucida disappears, and the blastocyst implants itself by burrowing into the uterine lining and penetrating down toward the maternal capillaries until it is completely covered. 5. The cells of the trophoblast grow down into the thickened lining, forming the chorionic villi.
20) A client at 34 weeks' gestation complains about pyrosis. The nurse teaches the patient that approaches to relieve the pyrosis include which of the following? Select all that apply. 1. Eat small, frequent meals 2. Use high-sodium antacids 3. Avoid fried, fatty foods 4. Take sodium bicarbonate after meals 5. Do not lie down after eating
Answer: 1, 3, 5 Explanation: 1. Pyrosis (heartburn) can be relieved by eating small, more frequent meals.
14) The nurse is teaching a group of adolescents that sperm must undergo the process of capacitation in order to fertilize the ova. The characteristics of sperm that have undergone capacitation include which of the following? Select all that apply. 1. Ability to undergo an acrosomal reaction 2. Ability to block polyspermy 3. Ability to bind to the zona pellucida 4. Ability to release norepinephrine 5. Acquisition of hypermotility
Answer: 1, 3, 5 Explanation: 1. Sperm that undergo capacitation take on the ability to undergo the acrosomal reaction. 3. Sperm that undergo capacitation take on the ability to bind to the zona pellucida. 5. Sperm that undergo capacitation have the acquisition of hypermotility.
26) The nurse recognizes the importance of the interaction between the nervous and endocrine systems in the female reproductive cycle. The interaction involves which of the following? Select all that apply. 1. Hypothalamus 2. Adrenal cortex 3. Ovaries 4. Thyroid 5. Anterior pituitary
Answer: 1, 3, 5 Explanation: 1. The female reproductive cycle is controlled by complex interactions between the nervous and endocrine systems and their target tissues. These interactions involve the hypothalamus. 3. The female reproductive cycle is controlled by complex interactions between the nervous and endocrine systems and their target tissues. These interactions involve the ovaries. 5. The female reproductive cycle is controlled by complex interactions between the nervous and endocrine systems and their target tissues. These interactions involve the anterior pituitary.
2) A clinic nurse is preparing diagrams of pelvic shapes. Which pelvic shapes are considered least adequate for vaginal childbirth? Select all that apply. 1. Android 2. Anthropoid 3. Gynecoid 4. Platypelloid 5. Lambdoidal suture
Answer: 1, 4 Explanation: 1. In the android and platypelloid types, the pelvic diameters are diminished. Labor is more likely to be difficult (longer) and a cesarean birth is more likely. 4. In the android and platypelloid types, the pelvic diameters are diminished. Labor is more likely to be difficult (longer) and a cesarean birth is more likely.
26) To answer a client's question about home pregnancy tests and their accuracy, the nurse must know that accuracy is affected by which of the following? Select all that apply. 1. Unclear directions 2. Unable to comprehend the directions 3. Blood in the specimen giving a false reading 4. Completing the test too late 5. Tagged antibodies becoming outdated
Answer: 1, 4 Explanation: 1. Women may not comprehend the HPT instructions, which can affect the accuracy results. 4. False-negative results typically occur when the test is completed too early or too late.
13) A client who is having false labor most likely would have which of the following? Select all that apply. 1. Contractions that do not intensify while walking 2. An increase in the intensity and frequency of contractions 3. Progressive cervical effacement and dilatation 4. Pain in the abdomen that does not radiate 5. Contractions that lessen with rest and warm tub baths
Answer: 1, 4, 5
12) A client at 32 weeks' gestation is admitted with painless vaginal bleeding. Placenta previa has been confirmed by ultrasound. What should be included in the nursing plan? Select all that apply. 1. No vaginal exams 2. Encouraging activity 3. No intravenous access until labor begins 4. Evaluating fetal heart rate with an external monitor 5. Monitoring blood loss, pain, and uterine contractility
Answer: 1, 4, 5 Explanation: 1. Expectant management of placenta previa is made by localizing the placenta via tests that require no vaginal examination. 4. Expectant management of placenta previa, when the client is at less than 37 weeks' gestation, includes evaluating FHR with an external monitor. 5. Expectant management of placenta previa, when the client is at less than 37 weeks' gestation, includes monitoring blood loss, pain, and uterine contractility.
32) The clinic nurse assesses a newborn that is not progressing as expected. Genetic tests are ordered. The nurse explains to the parents that the laboratory tests to be done include which of the following? Select all that apply. 1. Chromosome analysis 2. Complete blood count 3. Phenylketonuria 4. Enzyme assay 5. Antibody titers
Answer: 1, 4, 5 Explanation: 1. Laboratory analysis includes chromosome analysis. 4. Laboratory analysis includes enzyme assay for inborn errors of metabolism. 5. Laboratory analysis includes antibody titers for infectious teratogens.
34) A client is admitted to the labor suite. It is essential that the nurse assess the woman's status in relation to which infectious diseases? Select all that apply. 1. Chlamydia trachomatis 2. Rubeola 3. Varicella 4. Group B streptococcus 5. Acute pyelonephritis
Answer: 1, 4, 5 Explanation: 1. The infant may develop chlamydial pneumonia and Chlamydia trachomatis may be responsible for premature labor and fetal death. Chlamydial infection should be assessed. 4. Women may transmit GBS to their fetus in utero or during childbirth. GBS is a leading infectious cause of neonatal sepsis and mortality and should be assessed. 5. Acute pyelonephritis should be assessed as there is an increased risk of premature birth and intrauterine growth restriction (IUGR).
27) The pregnant teen who was prescribed prenatal vitamins at her initial prenatal visit states that she does not like to take them. How should the nurse respond? Select all that apply. 1. "Folic acid has been found to be essential for minimizing the risk of neural tube defects." 2. "You do not have to take these supplements if you think you are healthy enough." 3. "These medications do the same thing. I will call your doctor to cancel one of your medications." 4. "You can trust your doctor to know what you need." 5. "You need the supplements because your dietary intake may not be adequate for fetal development."
Answer: 1, 5 Explanation: 1. The CDC estimates that most neural tube defects could be prevented if women followed folic acid supplementation recommendations before they know they are pregnant. 5. One role of the nurse is educator, and this client needs additional information on why she needs the supplements. This response answers the client's concerns.
23) The nurse is caring for a client in the transition phase of labor and notes that the fetal monitor tracing shows average short-term and long-term variability with a baseline of 142 beats per minute. What actions should the nurse take in this situation? Select all that apply. 1. Provide caring labor support. 2. Administer oxygen via face mask. 3. Change the client's position. 4. Speed up the client's intravenous. 5. Reassure the client and her partner that she is doing fine.
Answer: 1, 5 Explanation: 1. The tracing is normal, so the nurse can continue support of the labor. 5. The nurse can reassure the client at this time, as the tracing is normal.
10) A 26-year-old client is 28 weeks pregnant. She has developed gestational diabetes. She is following a program of regular exercise, which includes walking, bicycling, and swimming. What instructions should be included in a teaching plan for this client? 1. "Exercise either just before meals or wait until 2 hours after a meal." 2. "Carry hard candy (or other simple sugar) when exercising." 3. "If your blood sugar is 120 mg/dL, eat 20 g of carbohydrate." 4. "If your blood sugar is more than 120 mg/dL, drink a glass of whole milk."
Answer: 2
10) The breastfeeding mother is concerned that her milk production has decreased. The nurse knows that further client teaching is needed based on which statement? 1. "I am drinking a minimum of 8 to 10 glasses of liquid a day." 2. "I have started cutting back on my protein intake." 3. "At least three times a day, I drink a glass of milk." 4. "My calorie intake is higher than during the pregnancy."
Answer: 2
11) The prenatal clinic nurse is caring for a client with hyperemesis gravidarum at 14 weeks' gestation. The vital signs are: blood pressure 95/48, pulse 114, respirations 24. Which order should the nurse implement first? 1. Weigh the client. 2. Give 1 liter of lactated Ringer's solution IV. 3. Administer 30 mL Maalox (magnesium hydroxide) orally. 4. Encourage clear liquids orally.
Answer: 2
16) A client presents to the antepartum clinic with a history of a 20-pound weight loss. Her pregnancy test is positive. She is concerned about gaining the weight back, and asks the nurse if she can remain on her diet. What is the nurse's best response? 1. "As long as you supplement your diet with the prenatal vitamin, the amount of weight you gain in pregnancy is not significant." 2. "I understand that gaining weight after such an accomplishment might not look attractive, but weight gain during pregnancy is important for proper fetal growth." 3. "Dieting during pregnancy is considered child neglect." 4. "Excessive weight gain in pregnancy is due to water retention, so weight loss following birth will not be an issue."
Answer: 2
34) Nurses who are interacting with expectant families from a different culture or ethnic group can provide more effective, culturally sensitive nursing care by doing what? 1. Recognizing that ultimately it is the family's right to make a woman's healthcare choices. 2. Obtaining a medical interpreter of the language the client speaks. 3. Evaluating whether the client's healthcare beliefs have any positive consequences for her health. 4. Accepting personal biases, attitudes, stereotypes, and prejudices.
Answer: 2
5) Which of the following tests provides information about the fetal number? 1. Amniocentesis 2. Standard second-trimester sonogram 3. Beta hCG 4. Maternal serum alpha-fetoprotein
Answer: 2
6) A woman is 16 weeks pregnant. She has had cramping, backache, and mild bleeding for the past 3 days. Her physician determines that her cervix is dilated to 2 centimeters, with 10% effacement, but membranes are still intact. She is crying, and says to the nurse, "Is my baby going to be okay?" In addition to acknowledging the client's fear, what should the nurse also say? 1. "Your baby will be fine. We'll start IV, and get this stopped in no time at all." 2. "Your cervix is beginning to dilate. That is a serious sign. We will continue to monitor you and the baby for now." 3. "You are going to miscarry. But you should be relieved because most miscarriages are the result of abnormalities in the fetus." 4. "I really can't say. However, when your physician comes, I'll ask her to talk to you about it."
Answer: 2
16) After several hours of labor, the electronic fetal monitor (EFM) shows repetitive variable decelerations in the fetal heart rate. The nurse would interpret the decelerations to be consistent with which of the following? 1. Breech presentation 2. Uteroplacental insufficiency 3. Compression of the fetal head 4. Umbilical cord compression
Answer: 4
17) A client in her third trimester of pregnancy reports frequent leg cramps. What strategy would be most appropriate for the nurse to suggest? 1. Point the toes of the affected leg 2. Increase intake of protein-rich foods 3. Limit activity for several days 4. Flex the foot to stretch the calf
Answer: 4
20) The nurse is reviewing amniocentesis results. Which of the following would indicate that client care was appropriate? 1. The client who is Rh-positive received Rh immune globulin after the amniocentesis. 2. The client was monitored for 30 minutes after completion of the test. 3. The client began vaginal spotting before leaving for home after the test. 4. The client identified that she takes insulin before each meal and at bedtime.
Answer: 2 Explanation: 2. 20 to 30 minutes of fetal monitoring is performed after the amniocentesis.
28) A nurse examining a prenatal client recognizes that a lag in progression of measurements of fundal height from week to week and month to month could signal what condition? 1. Twin pregnancy 2. Intrauterine growth restriction 3. Hydramnios 4. Breech position
Answer: 2 Explanation: 2. A lag in progression of measurements of fundal height from month to month could signal intrauterine growth restriction (IUGR).
29) Slowly removing some amniotic fluid is a treatment for hydramnios. What consequence can occur with the withdrawal of fluid? 1. Preterm labor 2. Prolapsed cord 3. Preeclampsia 4. Placenta previa
Answer: 2 Explanation: 2. A needle or a fetal scalp electrode is used to make a small puncture in the amniotic sac. There is a risk that the force of the fluid could make a larger hole in the amniotic sac, thus increasing the risk of a prolapsed cord.
21) The nurse is assessing a newly pregnant client. Which finding does the nurse note as a normal psychosocial adjustment in this client's first trimester? 1. An unlisted telephone number 2. Reluctance to tell the partner of the pregnancy 3. Parental disapproval of the woman's partner 4. Ambivalence about the pregnancy
Answer: 4
14) A client at 37 weeks' gestation has a mildly elevated blood pressure. Her antenatal testing demonstrates three contractions in 10 minutes, no decelerations, and accelerations four times in 1 hour. What would this test be considered? 1. Positive non-stress test 2. Negative contraction stress test 3. Positive contraction stress test 4. Negative non-stress test
Answer: 2 Explanation: 2. A negative CST shows three contractions of good quality lasting 40 or more seconds in 10 minutes without evidence of late decelerations. This is the desired result.
11) Narcotic analgesia is administered to a laboring client at 10:00 a.m. The infant is delivered at 12:30 p.m. What would the nurse anticipate that the narcotic analgesia could do? 1. Be used in place of preoperative sedation 2. Result in neonatal respiratory depression 3. Prevent the need for anesthesia with an episiotomy 4. Enhance uterine contractions
Answer: 2 Explanation: 2. Analgesia given too late is of no value to the woman and may cause neonatal respiratory depression.
10) A 25-year-old primigravida is at 20 weeks' gestation. The nurse takes her vital signs and notifies the healthcare provider immediately because of which finding? 1. Pulse 88/minute 2. Rhonchi in both bases 3. Temperature 37.4° C (99.3° F) 4. Blood pressure 122/78
Answer: 2 Explanation: 2. Any abnormal breath sounds should be reported to the healthcare provider.
28) Upon delivery of the newborn, what nursing intervention most promotes parental attachment? 1. Placing the newborn under the radiant warmer. 2. Placing the newborn on the mother's abdomen. 3. Allowing the mother a chance to rest immediately after delivery. 4. Taking the newborn to the nursery for the initial assessment.
Answer: 2 Explanation: 2. As the baby is placed on the mother's abdomen or chest, she frequently reaches out to touch and stroke her baby. When the newborn is placed in this position, the father or partner also has a very clear, close view and can also reach out to touch the baby.
26) The labor and delivery nurse is reviewing charts. The nurse should inform the supervisor about which client? 1. Client at 5 cm requesting labor epidural analgesia 2. Client whose cervix remains at 6 cm for 4 hours 3. Client who has developed nausea and vomiting 4. Client requesting her partner to stay with her
Answer: 2 Explanation: 2. Average cervical change in the active phase of the first stage of labor is 1.2 cm/hour; thus, this client's lack of cervical change is unexpected, and should be reported to the supervisor.
25) A woman in labor asks the nurse to explain the electronic fetal heart rate monitor strip. The fetal heart rate baseline is 150 with accelerations to 165, variable decelerations to 140, and moderate long-term variability. Which statement indicates that the client understands the nurse's teaching? 1. "The most important part of fetal heart monitoring is the absence of variable decelerations." 2. "The most important part of fetal heart monitoring is the presence of variability." 3. "The most important part of fetal heart monitoring is the fetal heart rate baseline." 4. "The most important part of fetal heart monitoring is the depth of decelerations."
Answer: 2 Explanation: 2. Baseline variability is a reliable indicator of fetal cardiac and neurologic function and well-being. The opposing "push-pull" balancing between the sympathetic nervous system and the parasympathetic nervous system directly affects the FHR.
15) The laboring client is having moderately strong contractions lasting 60 seconds every 3 minutes. The fetal head is presenting at a -2 station. The cervix is 6 cm and 100% effaced. The membranes spontaneously ruptured prior to admission, and clear fluid is leaking. Fetal heart tones are in the 140s with accelerations to 150. Which nursing action has the highest priority? 1. Encourage the husband to remain in the room. 2. Keep the client on bed rest at this time. 3. Apply an internal fetal scalp electrode. 4. Obtain a clean-catch urine specimen.
Answer: 2 Explanation: 2. Because the membranes are ruptured and the head is high in the pelvis at a -2 station, the client should be maintained on bed rest to prevent cord prolapse.
24) The nurse is presenting a class to pregnant clients. The nurse asks, "The fetal brain is developing rapidly, and the nervous system is complete enough to provide some regulation of body function on its own, at which fetal development stage?" It is clear that education has been effective when a participant makes which response? 1. "The 17th-20th week" 2. "The 25th-28th week" 3. "The 29th-32nd week" 4. "The 33rd-36th week"
Answer: 2 Explanation: 2. Between the 25th and 28th week, the brain is developing rapidly, and the nervous system is complete enough to provide some degree of regulation of body functions.
9) A newly diagnosed insulin-dependent type 1 diabetic with good blood sugar control is at 20 weeks' gestation. She asks the nurse how her diabetes will affect her baby. What would the best explanation include? 1. "Your baby could be smaller than average at birth." 2. "Your baby will probably be larger than average at birth." 3. "As long as you control your blood sugar, your baby will not be affected at all." 4. "Your baby might have high blood sugar for several days."
Answer: 2 Explanation: 2. Characteristically, infants of mothers with diabetes are large for gestational age (LGA) as a result of high levels of fetal insulin production stimulated by the high levels of glucose crossing the placenta from the mother. Sustained fetal hyperinsulinism and hyperglycemia ultimately lead to excessive growth, called macrosomia, and deposition of fat.
15) When planning care for a client who has undergone an episiotomy, it would be important for the nurse to include a goal that addresses the need for pain relief of which part of the body? 1. Mons pubis 2. Perineal body 3. Labia minora 4. Hymen
Answer: 2 Explanation: 2. During the last part of labor, the perineal body thins out until it is just a few centimeters thick. This tissue is often the site of an episiotomy or lacerations during childbirth.
13) Which of the following is important for the development of the central nervous system of the fetus? 1. Calcium and phosphorus 2. Essential fatty acids 3. Iron 4. Vitamin D
Answer: 2 Explanation: 2. Essential fatty acids are important for the development of the central nervous system of the fetus. Of particular interest are the omega-3 fatty acids and their derivatives.
22) What can be determined based on ultrasound visualization or the lack of visualization of an intertwin membrane? 1. Toxicity 2. Amnionicity 3. Variability 4. Prematurity
Answer: 2 Explanation: 2. Evidence supports the use of ultrasound for accurately determining chorionicity and amnionicity in multiple pregnancies. Determination of amnionicity is based on ultrasound visualization or the lack of visualization of an intertwin membrane.
25) The client at 39 weeks' gestation is undergoing a cesarean birth due to breech presentation. General anesthesia is being used. Which situation requires immediate intervention? 1. The baby's hands and feet are blue at 1 minute after birth. 2. The fetal heart rate is 70 prior to making the skin incision. 3. Clear fluid is obtained from the baby's oropharynx. 4. The neonate cries prior to delivery of the body.
Answer: 2 Explanation: 2. Fetal bradycardia occurs when the fetal heart rate falls below 110 beats/minute during a 10-minute period of continuous monitoring. When fetal bradycardia is accompanied by decreased variability, it is considered ominous and could be a sign of fetal compromise.
16) A client who is experiencing her first pregnancy has just completed the initial prenatal examination with a certified nurse-midwife. Which statement indicates that the client needs additional information? 1. "Because we heard the baby's heartbeat, I am undoubtedly pregnant." 2. "Because I haven't felt the baby move yet, we don't know whether I'm pregnant." 3. "My last period was 2 months ago, which means I'm 2 months along." 4. "The increased size of my uterus means that I am finally pregnant."
Answer: 2 Explanation: 2. Fetal movement is a subjective, or presumptive, change of pregnancy, and is not a reliable indicator in the early months of pregnancy.
33) The client is in the second stage of labor. The fetal heart rate baseline is 170, with minimal variability present. The nurse performs fetal scalp stimulation. The client's partner asks why the nurse did that. What is the best response by the nurse? 1. "I stimulated the top of the fetus's head to wake him up a little." 2. "I stimulated the top of the fetus's head to try to get his heart rate to accelerate." 3. "I stimulated the top of the fetus's head to calm the fetus down before birth." 4. "I stimulated the top of the fetus's head to find out whether he is in distress."
Answer: 2 Explanation: 2. Fetal scalp stimulation is done when there is a question regarding fetal status. An acceleration indicates fetal well-being.
6) During a class on genetics for pregnant families, the nurse is discussing the how the egg and sperm are formed before fertilization takes place. The nurse explains that these cells have only half the number of chromosomes, so when fertilization takes place, there will be the correct number. What is the process by which the egg and sperm are formed called? 1. Oogenesis 2. Gametogenesis 3. Meiosis 4. Spermatogenesis
Answer: 2 Explanation: 2. Gametogenesis is the process by which ovum and sperm are produced.
5) The nurse is assessing a client who has severe preeclampsia. What assessment finding should be reported to the physician? 1. Excretion of less than 300 mg of protein in a 24-hour period 2. Platelet count of less than 100,000/mm3 3. Urine output of 50 mL per hour 4. 12 respirations
Answer: 2 Explanation: 2. HELLP syndrome (hemolysis, elevated liver enzymes, and low platelet count) complicates 10% to 20% of severe preeclampsia cases and develops prior to 37 weeks' gestation 50% of the time. Vascular damage is associated with vasospasm, and platelets aggregate at sites of damage, resulting in low platelet count (less than 100,000/mm3).
20) The client at 9 weeks' gestation has been told that her HIV test was positive. The client is very upset, and tells the nurse, "I didn't know I had HIV! What will this do to my baby?" The nurse knows teaching has been effective when the client makes which statement? 1. "I cannot take the medications that control HIV during my pregnancy, because they will harm the baby." 2. "My baby can get HIV during the pregnancy and through my breast milk." 3. "The pregnancy will increase the progression of my disease and will reduce my CD4 counts." 4. "The HIV won't affect my baby, and I will have a low-risk pregnancy without additional testing."
Answer: 2 Explanation: 2. HIV transmission can occur during pregnancy and through breast milk; however, it is believed that the majority of all infections occur during labor and birth.
7) A client at 16 weeks' gestation has a hematocrit of 35%. Her prepregnancy hematocrit was 40%. Which statement by the nurse best explains this change? 1. "Because of your pregnancy, you're not making enough red blood cells." 2. "Because your blood volume has increased, your hematocrit count is lower." 3. "This change could indicate a serious problem that might harm your baby." 4. "You're not eating enough iron-rich foods like meat."
Answer: 2 Explanation: 2. Hemoglobin and hematocrit levels drop in early to mid-pregnancy as a result of pregnancy-associated hemodilution. Because the plasma volume increase (50%) is greater than the erythrocyte increase (25%), the hematocrit decreases slightly.
27) A client is at 12 weeks' gestation with her first baby. She has cardiac disease, class III. She states that she had been taking sodium warfarin (Coumadin), but her physician changed her to heparin. She asks the nurse why this was done. What should the nurse's response be? 1. "Heparin is used when coagulation problems are resolved." 2. "Heparin is safer because it does not cross the placenta." 3. "They are the same drug, but heparin is less expensive." 4. "Coumadin interferes with iron absorption in the intestines."
Answer: 2 Explanation: 2. Heparin is safest for the client to take because it does not cross the placental barrier.
32) If oligohydramnios occurs in the first part of pregnancy, the nurse knows that there is a danger of which of the following? 1. Major congenital anomalies 2. Fetal adhesions 3. Maternal diabetes 4. Rh sensitization
Answer: 2 Explanation: 2. If oligohydramnios occurs in the first part of pregnancy, there is a danger of fetal adhesions (one part of the fetus may adhere to another part).
20) A nurse teaches newly pregnant clients that if an ovum is fertilized and implants in the endometrium, the hormone the fertilized egg begins to secrete is which of the following? 1. Estrogen 2. Human chorionic gonadotropin (hCG) 3. Progesterone 4. Luteinizing hormone
Answer: 2 Explanation: 2. If the ovum is fertilized and implants in the endometrium, the fertilized egg begins to secrete human chorionic gonadotropin (hCG), which is needed to maintain the corpus luteum.
6) A client arrives in the labor and delivery unit and describes her contractions as occurring every 10-12 minutes, lasting 30 seconds. She is smiling and very excited about the possibility of being in labor. On exam, her cervix is dilated 2 cm, 100% effaced, and -2 station. What best describes this labor? 1. Second phase 2. Latent phase 3. Active phase 4. Transition phase
Answer: 2 Explanation: 2. In the early or latent phase of the first stage of labor, contractions are usually mild. The woman feels able to cope with the discomfort. The woman is often talkative and smiling and is eager to talk about herself and answer questions.
12) A client calls the labor and delivery unit and tells the nurse that she is 39 weeks pregnant and that over the last 4 or 5 days, she has noticed that although her breathing has become easier, she is having leg cramps, a slight amount of edema in her lower legs, and an increased amount of vaginal secretions. The nurse tells the client that she has experienced which of the following? 1. Engagement 2. Lightening 3. Molding 4. Braxton Hicks contractions
Answer: 2 Explanation: 2. Lightening describes the effect occurring when the fetus begins to settle into the pelvic inlet.
16) The laboring client brought a written birth plan indicating that she wanted to avoid pain medications and an epidural. She is now at 6 cm and states, "I can't stand this anymore! I need something for pain! How will an epidural affect my baby?" What is the nurse's best response? 1. "The narcotic in the epidural will make both you and the baby sleepy." 2. "It is unlikely that an epidural will decrease your baby's heart rate." 3. "Epidurals tend to cause low blood pressure in babies after birth." 4. "I can't get you an epidural, because of your birth plan."
Answer: 2 Explanation: 2. Maternal hypotension results in uteroplacental insufficiency in the fetus, which is manifested as late decelerations on the fetal monitoring strip. The risk of hypotension can be minimized by hydrating the vascular system with 500 to 1000 mL of IV solution before the procedure and changing the woman's position and/or increasing the IV rate afterward.
22) A pregnant client who swims 3-5 times per week asks the nurse whether she should stop this activity. What is the appropriate nursing response? 1. "You should decrease the number of times you swim per week." 2. "Continuing your exercise program would be beneficial." 3. "You should discontinue your exercise program immediately." 4. "You should consider a less strenuous type of exercise."
Answer: 2 Explanation: 2. Mild to moderate exercise is beneficial during pregnancy. Regular exercise-at least 30 minutes of moderate exercise daily or at least most days of the week-is preferred.
11) The clinic nurse is assisting with an initial prenatal assessment. The following findings are present: spider nevi present on lower legs; dark pink, edematous nasal mucosa; mild enlargement of the thyroid gland; mottled skin and pallor on palms and nail beds; heart rate 88 with murmur present. What is the best action for the nurse to take based on these findings? 1. Document the findings on the prenatal chart. 2. Have the physician see the client today. 3. Instruct the client to avoid direct sunlight. 4. Analyze previous thyroid hormone lab results.
Answer: 2 Explanation: 2. Mottling of the skin is indicative of possible anemia. These abnormalities must be reported to the healthcare provider immediately.
23) A client received epidural anesthesia during the first stage of labor. The epidural is discontinued immediately after delivery. This client is at increased risk for which problem during the fourth stage of labor? 1. Nausea 2. Bladder distention 3. Uterine atony 4. Hypertension
Answer: 2 Explanation: 2. Nursing care following an epidural block includes frequent assessment of the bladder to avoid bladder distention.
32) The nurse is caring for a client at 35 weeks' gestation who has been critically injured in a shooting. Which statement by the paramedics bringing the woman to the hospital would cause the greatest concern? 1. "Blood pressure 110/68, pulse 90." 2. "Entrance wound present below the umbilicus." 3. "Client is positioned in a left lateral tilt." 4. "Clear fluid is leaking from the vagina."
Answer: 2 Explanation: 2. Penetrating trauma includes gunshot wounds and stab wounds. The mother generally fares better than the fetus if the penetrating trauma involves the abdomen as the enlarged uterus is likely to protect the mother's bowel from injury.
27) Couples at risk for having a detectable single gene or chromosomal anomaly may wish to undergo which procedure? 1. Preimplantation genetic screening (PGS) 2. Preimplantation genetic diagnosis (PGD) 3. Intracytoplasmic sperm injection (ICSI) 4. Gamete intrafallopian transfer (GIFT)
Answer: 2 Explanation: 2. Preimplantation genetic diagnosis (PGD) is a term used when one or both genetic parents carry a gene mutation and testing is performed to determine whether that mutation or unbalanced chromosomal compliment has been passed to the oocyte or embryo.
21) A woman has been unable to complete a full-term pregnancy because the fertilized ovum failed to implant in the uterus. This is most likely due to a lack of which hormone? 1. Estrogen 2. Progesterone 3. FSH 4. LH
Answer: 2 Explanation: 2. Progesterone is often called the hormone of pregnancy because it inhibits uterine contractions and relaxes smooth muscle to cause vasodilation, allowing pregnancy to be maintained.
19) Four minutes after the birth of a baby, there is a sudden gush of blood from the mother's vagina, and about 8 inches of umbilical cord slides out. What action should the nurse take first? 1. Place the client in McRoberts position. 2. Watch for the emergence of the placenta. 3. Prepare for the delivery of an undiagnosed twin. 4. Place the client in a supine position.
Answer: 2 Explanation: 2. Signs of placental separation usually appear around 5 minutes after birth of the infant, but can take up to 30 minutes to manifest. These signs are (1) a globular-shaped uterus, (2) a rise of the fundus in the abdomen, (3) a sudden gush or trickle of blood, and (4) further protrusion of the umbilical cord out of the vagina.
3) A client who is in the second trimester of pregnancy tells the nurse that she has developed a darkening of the line in the midline of her abdomen from the symphysis pubis to the umbilicus. What other expected changes during pregnancy might she also notice? 1. Lightening of the nipples and areolas 2. Reddish streaks called striae on her abdomen 3. A decrease in hair thickness 4. Small purplish dots on her face and arms
Answer: 2 Explanation: 2. Striae, or stretch marks, are reddish, wavy, depressed streaks that may occur over the abdomen, breasts, and thighs as pregnancy progresses.
12) The nurse begins a prenatal assessment on a 25-year-old primigravida at 20 weeks' gestation and immediately contacts the healthcare provider because of which finding? 1. Pulse 88/minute 2. Respirations 30/minute 3. Temperature 37.4° C (99.3° F) 4. Blood pressure 118/82
Answer: 2 Explanation: 2. Tachypnea is not a normal finding and requires medical care.
24) The nurse anticipates that the physician will most likely order a cervicovaginal fetal fibronectin test for which client? 1. The client at 34 weeks' gestation with gestational diabetes 2. The client at 32 weeks' gestation with regular uterine contractions 3. The client at 37 weeks' multi-fetal gestation 4. The client at 20 weeks' gestation with ruptured amniotic membranes
Answer: 2 Explanation: 2. The absence of cervicovaginal fFN between 20 and 34 weeks' gestation has been shown to be a strong predictor of a woman not experiencing preterm birth due to spontaneous preterm labor or premature rupture of membranes. Positive findings indicate a 99% probability of birth within the next 2 weeks.
16) Of all the clients who have been scheduled to have a biophysical profile, the nurse should check with the physician and clarify the order for which client? 1. A gravida with intrauterine growth restriction 2. A gravida with mild hypotension of pregnancy 3. A gravida who is postterm 4. A gravida who complains of decreased fetal movement for 2 days
Answer: 2 Explanation: 2. The biophysical profile is used when there is a risk of placental and/or fetal compromise. The gravida with mild hypotension will need to be monitored more closely throughout the pregnancy, but is not a candidate at present for a biophysical profile.
18) On examination of the prenatal client, the nurse is aware that she will assess for a bluish pigmentation of the vagina. What is this objective (probable) sign of pregnancy also known as? 1. Hegar sign 2. Chadwick sign 3. Nightingale sign 4. Goodell sign
Answer: 2 Explanation: 2. The blue-purple discoloration of the cervix is Chadwick sign.
1) The laboring client is at 7 cm, with the vertex at a +1 station. Her birth plan indicates that she and her partner took Lamaze prenatal classes, and they have planned on a natural, unmedicated birth. Her contractions are every 3 minutes and last 60 seconds. She has used relaxation and breathing techniques very successfully in her labor until the last 15 minutes. Now, during contractions, she is writhing on the bed and screaming. Her labor partner is rubbing the client's back and speaking to her quietly. Which nursing diagnosis should the nurse incorporate into the plan of care for this client? 1. Fear/Anxiety related to discomfort of labor and unknown labor outcome 2. Pain, Acute, related to uterine contractions, cervical dilatation, and fetal descent 3. Coping: Family, Compromised, related to labor process 4. Knowledge, Deficient, related to lack of information about normal labor process and comfort measures
Answer: 2 Explanation: 2. The client is exhibiting signs of acute pain, which is both common and expected in the transitional phase of labor.
14) A client at 36 weeks' gestation is complaining of dyspnea when lying flat. What is the clinical reason for this complaint? 1. Maternal hypertension 2. Fundal height 3. Hydramnios 4. Congestive heart failure
Answer: 2 Explanation: 2. The dyspnea is resulting from the pressure of the enlarging uterus on the diaphragm.
1) The nurse is admitting a client to the birthing unit. What question should the nurse ask to gain a better understanding of the client's psychosocial status? 1. "How did you decide to have your baby at this hospital?" 2. "Who will be your labor support person?" 3. "Have you chosen names for your baby yet?" 4. "What feeding method will you use for your baby?"
Answer: 2 Explanation: 2. The expectant mother's partner or support person is an important member of the birthing team, and assessments of the couple's coping, interactions, and teamwork are integral to the nurse's knowledge base. The nurse's physical presence with the laboring woman provides the best opportunity for ongoing assessment.
1) The external and internal female reproductive organs develop and mature in response to what hormones? 1. Adrenocorticotropic hormones (ACTH) 2. Estrogen and progesterone 3. Steroid hormones 4. Luteinizing hormones (LH)
Answer: 2 Explanation: 2. The external and internal female reproductive organs develop and mature in response to estrogen and progesterone.
35) The client at 38 weeks' gestation has been diagnosed with oligohydramnios. Which statement indicates that teaching about the condition has been effective? 1. "My gestational diabetes might have caused this problem to develop." 2. "When I go into labor, I should come to the hospital right away." 3. "This problem was diagnosed with blood and urine tests." 4. "Women with this condition usually do not have a cesarean birth."
Answer: 2 Explanation: 2. The incidence of cord compression and resulting fetal distress is high when there is an inadequate amount of amniotic fluid. The client with oligohydramnios should come to the hospital in early labor.
16) Which of the following may be the main presenting symptom of iron deficiency anemia? 1. Frequent urination 2. Fatigue 3. Nausea 4. Headaches
Answer: 2 Explanation: 2. The main presenting symptom of iron deficiency anemia may be fatigue.
20) A 16-year-old pregnant client is seen at her 10-weeks'-gestation visit. She tells the nurse that she felt the baby move that morning. What response by the nurse is appropriate? 1. "That is very exciting. The baby must be very healthy." 2. "Would you please describe what you felt for me?" 3. "That is impossible. The baby is not big enough yet." 4. "Would you please let me see whether I can feel the baby?"
Answer: 2 Explanation: 2. The nurse should ask the client to describe what she felt, as 10 weeks' gestation is too early to feel fetal movement.
27) The pregnant client has asked the nurse what kinds of medications cause birth defects. Which statement would best answer this question? 1. "Birth defects are very rare. Don't worry; your doctor will watch for problems." 2. "To be safe, don't take any medication without talking to your doctor." 3. "Too much vitamin C is one of the most common issues." 4. "Almost all medications will cause birth defects in the first trimester."
Answer: 2 Explanation: 2. The nurse should remind the client of the need to check with her caregiver about medications. If a woman has taken a drug in category D or X, she should be informed of the risks associated with that drug and of her alternatives.
32) During a prenatal examination, an adolescent client asks, "How does my baby get air?" What correct information would the nurse give? 1. "The lungs of the fetus carry out respiratory gas exchange in utero similar to what an adult experiences." 2. "The placenta assumes the function of the fetal lungs by supplying oxygen and allowing the excretion of carbon dioxide into your bloodstream." 3. "The blood from the placenta is carried through the umbilical artery, which penetrates the abdominal wall of the fetus." 4. "The fetus is able to obtain sufficient oxygen due to the fact that your hemoglobin concentration is 50% greater during pregnancy."
Answer: 2 Explanation: 2. The placenta assumes the function of the fetal lungs by supplying oxygen and allowing the excretion of carbon dioxide into the maternal bloodstream.
31) The nurse teaching a high school class explains that during the menstrual cycle, the endometrial glands begin to enlarge under the influence of estrogen and cervical mucosal changes occur; the changes peak at ovulation. In which phase of the menstrual cycle does this occur? 1. Menstrual 2. Proliferative 3. Secretory 4. Ischemic
Answer: 2 Explanation: 2. The proliferative phase begins when the endometrial glands begin to enlarge under the influence of estrogen and cervical mucosal changes occur; the changes peak at ovulation.
18) A laboring client has received an order for epidural anesthesia. In order to prevent the most common complication associated with this procedure, what would the nurse expect to do? 1. Observe fetal heart rate variability 2. Hydrate the vascular system with 500-1000 mL of intravenous fluids 3. Place the client in the semi-Fowler's position 4. Teach the client appropriate breathing techniques
Answer: 2 Explanation: 2. The risk of hypotension can be minimized by hydrating the vascular system with 500 to 1000 mL of IV solution before the procedure and changing the woman's position and/or increasing the IV rate afterward.
27) A prenatal educator is asking a partner about normal psychological adjustment of an expectant mother during the second trimester of pregnancy. Which answer by the partner would indicate a typical expectant mother's response to pregnancy? 1. "She is very body-conscious, and hates every little change." 2. "She daydreams about what kind of parent she is going to be." 3. "I haven't noticed anything. I just found out she was pregnant." 4. "She has been having dreams at night about misplacing the baby."
Answer: 2 Explanation: 2. The second trimester brings increased introspection and consideration of how she will parent. She might begin to get furniture and clothing as concrete preparation, and feels movement and is aware of the fetus and incorporates it into herself.
25) The neonate was born 5 minutes ago. The body is bluish. The heart rate is 150. The infant is crying strongly. The infant cries when the sole of the foot is stimulated. The arms and legs are flexed, and resist straightening. What should the nurse record as this infant's Apgar score? 1. 7 2. 8 3. 9 4. 10
Answer: 2 Explanation: 2. The strong cry earns 2 points. The crying with foot sole stimulation earns 2 points. The limb flexion and resistance earn 2 points each. Bluish color earns 0 points. The Apgar score is 8.
7) During the nursing assessment of a woman with ruptured membranes, the nurse suspects a prolapsed umbilical cord. What would the nurse's priority action be? 1. To help the fetal head descend faster 2. To use gravity and manipulation to relieve compression on the cord 3. To facilitate dilation of the cervix with prostaglandin gel 4. To prevent head compression
Answer: 2 Explanation: 2. The top priority is to relieve compression on the umbilical cord to allow blood flow to reach the fetus. It is because some obstetric maneuvers to relieve cord compression are complicated that cesarean birth is sometimes necessary.
13) The true moment of fertilization occurs when what happens? 1. Cortical reaction occurs 2. Nuclei unite 3. Spermatozoa propel themselves up the female tract 4. Sperm surrounding the ovum release their enzymes
Answer: 2 Explanation: 2. The true moment of fertilization occurs as the nuclei unite. Their individual nuclear membranes disappear, and their chromosomes pair up to produce the diploid zygote.
27) Which client requires immediate intervention by the labor and delivery nurse? 1. Client at 8 cm, systolic blood pressure has increased 35 mm Hg 2. Client who delivered 1 hour ago with WBC of 50,000 3. Client at 5 cm with a respiratory rate of 22 between contractions 4. Client in active labor with polyuria
Answer: 2 Explanation: 2. The white blood cell (WBC) count increases to 25,000/mm3 to 30,000/mm3 during labor and early postpartum. This count is abnormally high, and requires further assessment and provider notification.
23) The adolescent client reports to the clinic nurse that her period is late, but that her home pregnancy test is negative. What should the nurse explain that these findings most likely indicate? 1. "This means you are not pregnant." 2. "You might be pregnant, but it might be too early for your home test to be accurate." 3. "We don't trust home tests. Come to the clinic for a blood test." 4. "Most people don't use the tests correctly. Did you read the instructions?"
Answer: 2 Explanation: 2. This is a true statement. Most home pregnancy tests have low false-positive rates, but the false-negative rate is slightly higher. Repeating the test in a week is recommended.
10) The client at 40 weeks' gestation reports to the nurse that she has had increased pelvic pressure and increased urinary frequency. Which response by the nurse is best? 1. "Unless you have pain with urination, we don't need to worry about it." 2. "These symptoms usually mean the baby's head has descended further." 3. "Come in for an appointment today and we'll check everything out." 4. "This might indicate that the baby is no longer in a head-down position."
Answer: 2 Explanation: 2. This is the best response because it most directly addresses what the client has reported.
8) The prenatal clinic nurse is designing a new prenatal intake information form for pregnant clients. Which question is best to include on this form? 1. Where was the father of the baby born? 2. Do genetic diseases run in the family of the baby's father? 3. What is the name of the baby's father? 4. Are you married to the father of the baby?
Answer: 2 Explanation: 2. This question has the highest priority because it gets at the physiologic issue of inheritable genetic diseases that might directly impact the baby.
3) The nurse is working with a pregnant adolescent. The client asks the nurse how the baby's condition is determined during labor. The nurse's best response is that during labor, the nurse will do which of the following? 1. Check the client's cervix by doing a pelvic exam every 2 hours. 2. Assess the fetus's heart rate with an electronic fetal monitor. 3. Look at the color and amount of bloody show that the client has. 4. Verify that the client's contractions are strong but not too close together.
Answer: 2 Explanation: 2. This statement best answers the question the client has asked.
26) Before applying a cord clamp, the nurse assesses the umbilical cord. The mother asks why the nurse is doing this. What should the nurse reply? 1. "I'm checking the blood vessels in the cord to see whether it has one artery and one vein." 2. "I'm checking the blood vessels in the cord to see whether it has two arteries and one vein." 3. "I'm checking the blood vessels in the cord to see whether it has two veins and one artery." 4. "I'm checking the blood vessels in the cord to see whether it has two arteries and two veins."
Answer: 2 Explanation: 2. Two arteries and one vein are present in a normal umbilical cord.
22) A woman is 32 weeks pregnant. She is HIV-positive but asymptomatic. The nurse knows what would be important in managing her pregnancy and delivery? 1. An amniocentesis at 30 and 36 weeks 2. Weekly non-stress testing beginning at 32 weeks' gestation 3. Application of a fetal scalp electrode as soon as her membranes rupture in labor 4. Administration of intravenous antibiotics during labor and delivery
Answer: 2 Explanation: 2. Weekly non-stress testing (NST) is begun at 32 weeks' gestation and serial ultrasounds are done to detect IUGR.
25) Women with HIV should be evaluated and treated for other sexually transmitted infections and for what condition occurring more commonly in women with HIV? 1. Syphilis 2. Toxoplasmosis 3. Gonorrhea 4. Herpes
Answer: 2 Explanation: 2. Women with HIV should be evaluated and treated for other sexually transmitted infections and for conditions occurring more commonly in women with HIV, such as tuberculosis, cytomegalovirus, toxoplasmosis, and cervical dysplasia.
31) The nurse is admitting a client with possible hydramnios. When is hydramnios most likely suspected? 1. Hydramnios is most likely suspected when there is less amniotic fluid than normal for gestation. 2. Hydramnios is most likely suspected when the fundal height increases disproportionately to the gestation. 3. Hydramnios is most likely suspected when the woman has a twin gestation. 4. Hydramnios is most likely suspected when the quadruple screen comes back positive.
Answer: 2 Explanation: 2. Hydramnios should be suspected when the fundal height increases out of proportion to the gestational age.
5) The nurse is preparing a class for expectant fathers. Which information should the nurse include? 1. Siblings adjust readily to the new baby. 2. Sexual activity is safe for normal pregnancy. 3. The expectant mother decides the feeding method. 4. Fathers are expected to be involved in labor and birth.
Answer: 2 Explanation: 2. In a healthy pregnancy, there is no medical reason to limit sexual activity.
10) The laboring client and her partner have arrived at the birthing unit. Which step of the admission process should be undertaken first? 1. The sterile vaginal exam 2. Welcoming the couple 3. Auscultation of the fetal heart rate 4. Checking for ruptured membranes
Answer: 2 Explanation: 2. It is important to establish rapport and to create an environment in which the family feels free to ask questions. The support and encouragement of the nurse in maintaining a caring environment begin with the initial admission.
28) The nurse is caring for a client with hydramnios. What will the nurse watch for? 1. Possible intrauterine growth restriction 2. Newborn congenital anomalies 3. Newborn postmaturity and renal malformations 4. Fetal adhesions
Answer: 2 Explanation: 2. Newborn congenital anomalies occur with hydramnios.
22) The nurse is assessing a primiparous client who indicates that her religion is Judaism. Why is this information is pertinent for the nurse to assess? 1. Religious and cultural background can impact what a client eats during pregnancy. 2. It provides a baseline from which to ask questions about the client's religious and cultural background. 3. Knowing the client's beliefs and behaviors regarding pregnancy is not important. 4. Clients sometimes encounter problems in their pregnancies based on what religion they practice.
Answer: 2 Explanation: 2. Nurses have an obligation to be aware of other cultures and develop a culturally sensitive plan of care to meet the needs of the childbearing woman and her family.
5) A client was admitted to the labor area at 5 cm with ruptured membranes about 14 hours ago. What assessment data would be most beneficial for the nurse to collect? 1. Blood pressure 2. Temperature 3. Pulse 4. Respiration
Answer: 2 Explanation: 2. Rupture of membranes places the mother at risk for infection. The temperature is the primary and often the first indication of a problem.
6) The nurse receives a phone call from a client who claims she is pregnant. The client reports that she has regular menses that occur every 28 days and last 5 days. The first day of her last menses was April 10. What would the client's estimated date of delivery (EDD) be if she is pregnant? 1. Nov. 13 2. Jan. 17 3. Jan. 10 4. Dec. 3
Answer: 2 Explanation: 2. The due date is Jan. 17. Nagele's rule is to add 7 days to the last menstrual period and subtract 3 months. The last menstrual period is April 10, therefore Jan. 17 is the EDD.
29) A client at 10 weeks' gestation has developed cholecystitis. If surgery is required, what is the safest time during pregnancy? 1. Immediately, before the fetus gets any bigger 2. Early in the second trimester 3. As close to term as possible 4. The risks are too high to do it anytime in pregnancy
Answer: 2 Explanation: 2. The early second trimester is the best time to operate because there is less risk of spontaneous abortion or early labor, and the uterus is not so large as to impinge on the abdominal field.
14) The nurse working in an outpatient obstetric clinic assesses four primigravida clients. Which client findings would the nurse tell the physician about? 1. 17 weeks' gestation and client denies feeling fetal movement 2. 24 weeks' gestation and fundal height is at the umbilicus 3. 4-6 weeks' gestation and softening of the cervix 4. 34 weeks' gestation and complains of hemorrhoidal pain
Answer: 2 Explanation: 2. The fundal height at 24 weeks should be 24 cm. The fundal height is usually at the umbilicus at 20-22 weeks.
26) The school nurse is planning a class about nutrition for pregnant teens, several of whom have been diagnosed with iron-deficiency anemia. In order to increase iron absorption, the nurse would encourage the teens to consume more of what beverage? 1. Gatorade 2. Orange juice 3. Milk 4. Green tea
Answer: 2 Explanation: 2. Vitamin C is found in citrus fruits and juices, and is known to enhance the absorption of iron from meat and non-meat sources.
1) A pregnant teenage client is diagnosed with iron-deficiency anemia. Which nutrient should the nurse encourage her to take to increase iron absorption? 1. Vitamin A 2. Vitamin C 3. Vitamin D 4. Vitamin E
Answer: 2 Explanation: 2. Vitamin C is known to enhance the absorption of iron from meat and nonmeat sources.
13) The nurse is planning an in-service educational program to talk about disseminated intravascular coagulation (DIC). The nurse should identify which conditions as risk factors for developing DIC? Select all that apply. 1. Diabetes mellitus 2. Abruptio placentae 3. Fetal demise 4. Multiparity 5. Preterm labor
Answer: 2, 3 Explanation: 2. As a result of the damage to the uterine wall and the retroplacental clotting with covert abruption, large amounts of thromboplastin are released into the maternal blood supply, which in turn triggers the development of disseminated intravascular coagulation (DIC) and the resultant hypofibrinogenemia. 3. Perinatal mortality associated with abruptio placentae is approximately 25%. If fetal hypoxia progresses unchecked, irreversible brain damage or fetal demise may result.
30) The nurse teaches a client that luteinizing hormone (LH) is important in the ovarian cycle for which purposes? Select all that apply. 1. Proliferation of the endometrial mucosa 2. Ovulation 3. Corpus luteum development 4. Maturation of the ovarian follicle 5. Cyclic changes that allow pregnancy not to occur
Answer: 2, 3 Explanation: 2. During the follicular phase, the primordial follicle matures under the influence of FSH and LH until ovulation occurs. 3. The corpus luteum develops under the influence of LH during the luteal phase.
5) A client has just arrived in the birthing unit. What steps would be most important for the nurse to perform to gain an understanding of the physical status of the client and her fetus? Select all that apply. 1. Check for ruptured membranes and apply a fetal scalp electrode. 2. Auscultate the fetal heart rate between and during contractions. 3. Palpate contractions and resting uterine tone. 4. Assess the blood pressure, temperature, respiratory rate, and pulse rate. 5. Perform a vaginal exam for cervical dilation, and perform Leopold maneuvers.
Answer: 2, 3 Explanation: 2. Fetal heart rate auscultation gives information about the physical status of the fetus. 3. Contraction palpation provides information about the frequency, duration, and intensity of the contractions.
27) The client presents to the clinic for an initial prenatal examination. She asks the nurse whether there might be a problem for her baby because she has type B Rh-positive blood and her husband has type O Rh-negative blood, or because her sister's baby had ABO incompatibility. What is the nurse's best answer? Select all that apply. 1. "Your baby would be at risk for Rh problems if your husband were Rh-negative." 2. "Rh problems only occur when the mother is Rh-negative and the father is not." 3. "ABO incompatibility occurs only after the baby is born." 4. "We don't know for sure, but we can test for ABO incompatibility." 5. "Your husband's being type B puts you at risk for ABO incompatibility."
Answer: 2, 3 Explanation: 2. Rh incompatibility is a possibility when the mother is Rh-negative and the father is Rh-positive. 3. ABO incompatibility is limited to type O mothers with a type A or B fetus and occurs after the baby is born.
19) The nurse knows that which of the following are advantages of spinal block? Select all that apply. 1. Intense blockade of sympathetic fibers 2. Relative ease of administration 3. Maternal compartmentalization of the drug 4. Immediate onset of anesthesia 5. Larger drug volume
Answer: 2, 3, 4
28) A pregnant client complains to the clinic nurse that her varicose veins are causing more discomfort than before. Which recommendations does the nurse make to the client? Select all that apply. 1. Increase the time she stands. 2. Walk on a daily basis. 3. Not cross her legs at the knees. 4. Wear support hose. 5. Hyperextend her knee with her feet up.
Answer: 2, 3, 4
32) The nurse is preparing a brochure for couples considering pregnancy after the age of 35. Which statements should be included? Select all that apply. 1. There is a decreased risk of Down syndrome. 2. Preexisting medical conditions can complicate pregnancy. 3. Preterm births are more common. 4. Amniocentesis can be performed to detect genetic anomalies. 5. The increased fertility of women over age 35 makes conception easier.
Answer: 2, 3, 4
22) The nurse recognizes that subjective pregnancy changes such as amenorrhea can be caused by which conditions? Select all that apply. 1. Goodell sign 2. Anemia 3. Pseudocyesis 4. Thyroid dysfunction 5. Fetal heartbeat
Answer: 2, 3, 4 Explanation: 2. Anemia can cause amenorrhea, and is a subjective sign of pregnancy. 3. Pseudocyesis (intense desire for pregnancy) can cause amenorrhea. 4. Thyroid dysfunction can cause amenorrhea, and is a subjective sign of pregnancy.
29) Absolute contraindications to exercise while pregnant include which of the following? Select all that apply. 1. Abruptio placentae 2. Placenta previa after 26 weeks' gestation 3. Preeclampsia-eclampsia 4. Cervical insufficiency (cerclage) 5. Intrauterine growth restriction (IUGR)
Answer: 2, 3, 4 Explanation: 2. Placenta previa after 26 weeks' gestation is an absolute contraindication to exercise. 3. Preeclampsia-eclampsia is an absolute contraindication to exercise. 4. Cervical insufficiency (cerclage) is an absolute contraindication to exercise.
17) The nurse is seeing a client who asks about the accuracy of Nagele's rule. The nurse explains that accuracy can be compromised under which conditions? Select all that apply. 1. There is a history of regular menses every 28 days. 2. Amenorrhea is present and ovulation occurs with breastfeeding. 3. Oral contraception was discontinued, but no regular menstruation was established. 4. There has been 1 or more months of amenorrhea. 5. There is an accurate date for the last menstrual period.
Answer: 2, 3, 4 Explanation: 2. Nagele's rule is not always accurate for women who have amenorrhea but are ovulating and conceive while breastfeeding. 3. Nagele's rule is not always accurate for women who conceive before regular menstruation is established following discontinuation of oral contraceptives or termination of a pregnancy. 4. Nagele's rule is not always accurate for women with markedly irregular periods that include 1 or more months of amenorrhea.
17) The nurse is aware of the different breathing techniques that are used during labor. Why are breathing techniques used during labor? Select all that apply. 1. They are a form of anesthesia. 2. They are a source of relaxation. 3. They increase the ability to cope with contractions. 4. They are a source of distraction. 5. They increase a woman's pain threshold.
Answer: 2, 3, 4, 5
7) The nurse educator teaching reproductive anatomy wants to make sure the students understand what stabilizes the uterus. Which statements about the individual ligaments would the nurse include? Select all that apply. 1. The infundibulopelvic ligaments suspend and support the uterus. 2. The broad ligament keeps the uterus centrally placed. 3. The uterosacral ligaments sweep back around the rectum and insert on the sides of the first and second sacral vertebrae. 4. The ovarian ligaments anchor the ovary to the uterus. 5. The cardinal ligaments prevent uterine prolapse and support the upper vagina.
Answer: 2, 3, 4, 5 Explanation: 2. The broad ligament keeps the uterus centrally placed. 3. The uterosacral ligaments sweep back around the rectum and insert on the sides of the first and second sacral vertebrae. 4. The ovarian ligaments anchor the ovary to the uterus. 5. The cardinal ligaments prevent uterine prolapse and support the upper vagina.
19) The clinic nurse is teaching a pregnant client about her iron supplement. Which information is included in the teaching? Select all that apply. 1. Iron does not affect the gastrointestinal tract. 2. A stool softener might be needed. 3. Start a low dose, and increase it gradually. 4. Expect the stools to be black and bloody. 5. Iron absorption is poor if taken with meals.
Answer: 2, 3, 5
27) A pregnant client at 28 weeks' gestation asks the nurse what her baby is like at this stage of pregnancy. How would the nurse describe the fetus. Select all that apply. 1. The fetus is developing subcutaneous fat. 2. The fetus is now opening and closing her eyes. 3. The baby could now breathe on her own, if she were born. 4. The fetus has fingernails and toenails. 5. The fetus is forming surfactant needed for lung function.
Answer: 2, 3, 5 Explanation: 2. At 28 weeks, the eyes begin to open and close. 3. At 28 weeks, the delivered baby can breathe. 5. At 28 weeks, the fetus has the surfactant formed needed for breathing.
5) The nurse working with pediatric clients knows that the primary hormone secretions that induce puberty include which of the following? Select all that apply. 1. Thyroid hormone 2. Follicle-stimulating hormone 3. Leuteinizing hormone 4. Adrenocorticotropic hormone 5. Gonadotropin-releasing hormones
Answer: 2, 3, 5 Explanation: 2. Follicle-stimulating hormone (FSH) is part of the process that induces puberty. 3. The luteinizing hormone (LH) is a part of the process that induces puberty. 5. The central nervous system releases a neurotransmitter that stimulates the hypothalamus to synthesize and release gonadotropin-releasing hormone (GnRH).
24) The nurse is evaluating the goal "Client will remain free of opportunistic infections" for an HIV-positive pregnant client. The nurse determines the goal was met when the client has which of the following? Select all that apply. 1. An absolute CD4+ T-lymphocyte count below 200 2. No complaint of chills or fever during the pregnancy 3. Weight gain of 30 lbs during the pregnancy 4. ESR above 20 mm/hr 5. Normal erythrocyte sedimentation rate maintained during the pregnancy
Answer: 2, 3, 5 Explanation: 2. Not having chills, fever, or a sore throat throughout the pregnancy is an indication the client did not have an infection. 3. Weight gain of 25 to 35 pounds is normal for a pregnancy. This client met the goal for nutrition and remaining infection-free. 5. Having a normal erythrocyte sedimentation rate during the pregnancy is an expected outcome.
9) During the initial prenatal visit, the nurse assesses the history of the father of the child for which of the following? Select all that apply. 1. Stability of living conditions 2. Blood type and Rh type 3. Significant health problems 4. Nutritional history 5. Current use of tobacco
Answer: 2, 3, 5 Explanation: 2. The father of the fetus should be assessed for blood type and Rh factor. 3. The father of the fetus should be assessed for significant health problems. 5. The father of the fetus should be assessed for current alcohol intake, drug use, and tobacco use.
18) Under the influence of progesterone, which of the following occur? Select all that apply. 1. Temperature decreases 2. Cervix secretes thick, viscous mucus 3. Breasts prepare for lactation 4. Breast glandular tissue decreases in size 5. Vaginal epithelium proliferates
Answer: 2, 3, 5 Explanation: 2. Under the influence of progesterone, the cervix secretes thick, viscous mucus. 3. Under the influence of progesterone, breasts prepare for lactation. 5. Under the influence of progesterone, vaginal epithelium proliferates.
11) The nurse in the prenatal clinic will tell the client at 38-weeks' gestation to lie on her left side when the client complains of which of the following? Select all that apply. 1. Nausea 2. Pallor 3. Clamminess 4. Constipation 5. Dizziness
Answer: 2, 3, 5 Explanation: 2. Vena caval syndrome can cause pallor, which is relieved when the client turns to lie on her left side. 3. Vena caval syndrome can cause clamminess, which is relieved when the client turns to lie on her left side. 5. Vena caval syndrome can cause dizziness, which is relieved when the client turns to lie on her left side.
15) What signs would indicate that a pregnant client's urinalysis culture was abnormal? Select all that apply. 1. pH 4.6-8 2. Alkaline urine 3. Cloudy appearance 4. Negative for protein and red blood cells 5. Hemoglobinuria
Answer: 2, 3, 5 Explanation: 2. Alkaline urine could indicate metabolic alkalemia, Proteus infection, or an old specimen. 3. A cloudy appearance could indicate an infection. 5. Hemoglobinuria would be indicated by an abnormal urine color.
25) A couple is at the clinic for preconceptual counseling. Both parents are 40 years old. The nurse knows that the education session has been successful when the wife makes which statement? Select all that apply. 1. "We are at low risk for having a baby with Down syndrome." 2. "Our children are more likely to have genetic defects." 3. "Children born to parents this age have sex-linked disorders." 4. "The tests for genetic defects can be done early in pregnancy." 5. "It will be almost impossible for us to conceive a child."
Answer: 2, 4 Explanation: 2. Women 35 or older are at greater risk for having children with chromosome abnormalities. 4. Genetic testing such as amniocentesis and chorionic villus sampling are done in the first trimester.
34) The nurse determines that a client is carrying her fetus in the vertical (longitudinal) lie. The nurse's judgment should be questioned if the fetal presenting part is which of the following? Select all that apply. 1. Sacrum 2. Left arm 3. Mentum 4. Left scapula 5. Right scapula
Answer: 2, 4, 5 Explanation: 2. A fetus with an arm presenting is likely in a horizontal lie. 4. A fetus with a left scapula presenting is in a horizontal lie. 5. A fetus with a right scapula presenting is in a horizontal lie.
10) What are the three functions of cervical mucosa? Select all that apply. 1. Form the relatively fixed axis of the birth passage 2. Provide lubrication for the vaginal canal 3. Provide nourishment and protective maternal antibodies to infants 4. Provide an alkaline environment to shelter deposited sperm from the acidic vaginal secretions 5. Act as a bacteriostatic agent
Answer: 2, 4, 5 Explanation: 2. The cervical mucosa provides lubrication for the vaginal canal. 4. The cervical mucosa provides an alkaline environment to shelter deposited sperm from the acidic vaginal secretions. 5. The cervical mucosa acts as a bacteriostatic agent.
19) A pregnant client who is a lacto-vegetarian asks the nurse for assistance with her diet. What instruction should the nurse give? Select all that apply. 1. "Protein is important; therefore, the addition of one serving of meat a day is necessary." 2. "A daily supplement of vitamin B12 is important." 3. "The high fiber in a vegetarian diet is dangerous for pregnant women." 4. "Eggs are important to add to your diet. Eat six eggs per week." 5. "Milk products contain protein, but they are very low in iron."
Answer: 2, 5
34) A client tells the nurse that she does not like citrus fruits, and would like suggestions for alternate vitamin C sources. What should the nurse suggest as good sources of vitamin C? Select all that apply. 1. Barley and brown rice 2. Strawberries and potatoes 3. Buckwheat and lentils 4. Wheat flour and figs 5. Blueberries and broccoli
Answer: 2, 5
34) The primary care provider is performing a fetal scalp stimulation test. What result would the nurse hope to observe? Select all that apply. 1. Spontaneous fetal movement 2. Fetal heart acceleration 3. Increase in fetal heart variability 4. Resolution of late decelerations 5. Reactivity associated with the stimulation
Answer: 2, 5
31) An anesthesiologist informs the nurse that a client scheduled for a caesarean section will be having general anesthesia with postoperative self-controlled analgesia. For which clients would a general anesthesia be recommended? Select all that apply. 1. The client with a history of hypertension 2. The client who has had a lower back fusion 3. The client who is 13 years old 4. The client who is allergic to morphine sulfate 5. The client who has had surgery for scoliosis
Answer: 2, 5 Explanation: 2. Contraindications for epidural block include patients with previous back surgery. 5. Contraindications for epidural block include patients with previous back surgery.
35) The primary physician orders a narcotic analgesic for a client in labor. Which situations would lead the nurse to hold the medication? Select all that apply. 1. Contraction pattern every 3 minutes for 60 seconds 2. Fetal monitor tracing showing late decelerations 3. Client sleeping between contractions 4. Blood pressure 150/90 5. Blood pressure 80/42
Answer: 2, 5 Explanation: 2. Maternal hypotension results in uteroplacental insufficiency in the fetus, which is manifested as late decelerations on the fetal monitoring strip.
20) A client is being admitted to the labor area with the diagnosis of eclampsia. Which actions by the nurse are appropriate at this time? Select all that apply. 1. Tape a tongue blade to the head of the bed. 2. Pad the side rails. 3. Have the woman sit up. 4. Provide the client with grief counseling. 5. The airway should be maintained and oxygen administered.
Answer: 2, 5 Explanation: 2. Side rails should be up and padded. 5. Suctioning may be necessary to keep the airway clear.
12) A pregnant client has been admitted with a diagnosis of hyperemesis. Which orders written by the primary healthcare provider are the highest priorities for the nurse to implement? Select all that apply. 1. Obtain complete blood count. 2. Start intravenous fluid with multivitamins. 3. Check admission weight. 4. Obtain urine for urinalysis. 5. Give a medication to stop the nausea and vomiting.
Answer: 2, 5 Explanation: 2. Starting intravenous fluid with multivitamins is a priority if the client has been vomiting. 5. Giving a medication to stop the nausea and vomiting is a priority.
15) A woman asks her nurse what she can do before she begins trying to get pregnant to help her baby, as she is prone to anemia. What would the nurse correctly advise her to do? 1. Get pregnant, then start iron supplementation. 2. Add more carbohydrates to her diet. 3. Begin taking folic acid supplements daily. 4. Have a hemoglobin baseline done now so her progress can be followed.
Answer: 3
19) The nurse educator is describing the different kinds of abruptio placentae to a group of students, explaining that in a complete abruptio placentae, which of the following occurs? 1. Separation begins at the periphery of the placenta. 2. The placenta separates centrally and blood is trapped between the placenta and the uterine wall. 3. There is massive vaginal bleeding in the presence of almost total separation. 4. Blood passes between the fetal membranes and the uterine wall, and escapes vaginally.
Answer: 3
2) When assisting with a transabdominal sampling, which of the following would the nurse do? 1. Obtain preliminary urinary samples. 2. Have the woman empty her bladder before the test begins. 3. Assist the woman into a supine position on the examining table. 4. Instruct the woman to eat a fat-free meal 2 hours before the scheduled test time.
Answer: 3
21) The pregnant client in her second trimester states, "I didn't know my breasts would become so large. How do I find a good bra?" The best answer for the nurse to make would be which of the following? 1. "Avoid cotton fabrics and get an underwire bra; they fit everyone best." 2. "Just buy a bra one cup size bigger than usual, and it will fit." 3. "Look for wide straps and cups big enough for all of your breast tissue." 4. "There isn't much you can do for comfort. Try not wearing a bra at all."
Answer: 3
25) When counseling a newly pregnant client at 8 weeks' gestation of twins, the nurse teaches the woman about the need for increased caloric intake. What would the nurse tell the woman that the minimum recommended intake should be? 1. 2500 kcal and 120 grams protein 2. 3000 kcal and 150 grams protein 3. 4000 kcal and 135 grams protein 4. 5000 kcal and 190 grams protein
Answer: 3
26) If the woman is Rh negative and not sensitized, she is given Rh immune globulin to prevent what? 1. The potential for hemorrhage 2. Hyperhomocysteinemia 3. Antibody formation 4. Tubal pregnancy
Answer: 3
29) The labor and delivery nurse is assigned to four clients in early labor. Which electronic fetal monitoring finding would require immediate intervention? 1. Early decelerations with each contraction 2. Variable decelerations that recover to the baseline 3. Late decelerations with minimal variability 4. Accelerations
Answer: 3
3) The nurse is preparing a client in her second trimester for a three-dimensional ultrasound examination. Which statement indicates that teaching has been effective? 1. "If the ultrasound is normal, it means my baby has no abnormalities." 2. "The nuchal translucency measurement will diagnose Down syndrome." 3. "I might be able to see who the baby looks like with the ultrasound." 4. "Measuring the length of my cervix will determine whether I will deliver early."
Answer: 3
31) After noting meconium-stained amniotic fluid and fetal heart rate decelerations, the physician diagnoses a depressed fetus. The appropriate nursing action at this time would be to do what? 1. Increase the mother's oxygen rate. 2. Turn the mother to the left lateral position. 3. Prepare the mother for a higher-risk delivery. 4. Increase the intravenous infusion rate.
Answer: 3
7) The client is being admitted to the birthing unit. As the nurse begins the assessment, the client's partner asks why the fetus's heart rate will be monitored. After the nurse explains, which statement by the partner indicates a need for further teaching? 1. "The fetus's heart rate will vary between 110 and 160." 2. "The heart rate is monitored to see whether the fetus is tolerating labor." 3. "By listening to the heart, we can tell the gender of the fetus." 4. "After listening to the heart rate, you will contact the midwife."
Answer: 3
30) The nurse is presenting a class to newly pregnant families. What form of trauma will the nurse describe as the leading cause of fetal and maternal death? 1. Falls 2. Domestic violence 3. Gun accidents 4. Motor vehicle accidents
Answer: 4
24) Which maternal-child client should the nurse see first? 1. Blood type O, Rh-negative 2. Indirect Coombs' test negative 3. Direct Coombs' test positive 4. Blood type B, Rh-positive
Answer: 3 Explanation: 3. Direct Coombs' test is done on the infant's blood to detect antibody-coated Rh-positive RBCs. If the mother's indirect Coombs' test is positive and her Rh-positive infant has a positive direct Coombs' test, Rh immune globulin is not given; in this case, the infant is carefully monitored for hemolytic disease.
23) What would the nurse include as part of a routine physical assessment for a second-trimester primiparous patient whose prenatal care began in the first trimester and is ongoing? 1. Pap smear 2. Hepatitis B screening (HBsAg) 3. Fundal height measurement 4. Complete blood count
Answer: 3 Explanation: 3. At each prenatal visit, the blood pressure, pulse, and weight are assessed, and the size of the fundus is measured. Fundal height should be increasing with each prenatal visit.
34) A 58-year-old father and a 45-year-old mother gave birth to a baby boy 2 days ago. The nurse assesses a single palmar crease and low-set ears on the newborn. The nurse plans to counsel the couple about which chromosomal abnormality? 1. Trisomy 13 2. Trisomy 18 3. Trisomy 21 4. Trisomy 26
Answer: 3 Explanation: 3. A single palmar crease and low-set ears are characteristics of trisomy 21 (Down syndrome).
24) The client is at 6 weeks' gestation, and is spotting. The client had an ectopic pregnancy 1 year ago, so the nurse anticipates that the physician will order which intervention? 1. A urine pregnancy test 2. The client to be seen next week for a full examination 3. An antiserumpregnancy test 4. An ultrasound to be done
Answer: 3 Explanation: 3. A β-Subunit radioimmunoassay (RIA) uses an antiserum with specificity for the β-subunit of hCG in blood plasma. This test may not only detect pregnancy but also detect an ectopic pregnancy or trophoblastic disease.
3) A client is admitted to the labor and delivery unit with a history of ruptured membranes for 2 hours. This is her sixth delivery; she is 40 years old, and smells of alcohol and cigarettes. What is this client at risk for? 1. Gestational diabetes 2. Placenta previa 3. Abruptio placentae 4. Placenta accreta
Answer: 3 Explanation: 3. Abruptio placentae is more frequent in pregnancies complicated by smoking, premature rupture of membranes, multiple gestation, advanced maternal age, cocaine use, chorioamnionitis, and hypertension.
15) The nurse assessing a pregnant African American woman in the first trimester understands that a cultural practice is which of the following? 1. Use of herbs like dandelion during pregnancy to increase lactation 2. Drinking ginseng tea for faintness 3. Eating clay to supply dietary minerals 4. Consulting a spiritual advisor to ensure a healthy pregnancy and birth
Answer: 3 Explanation: 3. African American pregnant women may be guided by their extended family into common practices such as geophagia, the ingestion of dirt or clay, which is believed to alleviate mineral deficiencies.
17) The nurse is assessing a new client in the clinic. The nurse knows that the subjective (presumptive) signs and symptoms of pregnancy include which of the following? 1. Positive urine pregnancy test, enlarged abdomen, and Braxton Hicks contractions 2. Positive urine pregnancy test, amenorrhea, changes in pigmentation of the skin, and softening of the cervix 3. Increase in urination, amenorrhea, fatigue, breast tenderness, and quickening 4. Enlarged abdomen and fetal heartbeat
Answer: 3 Explanation: 3. An increase in urination, amenorrhea, fatigue, breast tenderness, and quickening are all subjective (presumptive) changes of pregnancy.
23) The nurse is presenting a class of important "dos and don'ts" during pregnancy, including travel considerations. What method of travel does the nurse recommend as most appropriate for a client in her 25th week of pregnancy? 1. Automobile 2. Airplane 3. Train 4. None; this client should not travel
Answer: 3 Explanation: 3. As pregnancy progresses, travel by train is generally recommended for long distances.
31) The nurse is discussing sexual intimacy with a pregnant couple. What should be included in the teaching plan? 1. Intercourse should stop by the beginning of the third trimester. 2. Breast fondling should be discouraged due to the potential for preterm labor. 3. The couple might need to experiment with different positions. 4. Use vaginal lubricant sparingly.
Answer: 3 Explanation: 3. As the uterus enlarges, the couple will have to experiment with different positions.
19) A woman is experiencing preterm labor. The client asks why she is on betamethasone. Which is the nurse's best response? 1. "This medication will halt the labor process until the baby is more mature." 2. "This medication will relax the smooth muscles in the infant's lungs so the baby can breathe." 3. "This medication is effective in stimulating lung development in the preterm infant." 4. "This medication is an antibiotic that will treat your urinary tract infection, which caused preterm labor."
Answer: 3 Explanation: 3. Betamethasone or dexamethasone is often administered to the woman whose fetus has an immature lung profile to promote fetal lung maturation.
7) The nurse is supervising care in the emergency department. Which situation most requires an intervention? 1. Moderate vaginal bleeding at 36 weeks' gestation; client has an IV of lactated Ringer's solution running at 125 mL/hour 2. Spotting of pinkish-brown discharge at 6 weeks' gestation and abdominal cramping; ultrasound scheduled in 1 hour 3. Bright red bleeding with clots at 32 weeks' gestation; pulse = 110, blood pressure 90/50, respirations = 20 4. Dark red bleeding at 30 weeks' gestation with normal vital signs; client reports an absence of fetal movement
Answer: 3 Explanation: 3. Bleeding in the third trimester is usually a placenta previa or placental abruption. Observe the woman for indications of shock, such as pallor, clammy skin, perspiration, dyspnea, or restlessness. Monitor vital signs, particularly blood pressure and pulse, for evidence of developing shock.
14) The nurse is preparing a client education handout on the differences between false labor and true labor. What information is most important for the nurse to include? 1. True labor contractions begin in the back and sweep toward the front. 2. False labor often feels like abdominal tightening, or "balling up." 3. True labor can be diagnosed only if cervical change occurs. 4. False labor contractions do not increase in intensity or duration.
Answer: 3 Explanation: 3. Cervical change is the only factor that actually distinguishes false from true labor. The contractions of true labor produce progressive dilatation and effacement of the cervix. The contractions of false labor do not produce progressive cervical effacement and dilatation.
34) The nurse working with mothers over 35 having their first baby knows there are some disadvantages. For what disadvantage would the nurse carefully assess in each client? 1. What kind of insurance the client has for maternity care 2. Whether the client is married 3. Whether the client will continue working after the baby arrives 4. Whether the client has any chronic disease that will have to be addressed
Answer: 4
4) The nurse is listening to the fetal heart tones of a client at 37 weeks' gestation while the client is in a supine position. The client states, "I'm getting lightheaded and dizzy." What is the nurse's best action? 1. Assist the client to sit up. 2. Remind the client that she needs to lie still to hear the baby. 3. Help the client turn onto her left side. 4. Check the client's blood pressure.
Answer: 3 Explanation: 3. During pregnancy the enlarging uterus may put pressure on the vena cava when the woman is supine, resulting in supine hypotensive syndrome. This pressure interferes with returning blood flow and produces a marked decrease in blood pressure with accompanying dizziness, pallor, and clamminess, which can be corrected by having the woman lie on her left side.
4) The nurse educator is lecturing on the changes that take place during puberty. What is a change that girls experience? 1. Elongation of the hips 2. Deepening of the voice 3. Broadening of the hips and budding of breasts 4. Preparation of the uterus for pregnancy
Answer: 3 Explanation: 3. During puberty, girls' hips start to broaden and their breasts begin to form.
8) During her first months of pregnancy, a client tells the nurse, "It seems like I have to go to the bathroom every 5 minutes." The nurse explains to the client that this is because of which of the following? 1. The client probably has a urinary tract infection. 2. Bladder capacity increases throughout pregnancy. 3. The growing uterus puts pressure on the bladder. 4. Some women are very sensitive to body function changes.
Answer: 3 Explanation: 3. During the first trimester, the growing uterus puts pressure on the bladder, producing urinary frequency until the second trimester, when the uterus becomes an abdominal organ. Near term, when the presenting part engages in the pelvis, pressure is again exerted on the bladder.
32) The nurse is preparing a presentation on the menstrual cycle for a group of high school students. Which statement should the nurse include in this presentation? 1. "The menstrual cycle has five distinct phases that occur during the month." 2. "One hormone controls the phases of the menstrual cycle." 3. "The secretory phase occurs when a woman is most fertile." 4. "Menstrual cycle phases vary in order from one woman to another."
Answer: 3 Explanation: 3. During the secretory phase, the vascularity of the entire uterus increases greatly, providing a nourishing bed for implantation.
19) Persistent early decelerations are noted. What would the nurse's first action be? 1. Turn the mother on her left side and give oxygen. 2. Check for prolapsed cord. 3. Do nothing. This is a benign pattern. 4. Prepare for immediate forceps or cesarean delivery.
Answer: 3 Explanation: 3. Early decelerations are considered benign, and do not require any intervention.
6) The client has delivered her first child at 37 weeks. The nurse would describe this to the client as what type of delivery? 1. Preterm 2. Postterm 3. Early term 4. Near term
Answer: 3 Explanation: 3. Early term births extend from 37 to 38 weeks' gestation.
11) The laboring client with meconium-stained amniotic fluid asks the nurse why the fetal monitor is necessary, as she finds the belt uncomfortable. Which response by the nurse is most important? 1. "The monitor is necessary so we can see how your labor is progressing." 2. "The monitor will prevent complications from the meconium in your fluid." 3. "The monitor helps us to see how the baby is tolerating labor." 4. "The monitor can be removed, and oxygen given instead."
Answer: 3 Explanation: 3. Electronic fetal monitoring (EFM) provides a continuous tracing of the fetal heart rate (FHR), allowing characteristics of the FHR to be observed and evaluated.
12) Which statement, if made by a pregnant client, would indicate that she understands health promotion during pregnancy? 1. "I lie down after eating to relieve heartburn." 2. "I try to limit my fluid intake to 3 or 4 glasses each day." 3. "I elevate my legs while sitting at my desk." 4. "I am avoiding exercise to stay well rested."
Answer: 3 Explanation: 3. Elevating the legs can help decrease lower leg edema.
22) A client dilated to 5 cm has just received an epidural for pain. She complains of feeling lightheaded and dizzy within 10 minutes after the procedure. Her blood pressure was 120/80 before the procedure and is now 80/52. In addition to the bolus of fluids she has been given, which medication is preferred to increase her BP? 1. Epinephrine 2. Terbutaline 3. Ephedrine 4. Epifoam
Answer: 3 Explanation: 3. Ephedrine is the medication of choice to increase maternal blood pressure.
23) The student nurse encounters a 15-year-old girl who reports that she has no pubic or axillary hair and has not yet experienced growth of her breasts. The student asks the nurse about the physiology of this occurrence. The nurse explains that the client probably lacks which hormone? 1. Testosterone 2. Progesterone 3. Estrogen 4. Prolactin
Answer: 3 Explanation: 3. Estrogens influence the development of secondary sex characteristics.
29) The client with a normal pregnancy had an emergency cesarean birth under general anesthesia 2 hours ago. The client now has a respiratory rate of 30, pale blue nail beds, a pulse rate of 110, and a temperature of 102.6°F, and is complaining of chest pain. The nurse understands that the client most likely is experiencing which of the following? 1. Pulmonary embolus 2. Pneumococcal pneumonia 3. Pneumonitis 4. Gastroesophageal reflux disease
Answer: 3 Explanation: 3. Even when food and fluids have been withheld, the gastric juice produced during fasting is highly acidic and can produce chemical pneumonitis if aspirated. This pneumonitis is known as Mendelson syndrome. The signs and symptoms are chest pain, respiratory distress, cyanosis, fever, and tachycardia. Women undergoing emergency cesarean births appear to be at considerable risk for adverse events.
24) The nurse is reviewing preconception questionnaires in charts. Which couple are the most likely candidates for preconceptual genetic counseling? 1. Wife is 30 years old, husband is 31 years old 2. Wife and husband are both 29 years old, first baby for husband, wife has a normal 4-year-old 3. Wife's family has a history of hemophilia 4. Single 32-year-old woman is using donor sperm
Answer: 3 Explanation: 3. For families in which the woman is a known or possible carrier of an X-linked disorder, such as hemophilia, the risk of having an affected male fetus is 25%.
30) The nurse is providing guidance for a woman in her second trimester of pregnancy and telling her about some of the signs and symptoms that she might experience. Which statement by the client indicates that further teaching is necessary? 1. "During the third trimester, I might have frequent urination." 2. "During the third trimester, I might have heartburn." 3. "During the third trimester, I might have back pain." 4. "During the third trimester, I might have a persistent headache."
Answer: 4
26) The nurse is providing preoperative teaching to a client for whom a cesarean birth under general anesthesia is scheduled for the next day. Which statement by the client indicates that she requires additional information? 1. "General anesthesia can be accomplished with inhaled gases." 2. "General anesthesia usually involves administering medication into my IV." 3. "General anesthesia will provide good pain relief after the birth." 4. "General anesthesia takes effect faster than an epidural."
Answer: 3 Explanation: 3. General anesthesia provides no pain relief after birth, as regional anesthesia does.
33) A client's labor has progressed so rapidly that a precipitous birth is occurring. What should the nurse do? 1. Go to the nurse's station and immediately call the physician. 2. Run to the delivery room for an emergency birth pack. 3. Stay with the client and ask auxiliary personnel for assistance. 4. Hold back the infant's head forcibly until the physician arrives for the delivery.
Answer: 3 Explanation: 3. If birth is imminent, the nurse must not leave the client alone.
27) A woman is experiencing mittelschmerz and increased vaginal discharge. Her temperature has increased by 0.6°C (1.0°F) over the past 36 hours. This most likely indicates what? 1. Menstruation is about to begin. 2. Ovulation will occur soon. 3. Ovulation has occurred. 4. She is pregnant, and will not menstruate.
Answer: 3 Explanation: 3. In some women, ovulation is accompanied by mid-cycle pain, known as mittelschmerz. This pain may be caused by a thick tunica albuginea or by a local peritoneal reaction to the expelling of the follicular contents and body temperature increases about 0.3°C to 0.6°C (0.5°F to 1°F) 24 to 48 hours after the time of ovulation.
5) What is the increased vascularization causing the softening of the cervix known as? 1. Hegar sign 2. Chadwick sign 3. Goodell sign 4. McDonald sign
Answer: 3 Explanation: 3. Increased vascularization causes the softening of the cervix known as Goodell sign.
13) The nurse is teaching an early pregnancy class for clients in the first trimester of pregnancy. Which statement by a client requires immediate intervention by the nurse? 1. "When my nausea is bad, I will drink some ginger tea." 2. "The fatigue I am experiencing will improve in the second trimester." 3. "It is normal for my vaginal discharge to be green." 4. "I will urinate less often during the middle of my pregnancy."
Answer: 3 Explanation: 3. Increased whitish vaginal discharge, called leukorrhea, is common in pregnancy. Green discharge is not a normal finding, and indicates a vaginal infection.
11) A client in her second trimester is complaining of spotting. Causes for spotting in the second trimester are diagnosed primarily through the use of which of the following? 1. A nonstress test 2. A vibroacoustic stimulation test 3. An ultrasound 4. A contraction stress test
Answer: 3 Explanation: 3. Indirect diagnosis is made by localizing the placenta via tests that require no vaginal examination. The most commonly employed diagnostic test is the transabdominal ultrasound scan.
20) A pregnant client confides to the nurse that she is eating laundry starch daily. The nurse should assess the client for which of the following? 1. Alopecia 2. Weight loss 3. Iron deficiency anemia 4. Fecal impaction
Answer: 3 Explanation: 3. Iron deficiency anemia is the most common concern with pica. The ingestion of laundry starch or certain types of clay may contribute to iron deficiency by replacing iron-containing foods from the diet or by interfering with iron absorption.
25) Which of the following drugs and drug categories can cause multiple fetal central nervous system (CNS), facial, and cardiovascular anomalies? 1. Category C: Zidovudine 2. Category B: Penicillin 3. Category X: Isotretinoin 4. Category A: Vitamin C
Answer: 3 Explanation: 3. Isotretinoin (Accutane), the acne medication, can cause multiple central nervous system (CNS), facial, and cardiovascular anomalies.
28) The labor and delivery nurse is preparing a prenatal class about facilitating the progress of labor. Which of the following frequent responses to pain should the nurse indicate is most likely to impede progress in labor? 1. Increased pulse 2. Elevated blood pressure 3. Muscle tension 4. Increased respirations
Answer: 3 Explanation: 3. It is important for the woman to relax each part of her body. Be alert for signs of muscle tension and tightening. Dissociative relaxation, controlled muscle relaxation, and specified breathing patterns are used to promote birth as a normal process.
4) The client presents to the labor and delivery unit stating that her water broke 2 hours ago. Barring any abnormalities, how often would the nurse expect to take the client's temperature? 1. Every hour 2. Every 2 hours 3. Every 4 hours 4. Every shift
Answer: 3 Explanation: 3. Maternal temperature is taken every 4 hours unless it is above 37.5°C. If elevated, it is taken every hour.
20) Which statement by a pregnant client to the nurse would indicate that the client understood the nurse's teaching? 1. "Because of their birth relationship, fraternal twins are more similar to each other than if they had been born singly." 2. "Identical twins can be the same or different sex." 3. "Congenital abnormalities are more prevalent in identical twins." 4. "Identical twins occur more frequently than fraternal twins."
Answer: 3 Explanation: 3. Monozygotic twinning is considered a random event and occurs in approximately 3 to 4 per 1000 live births. Congenital anomalies are more prevalent and both twins may have the same malformation.
16) The nurse is creating a poster for pregnant mothers. Which description of fetal development should the nurse include? 1. Four primary germ layers form from the blastocyst. 2. After fertilization, the cells only become larger for several weeks. 3. Most organs are formed by 8 weeks after fertilization. 4. The embryonic stage is from fertilization until 5 months.
Answer: 3 Explanation: 3. Most organs are formed during the embryonic stage, which lasts from the 15th day after fertilization until the end of the 8th week after conception.
12) After nalbuphine hydrochloride (Nubain) is administered, labor progresses rapidly, and the baby is born less than 1 hour later. The baby shows signs of respiratory depression. Which medication should the nurse be prepared to administer to the newborn? 1. Fentanyl (Sublimaze) 2. Butorphanol tartrate (Stadol) 3. Naloxone (Narcan) 4. Pentobarbital (Nembutal)
Answer: 3 Explanation: 3. Narcan is useful for respiratory depression caused by nalbuphine (Nubain). Respiratory depression in the mother or fetus/newborn can be improved by the administration of naloxone (Narcan), which is a specific antagonist for this agent.
17) The community nurse is working with a client at 32 weeks' gestation who has been diagnosed with preeclampsia. Which statement by the client would indicate that additional information is needed? 1. "I should call the doctor if I develop a headache or blurred vision." 2. "Lying on my left side as much as possible is good for the baby." 3. "My urine could become darker and smaller in amount each day." 4. "Pain in the top of my abdomen is a sign my condition is worsening."
Answer: 3 Explanation: 3. Oliguria is a complication of preeclampsia. Specific gravity of urine readings over 1.040 correlate with oliguria and proteinuria and should be reported to the physician.
11) The client at 39 weeks' gestation calls the clinic and reports increased bladder pressure but easier breathing and irregular, mild contractions. She also states that she just cleaned the entire house. Which statement should the nurse make? 1. "You shouldn't work so much at this point in pregnancy." 2. "What you are describing is not commonly experienced in the last weeks." 3. "Your body may be telling you it is going into labor soon." 4. "If the bladder pressure continues, come in to the clinic tomorrow."
Answer: 3 Explanation: 3. One of the premonitory signs of labor is lightening: The fetus begins to settle into the pelvic inlet (engagement). With fetal descent, the uterus moves downward, and the fundus no longer presses on the diaphragm, which eases breathing.
11) The nurse is creating a handout on reproduction for teen clients. Which piece of information should the nurse include in this handout? 1. The fertilized ovum is called a gamete. 2. Prior to fertilization, the sperm are zygotes. 3. Ova survive 12-24 hours in the fallopian tube if not fertilized. 4. Sperm survive in the female reproductive tract up to a week.
Answer: 3 Explanation: 3. Ova are considered fertile for about 12 to 24 hours after ovulation.
10) The nurse is examining a pregnant woman in the third trimester. What skin changes should the nurse highlight as an alteration for the woman's healthcare provider? 1. Linea nigra 2. Melasma gravidarum 3. Petechiae 4. Vascular spider nevi
Answer: 3 Explanation: 3. Petechiae are pinpoint red or purple spots on the skin. They are seen in hemorrhagic conditions.
29) A young adolescent is transferred to the labor and delivery unit from the emergency department. The client is in active labor, but did not know she was pregnant. What is the most important nursing action? 1. Determine who might be the father of the baby for paternity testing. 2. Ask the client what kind of birthing experience she would like to have. 3. Assess blood pressure and check for proteinuria. 4. Obtain a Social Services referral to discuss adoption.
Answer: 3 Explanation: 3. Preeclampsia is more common among adolescents than in young adults, and is potentially life-threatening to both mother and fetus. This assessment is the highest priority.
26) The prenatal client in her third trimester tells the clinic nurse that she works 8 hours a day as a cashier and stands when at work. What response by the nurse is best? 1. "No problem. Your baby will be fine." 2. "Do you get regular breaks for eating?" 3. "Your risk of preterm labor is higher." 4. "Standing might increase ankle swelling."
Answer: 3 Explanation: 3. Pregnant women who are employed in jobs that require prolonged standing (more than 3 hours) do have a higher incidence of preterm birth.
1) A client with a normal prepregnancy weight asks why she has been told to gain 25-35 pounds during her pregnancy while her underweight friend was told to gain more weight. What should the nurse tell the client the recommended weight gain is during pregnancy? 1. 25-35 pounds, regardless of a client's prepregnant weight 2. More than 25-35 pounds for an overweight woman 3. Up to 40 pounds for an underweight woman 4. The same for a normal weight woman as for an overweight woman
Answer: 3 Explanation: 3. Prepregnant weight determines the recommended weight gain during pregnancy. Underweight women are advised to gain 28-40 pounds.
13) The nurse has received a phone call from a multigravida who is 21 weeks pregnant and has not felt fetal movement yet. What is the best action for the nurse to take? 1. Reassure the client that this is a normal finding in multigravidas. 2. Suggest that she should feel for movement with her fingertips. 3. Schedule an appointment for her with her physician for that same day. 4. Tell her gently that her fetus is probably dead.
Answer: 3 Explanation: 3. Quickening, or the mother's perception of fetal movement, occurs about 18 to 20 weeks after the LMP in a primigravida (a woman who is pregnant for the first time) but may occur as early as 16 weeks in a multigravida (a woman who has been pregnant more than once).
34) Screening for gestational diabetes mellitus (GDM) is typically completed between which of the following weeks of gestation? 1. 36 and 40 weeks 2. Before 20 weeks 3. 24 and 28 weeks 4. 30 and 34 weeks
Answer: 3 Explanation: 3. Screening for gestational diabetes mellitus (GDM) is typically completed between 24 and 28 weeks' gestation.
21) A client who wishes to have an unmedicated birth is in the transition stage. She is very uncomfortable and turns frequently in the bed. Her partner has stepped out momentarily. How can the nurse be most helpful? 1. Talk to the client the entire time. 2. Turn on the television to distract the client. 3. Stand next to the bed with hands on the railing next to the client. 4. Sit silently in the room away from the bed.
Answer: 3 Explanation: 3. Standing next to the bed is supportive without being irritating. The laboring woman fears being alone during labor. The woman's anxiety may be decreased when the nurse remains with her.
11) An expectant father has been at the bedside of his laboring partner for more than 12 hours. An appropriate nursing intervention would be to do which of the following? 1. Insist that he leave the room for at least the next hour. 2. Tell him he is not being as effective as he was, and that he needs to let someone else take over. 3. Offer to remain with his partner while he takes a break. 4. Suggest that the client's mother might be of more help.
Answer: 3 Explanation: 3. Support persons frequently are reluctant to leave the laboring woman to take care of their own needs. The laboring woman often fears being alone during labor. Even though there is a support person available, the woman's anxiety may be decreased when the nurse remains with her while he takes a break.
14) The client at 30 weeks' gestation is admitted with painless late vaginal bleeding. The nurse understands that expectant management includes which of the following? 1. Limiting vaginal exams to only one per 24-hour period. 2. Evaluating the fetal heart rate with an internal monitor. 3. Monitoring for blood loss, pain, and uterine contractibility. 4. Assessing blood pressure every 2 hours.
Answer: 3 Explanation: 3. Blood loss, pain, and uterine contractibility need to be assessed for client comfort and safety.
11) During a non-stress test, the nurse notes that the fetal heart rate decelerates about 15 beats during a period of fetal movement. The decelerations occur twice during the test, and last 20 seconds each. The nurse realizes these results will be interpreted as which of the following? 1. A negative test 2. A reactive test 3. A nonreactive test 4. An equivocal test
Answer: 3 Explanation: 3. The FHR acceleration must be at least 15 beats per minute above baseline for at least 15 seconds from baseline to baseline. A nonreactive NST is one that lacks sufficient FHR accelerations over a 40-minute period.
9) A laboring client asks the nurse, "Why does the physician want to use an intrauterine pressure catheter (IUPC) during my labor?" The nurse would accurately explain that the best rationale for using an IUPC is which of the following? 1. The IUPC can be used throughout the birth process. 2. A tocodynamometer is invasive. 3. The IUPC provides more accurate data than does the tocodynamometer. 4. The tocodynamometer can be used only after the cervix is dilated 2 cm.
Answer: 3 Explanation: 3. The IUPC has several benefits over an external tocotransducer or palpation. Because the IUPC is inserted directly into the uterus, it provides near-exact pressure measurements for contraction intensity and uterine resting tone. The increased sensitivity of the IUPC allows for very accurate timing of uterine contractions (UCs).
14) The student nurse is to perform Leopold maneuvers on a laboring client. Which assessment requires intervention by the staff nurse? 1. The client is assisted into supine position, and the position of the fetus is assessed. 2. The upper portion of the uterus is palpated, then the middle section. 3. After determining where the back is located, the cervix is assessed. 4. Following voiding, the client's abdomen is palpated from top to bottom.
Answer: 3 Explanation: 3. The cervical exam is not part of Leopold maneuvers. Abdominal palpation is the only technique used for Leopold maneuvers.
4) A woman is hospitalized with severe preeclampsia. The nurse is meal-planning with the client and encourages a diet that is high in what? 1. Sodium 2. Carbohydrates 3. Protein 4. Fruits
Answer: 3 Explanation: 3. The client who experiences preeclampsia is losing protein.
26) The nurse knows that a lecithin/sphingomyelin (L/S) ratio finding of 2:1 on amniotic fluid means which of the following? 1. Fetal lungs are still immature. 2. The fetus has a congenital anomaly. 3. Fetal lungs are mature. 4. The fetus is small for gestational age.
Answer: 3 Explanation: 3. The concentration of lecithin begins to exceed that of sphingomyelin, and at 35 weeks the L/S ratio is 2:1. When at least two times as much lecithin as sphingomyelin is found in the amniotic fluid, RDS is very unlikely.
12) The nurse has just palpated contractions and compares the consistency to that of the forehead to estimate the firmness of the fundus. What would the intensity of these contractions be identified as? 1. Mild 2. Moderate 3. Strong 4. Weak
Answer: 3 Explanation: 3. The consistency of strong contractions is similar to that of the forehead.
5) The nurse has completed a presentation on reproduction. Which statement indicates that the teaching has been successful? 1. "A male is born with all the sperm he will ever produce." 2. "Females create new ova throughout their reproductive life." 3. "Ova separate into two unequally sized cells." 4. "Each primary spermatocyte divides into four haploid cells."
Answer: 3 Explanation: 3. The first meiotic division produces two cells of unequal size with different amounts of cytoplasm but with the same number of chromosomes.
31) The nurse is aware that labor and birth will most likely proceed normally when the fetus is in what position? 1. Right-acromion-dorsal-anterior 2. Right-sacrum-transverse 3. Occiput anterior 4. Posterior position
Answer: 3 Explanation: 3. The most common fetal position is occiput anterior. When this position occurs, labor and birth are likely to proceed normally.
16) What is the most significant cause of neonatal morbidity and mortality? 1. Amenorrhea 2. Posttraumatic stress disorder 3. Prematurity 4. Endometriosis
Answer: 3 Explanation: 3. The most significant cause of neonatal morbidity and mortality is prematurity and its associated complications such as respiratory distress syndrome, necrotizing enterocolitis, and intraventricular hemorrhage.
18) The nurse in a prenatal clinic finds that four clients have called with complaints related to their pregnancies. Which call should the nurse return first? 1. Pregnant woman at 7 weeks' gestation reporting nasal stuffiness 2. Pregnant woman at 38 weeks' gestation experiencing rectal itching and hemorrhoids 3. Pregnant woman at 15 weeks' gestation with nausea and vomiting and a 15-pound weight loss 4. Pregnant woman at 32 weeks' gestation treating constipation with prune juice
Answer: 3 Explanation: 3. The nurse should return this call first because this patient is the highest priority. A 15-pound weight loss is not an expected finding. Although some nausea is common, the woman who suffers from extreme nausea coupled with vomiting requires further assessment.
9) The nurse educator describes the uterus and ovaries as being held in place in the pelvic cavity by what structures? 1. Muscles 2. Tendons 3. Ligaments 4. Peritoneum
Answer: 3 Explanation: 3. The ovaries and uterus are held in place in the pelvic cavity by a number of ligaments.
2) A client is admitted to the labor and delivery unit with contractions that are regular, are 2 minutes apart, and last 60 seconds. She reports that her labor began about 6 hours ago, and she had bloody show earlier that morning. A vaginal exam reveals a vertex presenting, with the cervix 100% effaced and 8 cm dilated. The client asks what part of labor she is in. The nurse should inform the client that she is in what phase of labor? 1. Latent phase 2. Active phase 3. Transition phase 4. Fourth stage
Answer: 3 Explanation: 3. The transition phase begins with 8 cm of dilatation, and is characterized by contractions that are closer and more intense.
16) A client is admitted to the labor and delivery unit with contractions that are 2 minutes apart, lasting 60 seconds. She reports that she had bloody show earlier that morning. A vaginal exam reveals that her cervix is 100 percent effaced and 8 cm dilated. The nurse knows that the client is in which phase of labor? 1. Active 2. Latent 3. Transition 4. Fourth
Answer: 3 Explanation: 3. The transition phase begins with 8 cm to 10 cm of dilatation, and contractions become more frequent, are longer in duration, and increase in intensity.
17) A prenatal client asks the nurse how the baby can possibly come out through her vagina, because a vagina is not nearly as big as a baby. How does the nurse best answer this client's question? 1. "The vagina usually tears as it stretches during childbirth." 2. "The vagina is designed to allow a baby come through." 3. "The vagina changes due to pregnancy allow the vagina to stretch more." 4. "The vagina dilates and effaces in labor so the baby can get out."
Answer: 3 Explanation: 3. The walls of the vagina are covered with ridges, or rugae, crisscrossing each other. These rugae allow the vaginal tissues to stretch enough for the fetus to pass through during childbirth.
8) A client who is 11 weeks pregnant presents to the emergency department with complaints of dizziness, lower abdominal pain, and right shoulder pain. Laboratory tests reveal a beta-hCG at a lower-than-expected level for this gestational age. An adnexal mass is palpable. Ultrasound confirms no intrauterine gestation. The client is crying and asks what is happening. The nurse knows that the most likely diagnosis is an ectopic pregnancy. Which statement should the nurse include? 1. "You're feeling dizzy because the pregnancy is compressing your vena cava." 2. "The pain is due to the baby putting pressure on nerves internally." 3. "The baby is in the fallopian tube; the tube has ruptured and is causing bleeding." 4. "This is a minor problem. The doctor will be right back to explain it to you."
Answer: 3 Explanation: 3. The woman who experiences one-sided lower abdominal pain or diffused lower abdominal pain, vasomotor disturbances such as fainting or dizziness, and referred right shoulder pain from blood irritating the subdiaphragmatic phrenic nerve is experiencing an ectopic pregnancy.
4) The nurse is caring for a client at 30 weeks' gestation who is experiencing preterm premature rupture of membranes (PPROM). Which statement indicates that the client needs additional teaching? 1. "If I were having a singleton pregnancy instead of twins, my membranes would probably not have ruptured." 2. "If I develop a urinary tract infection in my next pregnancy, I might rupture membranes early again." 3. "If I want to become pregnant again, I will have to plan on being on bed rest for the whole pregnancy." 4. "If I have aminocentesis, I might rupture the membranes again."
Answer: 3 Explanation: 3. There is no evidence that bed rest in a subsequent pregnancy decreases the risk for PPROM.
2) The nurse is admitting a client to the labor and delivery unit. Which aspect of the client's history requires notifying the physician? 1. Blood pressure 120/88 2. Father a carrier of sickle-cell trait 3. Dark red vaginal bleeding 4. History of domestic abuse
Answer: 3 Explanation: 3. Third-trimester bleeding is caused by either placenta previa or abruptio placentae. Dark red bleeding usually indicates abruptio placentae, which is life-threatening to both mother and fetus.
14) The client in the prenatal clinic tells the nurse that she is sure she is pregnant because she has not had a menstrual cycle for 3 months, and her breasts are getting bigger. What response by the nurse is best? 1. "Lack of menses and breast enlargement are presumptive signs of pregnancy." 2. "The changes you are describing are definitely indicators that you are pregnant." 3. "Lack of menses can be caused by many things. We need to do a pregnancy test." 4. "You're probably not pregnant, but we can check it out if you like."
Answer: 3 Explanation: 3. This is a true statement, and addresses that these changes could be caused by conditions other than pregnancy.
3) The nurse educator is discussing human chromosomes with her students, and knows that the teaching has been effective when a student makes which statement? 1. "All humans have 48 chromosomes and 2 sex chromosomes." 2. "Human chromosomes are shaped like a 'Y'." 3. "Humans have 46 chromosomes, 2 of which are the sex chromosomes." 4. "Only certain body cells contain the chromosomes."
Answer: 3 Explanation: 3. This is the correct answer. There are 22 pairs of similar cells and 2 sex chromosomes.
27) At 1 minute after birth, the infant has a heart rate of 100 beats per minute, and is crying vigorously. The limbs are flexed, the trunk is pink, and the feet and hands are cyanotic. The infant cries easily when the soles of the feet are stimulated. How would the nurse document this infant's Apgar score? 1. 7 2. 8 3. 9 4. 10
Answer: 3 Explanation: 3. Two points each are scored in each of the categories of heart rate, respiratory effort, muscle tone, and reflex irritability. One point is scored in the category of skin color. The total Apgar would be 9.
17) The client with an abnormal quadruple screen is scheduled for an ultrasound. Which statement indicates that the client understands the need for this additional antepartal fetal surveillance? 1. "After the ultrasound, my partner and I will decide how to decorate the nursery." 2. "During the ultrasound we will see which of us the baby looks like most." 3. "The ultrasound will show whether there are abnormalities with the baby's spine." 4. "The blood test wasn't run correctly, and now we need to have the sonogram."
Answer: 3 Explanation: 3. Ultrasound is used to detect neural tube defects. An abnormal serum quadruple screen is not the result of a lab error, and can indicate either an open neural tube defect or trisomy 18 or 21.
22) Whether sensitization is the result of a blood transfusion or maternal-fetal hemorrhage for any reason, what test can be performed to determine the amount of Rh(D) positive blood present in the maternal circulation and to calculate the amount of Rh immune globulin needed? 1. Indirect Coombs' test 2. Nonstress test 3. Kleihauer-Betke or rosette test 4. Direct Coombs' test
Answer: 3 Explanation: 3. A Kleihauer-Betke or rosette test can be performed to determine the amount of Rh(D) positive blood present in the maternal circulation and to calculate the amount of Rh immune globulin needed.
32) The nurse is aware that a fetus that is not in any stress would respond to a fetal scalp stimulation test by showing which change on the monitor strip? 1. Late decelerations 2. Early decelerations 3. Accelerations 4. Fetal dysrhythmia
Answer: 3 Explanation: 3. A fetus that is not experiencing stress responds to scalp stimulation with an acceleration of the FHR.
7) In early-pregnancy class, the nurse emphasizes the importance of 8-10 glasses of fluid per day. How many of these should be water? 1. 1 to 2 2. 2 to 4 3. 4 to 6 4. 3 to 5
Answer: 3 Explanation: 3. A pregnant woman should consume at least 8 to 12 (8 oz) glasses of fluid each day, of which 4 to 6 glasses should be water.
24) If a woman has the pre-existing condition of diabetes, the nurse knows that she would be prone to what high-risk factor when pregnant? 1. Vasospasm 2. Postpartum hemorrhage 3. Episodes of hypoglycemia and hyperglycemia 4. Cerebrovascular accident (CVA)
Answer: 3 Explanation: 3. Episodes of hypoglycemia and hyperglycemia would be a high-risk factor for a client with pre-existing diabetes.
7) Two hours after an epidural infusion has begun, a client complains of itching on her face and neck. What should the nurse do? 1. Remove the epidural catheter and apply a Band-Aid to the injection site. 2. Offer the client a cool cloth and let her know the itching is temporary. 3. Recognize that this is a common side effect, and follow protocol for administration of Benadryl. 4. Call the anesthesia care provider to re-dose the epidural catheter.
Answer: 3 Explanation: 3. Itching is a side effect of the medication used for an epidural infusion. Benadryl, an antihistamine, can be administered to manage pruritus.
26) A client comes into the prenatal clinic accompanied by her boyfriend. When asked by the nurse why she is there, the client looks down, and the boyfriend states, "She says she is pregnant. She constantly complains of feeling tired, and her vomiting is disgusting." What is a priority for the nurse to do at this point? 1. Ask the client what time of the day her fatigue is more common. 2. Recommend that the woman have a pregnancy test done as soon as possible. 3. Continue the interview of the client in private. 4. Give the woman suggestions on ways to decrease the vomiting.
Answer: 3 Explanation: 3. The nurse should suspect that the client is in an abusive relationship. The priority is for the nurse to get the client away from the boyfriend and continue the interview.
13) The labor and birth nurse is admitting a client. The nurse's assessment includes asking the client whom she would like to have present for the labor and birth, and what the client would prefer to wear. The client's partner asks the nurse the reason for these questions. What would the nurse's best response be? Select all that apply. 1. "These questions are asked of all women. It's no big deal." 2. "I'd prefer that your partner ask me all the questions, not you." 3. "A client's preferences for her birth are important for me to understand." 4. "Many women have beliefs about childbearing that affect these choices." 5. "I'm gathering information that the nurses will use after the birth."
Answer: 3, 4 Explanation: 3. The nurse incorporates the family's expectations into the plan of care to be culturally appropriate and to facilitate the birth. 4. The nurse incorporates the family's expectations into the plan of care to be culturally appropriate and to facilitate the birth.
14) A diabetic client goes into labor at 36 weeks' gestation. Provided that tests for fetal lung maturity are successful, the nurse will anticipate which of the following interventions? Select all that apply. 1. Administration of tocolytic therapy 2. Beta-sympathomimetic administration 3. Allowance of labor to progress 4. Hourly blood glucose monitoring 5. Cesarean birth may be indicated if evidence of reassuring fetal status exists
Answer: 3, 4 Explanation: 3. There will be no attempt to stop the labor, as this can compromise the mother and fetus. 4. To reduce incidence of congenital anomalies and other problems in the newborn, the woman should be euglycemic (have normal blood glucose) throughout the pregnancy.
1) A standard ultrasound examination is performed during the second or third trimester and includes an evaluation of which of the following? Select all that apply. 1. Confirm fetal heart activity. 2. Evaluate the cervix. 3. Determine fetal presentation. 4. Amniotic fluid volume. 5. Fetal number.
Answer: 3, 4, 5
32) An abbreviated systematic physical assessment of the newborn is performed by the nurse in the birthing area to detect any abnormalities. Normal findings would include which of the following? Select all that apply. 1. Skin color: Body blue with pinkish extremities 2. Umbilical cord: two veins and one artery 3. Respiration rate of 30-60 irregular 4. Temperature of above 36.5°C (97.8°F) 5. Sole creases that involve the heel
Answer: 3, 4, 5
1) The nurse is explaining the difference between meiosis and mitosis. Which statements would be best? Select all that apply. 1. Meiosis is the division of a cell into two exact copies of the original cell. 2. Mitosis is splitting one cell into two, each with half the chromosomes of the original cell. 3. Meiosis is a type of cell division by which gametes, or the sperm and ova, reproduce. 4. Mitosis occurs in only a few cells of the body. 5. Meiotic division leads to cells that halve the original genetic material.
Answer: 3, 5 Explanation: 3. Meiosis is a special type of cell division by which diploid cells give rise to gametes (sperm and ova). 5. Meiosis creates two cells that contain half the genetic material of the parent cell.
3) The nurse is caring for laboring clients. Which women are experiencing problems related to a critical factor of labor? Select all that apply. 1. Woman at 7 cm, fetus in general flexion 2. Woman at 3 cm, fetus in longitudinal lie 3. Woman at 4 cm, fetus with transverse lie 4. Woman at 6 cm, fetus at -2 station, mild contractions 5. Woman at 5 cm, fetal presenting part is right shoulder
Answer: 3, 4, 5 Explanation: 3. A transverse lie occurs when the cephalocaudal axis of the fetal spine is at a right angle to the woman's spine and is associated with a shoulder presentation and can lead to complications in the later stages of labor. 4. Station refers to the relationship of the presenting part to an imaginary line drawn between the ischial spines of the maternal pelvis. If the presenting part is higher than the ischial spines, a negative number is assigned, noting centimeters above zero station. A -2 station is high in the pelvis. Contractions should be strong to cause fetal descent. Mild contractions will not move the baby down or open the cervix. This client is experiencing a problem between the maternal pelvis and the presenting part. 5. When the fetal shoulder is the presenting part, the fetus is in a transverse lie and the acromion process of the scapula is the landmark. This type of presentation occurs less than 1% of the time. This client is experiencing a problem between the maternal pelvis and the presenting part.
31) The OB-GYN nurse is assessing a pregnant client, and recognizes genetic amniocentesis will be indicated. The nurse makes this conclusion because the indications for genetic amniocentesis include which of the following? Select all that apply. 1. Maternal age under 35 2. Fetus with no abnormalities on ultrasound 3. One child with a chromosome abnormality 4. A family history of neural tube defects 5. Both parents with an abnormal chromosome
Answer: 3, 4, 5 Explanation: 3. Couples who have had a child with trisomy 21, 18, or 13 have approximately a 1% risk or their age-related risk, whichever is higher, of a future child having a chromosome abnormality. 4. Family history of neural tube defects is an indication for genetic amniocentesis. 5. If both parents carry an autosomal recessive disease, they have a 25% chance with each pregnancy that the fetus will be affected.
35) Which couples may benefit from prenatal diagnosis? Select all that apply. 1. Couples including women under the age of 35 2. Couples with an unbalanced translocation 3. Couples with a family history of known or suspected single-gene disorder 4. Couples including women with a teratogenic risk secondary to an exposure or maternal health condition 5. Family history of birth defects and/or intellectual disability
Answer: 3, 4, 5 Explanation: 3. Couples with a family history of known or suspected single-gene disorder (e.g., cystic fibrosis, hemophilia A or B, Duchenne muscular dystrophy) may benefit from prenatal diagnosis. 4. Women with a teratogenic risk secondary to an exposure or maternal health condition (e.g., diabetes, seizure disorder) may benefit from prenatal diagnosis. 5. Family history of birth defects and/or intellectual disability (mental retardation) (e.g., neural tube defects, congenital heart disease, cleft lip and/or palate) may benefit from prenatal diagnosis.
34) While doing a prenatal assessment on a woman who has hepatitis B and intends to become pregnant, the nurse explains the impact of the hepatitis B on pregnancy and birth. Which statement does the nurse include in the teaching? 1. "Your baby contracted hepatitis B from you when she was conceived." 2. "Don't worry about your baby during the birth. You're more likely to be affected then by the hepatitis B." 3. "Your baby will be immune to your hepatitis B." 4. "Hepatitis B does not usually affect the course of pregnancy."
Answer: 4
4) A woman at 28 weeks' gestation is asked to keep a fetal activity record and to bring the results with her to her next clinic visit. One week later, she calls the clinic and anxiously tells the nurse that she has not felt the baby move for more than 30 minutes. Which of the following would be the nurse's most appropriate initial comment? 1. "You need to come to the clinic right away for further evaluation." 2. "Have you been smoking?" 3. "When did you eat last?" 4. "Your baby might be asleep."
Answer: 4
6) The nurse is preparing to assess a laboring client who has just arrived in the labor and birth unit. Which statement by the client indicates that additional education is needed? 1. "You are going to do a vaginal exam to see how dilated my cervix is." 2. "The reason for a pelvic exam is to determine how low in the pelvis my baby is." 3. "When you check my cervix, you will find out how thinned out it is." 4. "After you assess my pelvis, you will be able to tell when I will deliver."
Answer: 4
7) The client has just been diagnosed as diabetic. The nurse knows teaching was effective when the client makes which statement? 1. "Ketones in my urine mean that my body is using the glucose appropriately." 2. "I should be urinating frequently and in large amounts to get rid of the extra sugar." 3. "My pancreas is making enough insulin, but my body isn't using it correctly." 4. "I might be hungry frequently because the sugar isn't getting into the tissues the way it should."
Answer: 4
6) The nurse admits into the labor area a client who is in preterm labor. What assessment finding would constitute a diagnosis of preterm labor? 1. Cervical effacement of 30% or more 2. Cervical change of 0.5 cm per hour 3. 2 contractions in 30 minutes 4. 8 contractions in 1 hour
Answer: 4 Explanation: 4. 8 contractions in a 60 minute period does define a diagnosis of preterm labor.
32) The midwife performs a vaginal exam and determines that the fetal head is at a -2 station. What does this indicate to the nurse about the birth? 1. The birth is imminent. 2. The birth is likely to occur in 1-2 hours. 3. The birth will occur later in the shift. 4. The birth is difficult to predict.
Answer: 4 Explanation: 4. A -2 station means that the fetus is 2 cm above the ischial spines. The ischial spines as a landmark have been designated as zero station. If the presenting part is higher than the ischial spines, a negative number is assigned, noting centimeters above zero station. With the fetus's head that high in the pelvis, it is difficult to predict when birth will occur.
4) The nurse is caring for a pregnant client. The client's husband has come to the prenatal visit. Which question is the best for the nurse to use to assess the father's adaptation to the pregnancy? 1. "What kind of work do you do?" 2. "What furniture have you gotten for the baby?" 3. "How moody has your wife been lately?" 4. "How are you feeling about becoming a father?"
Answer: 4 Explanation: 4. A husband's adaptation to pregnancy includes his feelings about impending fatherhood.
26) Intervention to reduce preterm birth can be divided into primary prevention and secondary prevention. What does secondary prevention include? 1. Diagnosis and treatment of infections 2. Cervical cerclage 3. Progesterone administration 4. Antibiotic treatment and tocolysis
Answer: 4 Explanation: 4. Secondary prevention strategies are antibiotic treatment and tocolysis.
29) The couple at 12 weeks' gestation has been told that their fetus has sickle cell disease. Which statement by the couple indicates that they are adequately coping? 1. "We knew we were both carriers of sickle cell. We shouldn't have tried to have a baby." 2. "If we had been healthier when we conceived, our baby wouldn't have this disease now." 3. "Taking vitamins before we got pregnant would have prevented this from happening." 4. "The doctor told us there was a 25% chance that our baby would have sickle disease."
Answer: 4 Explanation: 4. A true statement indicates coping. When both parents are carriers of an autosomal recessive disease, there is a 25% risk for each pregnancy that the fetus will be affected.
10) The nurse explains to a preconception class that if only a small volume of sperm is discharged into the vagina, an insufficient quantity of enzymes might be released when they encounter the ovum. In that case, pregnancy would probably not result, because of which of the following? 1. Peristalsis of the fallopian tube would decrease, making it difficult for the ovum to enter the uterus. 2. The block to polyspermy (cortical reaction) would not occur. 3. The fertilized ovum would be unable to implant in the uterus. 4. Sperm would be unable to penetrate the zona pellucida of the ovum.
Answer: 4 Explanation: 4. About a thousand acrosomes must rupture to clear enough hyaluronic acid for even a single sperm to penetrate the ovum's zona pellucida successfully. If only a small amount of sperm were released, there most likely would be an insufficient quantity of acrosomes to penetrate the zona pellucida of the ovum and allow fertilization.
21) Each of the following pregnant women is scheduled for a 14-week antepartal visit. In planning care, the nurse would give priority teaching on amniotic fluid alpha-fetoprotein (AFP) screening to which client? 1. 28-year-old with history of rheumatic heart disease 2. 18-year-old with exposure to HIV 3. 20-year-old with a history of preterm labor 4. 35-year-old with a child with spina bifida
Answer: 4 Explanation: 4. Alpha-fetoprotein (AFP) is a fetal protein that is excreted from the fetal yolk sac during the first 6 weeks of pregnancy. AFP levels can be high or low, with each having different implications for the fetus. If the fetus has a neural tube defect (NTD), the AFP levels will be elevated. NTDs can range from anencephaly to spina bifida. With a past history of a child with spina bifida, this client would be strongly encouraged to have the AFP screening.
26) A newborn has been diagnosed with a disorder that occurs through an autosomal recessive inheritance pattern. The parents ask the nurse, "Which of us passed on the gene that caused the disorder?" Which answer should the nurse tell them? 1. The female 2. The male 3. Neither 4. Both
Answer: 4 Explanation: 4. An affected individual can have clinically normal parents, but both parents are generally carriers of the abnormal gene.
18) The client at 20 weeks' gestation has had an ultrasound that revealed a neural tube defect in her fetus. The client's hemoglobin level is 8.5. The nurse should include which statement when discussing these findings with the client? 1. "Your low iron intake has caused anemia, which leads to the neural tube defect." 2. "You should increase your vitamin C intake to improve your anemia." 3. "You are too picky about food. Your poor diet caused your baby's defect." 4. "You haven't had enough folic acid in your diet. You should take a supplement."
Answer: 4 Explanation: 4. An inadequate intake of folic acid has been associated with neural tube defects (NTDs) (e.g., spina bifida, anencephaly, meningomyelocele) in the fetus or newborn.
10) Toward the end of the first stage of labor, a pudendal block is administered transvaginally. What will the nurse anticipate the client's care will include? 1. Monitoring for hypotension every 15 minutes 2. Monitoring FHR every 15 minutes 3. Monitoring for bladder distention 4. No additional assessments
Answer: 4 Explanation: 4. Because a pudendal block is done using a local anesthetic, there is no need for additional monitoring of the mother or the fetus.
30) As compared with admission considerations for an adult woman in labor, the nurse's priority for an adolescent in labor would be which of the following? 1. Cultural background 2. Plans for keeping the infant 3. Support persons 4. Developmental level
Answer: 4 Explanation: 4. Because her cognitive development is incomplete, the younger adolescent may have fewer problem-solving capabilities. The very young woman needs someone to rely on at all times during labor. She may be more childlike and dependent than older teens.
23) The nurse is caring for a laboring client. A cervical exam indicates 8 cm dilation. The client is restless, frequently changing position in an attempt to get comfortable. Which nursing action is most important? 1. Leave the client alone so she can rest. 2. Ask the family to take a coffee-and-snack break. 3. Encourage the client to have an epidural for pain. 4. Reassure the client that she will not be left alone.
Answer: 4 Explanation: 4. Because the client is in the transitional phase of the first stage of labor, she will not want to be left alone; staying with the client and reassuring her that she will not be alone are the highest priorities at this time.
17) The client with thalassemia intermedia has a hemoglobin level of 9.0. The nurse is preparing an education session for the client. Which statement should the nurse include? 1. "You need to increase your intake of meat and other iron-rich foods." 2. "Your low hemoglobin could put you into preterm labor." 3. "Increasing your vitamin C intake will help your hemoglobin level." 4. "You should not take iron supplements."
Answer: 4 Explanation: 4. Folic acid supplements are indicated for women with thalassemia, but iron supplements are not given.
6) A client is having contractions that last 20-30 seconds and that are occurring every 8-20 minutes. The client is requesting something to help relieve the discomfort of contractions. What should the nurse suggest? 1. That a mild analgesic be administered 2. An epidural 3. A local anesthetic block 4. Nonpharmacologic methods of pain relief
Answer: 4 Explanation: 4. For this pattern of labor, nonpharmacologic methods of pain relief should be suggested. These can include back rubs, providing encouragement, and clean linens.
11) A 26-year-old client is 26 weeks pregnant. Her previous births include two large-for-gestational-age babies and one unexplained stillbirth. Which tests would the nurse anticipate as being most definitive in diagnosing gestational diabetes? 1. A 50g, 1-hour glucose screening test 2. A single fasting glucose level 3. A 100g, 1-hour glucose tolerance test 4. A 100g, 3-hour glucose tolerance test
Answer: 4 Explanation: 4. Gestational diabetes is diagnosed if two or more of the following values are met or exceeded after taking the 100 g, 3-hour OGTT: Fasting: 95 mg/dL; 1 hour: 180 mg/dL; 2 hours: 155 mg/dL; 3 hours: 140 mg/dL.
15) A pregnant woman is having a nipple-stimulated contraction stress test. Which result indicates hyperstimulation? 1. The fetal heart rate decelerates when three contractions occur within a 10-minute period. 2. The fetal heart rate accelerates when contractions last up to 60 seconds. 3. There are more than five fetal movements in a 10-minute period. 4. There are more than three uterine contractions in a 6-minute period.
Answer: 4 Explanation: 4. Hyperstimulation is characterized by contractions that occur more frequently than every 2 minutes or last longer than 90 seconds.
28) The nurse teaching the phases of the menstrual cycle should include the fact that the corpus luteum begins to degenerate, estrogen and progesterone levels fall, and extensive vascular changes occur in which phase? 1. Menstrual phase 2. Proliferative phase 3. Secretory phase 4. Ischemic phase
Answer: 4 Explanation: 4. In the ischemic phase, the corpus luteum begins to degenerate, and as a result, both estrogen and progesterone levels fall. Small blood vessels rupture, and the spiral arteries constrict and retract, causing a deficiency of blood in the endometrium, which becomes pale.
28) The partner of a client at 16 weeks' gestation accompanies her to the clinic. The partner tells the nurse that the baby just doesn't seem real to him, and he is having a hard time relating to his partner's fatigue and food aversions. Which statement would be best for the nurse to make? 1. "If you would concentrate harder, you'd be aware of the reality of this pregnancy." 2. "My husband had no problem with this. What was your childhood like?" 3. "You might need professional psychological counseling. Ask your physician." 4. "Many men feel this way. Feeling the baby move in a few weeks will help make it real to you."
Answer: 4 Explanation: 4. Initially, expectant fathers may have ambivalent feelings.The extent of ambivalence depends on many factors, including the father's relationship with his partner, his previous experience with pregnancy, his age, his economic stability, and whether the pregnancy was planned. The expectant father must first deal with the reality of the pregnancy and then struggle to gain recognition as a parent from his partner, family, friends, coworkers, society-and from his baby as well.
30) Intercourse is contraindicated if the pregnancy is vulnerable because of which diagnosis? 1. Gestational diabetes 2. Cervical insufficiency (cerclage) 3. Abruptio placentae 4. Placenta previa
Answer: 4 Explanation: 4. Intercourse is contraindicated if the pregnancy is vulnerable because of the diagnosis of threatened spontaneous abortion, placenta previa, or the risk of preterm labor.
25) A woman calls the clinic and tells a nurse that she thinks she might be pregnant. She wants to use a home pregnancy test before going to the clinic, and asks the nurse how to use it correctly. What information should the nurse give? 1. The false-positive rate of these tests is quite high. 2. If the results are negative, the woman should repeat the test in 2 weeks if she has not started her menstrual period. 3. A negative result merely indicates growing trophoblastic tissue and not necessarily a uterine pregnancy. 4. The client should follow up with a healthcare provider after taking the home pregnancy test.
Answer: 4 Explanation: 4. It is important that clients remember that the tests are not always accurate and they should follow up with a healthcare provider.
13) The nurse has presented a session on pain relief options to a prenatal class. Which statement indicates that additional teaching is needed? 1. "An epidural can be continuous or can be given in one dose." 2. "A spinal is usually used for a cesarean birth." 3. "Pudendal blocks are effective when a vacuum is needed." 4. "Local anesthetics provide good labor pain relief."
Answer: 4 Explanation: 4. Local anesthetics are not used for labor pain relief. They are used prior to episiotomy and for laceration repair.
16) The client has stated that she wants to avoid an epidural and would like an unmedicated birth. Which nursing action is most important for this client? 1. Encourage the client to vocalize during contractions. 2. Perform vaginal exams only between contractions. 3. Provide a CD of soft music with sounds of nature. 4. Offer to teach the partner how to massage tense muscles.
Answer: 4 Explanation: 4. Massage is helpful for many clients, especially during latent and active labor. Massage can increase relaxation and therefore decrease tension and pain.
27) The client is carrying monochorionic-monoamniotic twins. The nurse teaches the client what this is, and the implications of this finding. The nurse knows that teaching is successful when the client states which of the following? 1. "My babies came from two eggs." 2. "About two thirds of twins have this amniotic sac formation." 3. "My use of a fertility drug led to this issue." 4. "My babies have a lower chance of surviving to term than fraternal twins do."
Answer: 4 Explanation: 4. Monochorionic-monoamniotic twins are both in one amniotic sac. There is an increased risk of umbilical cords becoming tangled or knotted and a higher incidence of fetal demise.
5) The clinic nurse is compiling data for a yearly report. Which client would be classified as a primigravida? 1. A client at 18 weeks' gestation who had a spontaneous loss at 12 weeks 2. A client at 13 weeks' gestation who had an ectopic pregnancy at 8 weeks 3. A client at 14 weeks' gestation who has a 3-year-old daughter at home 4. A client at 15 weeks' gestation who has never been pregnant before
Answer: 4 Explanation: 4. Primigravida means a woman who is pregnant for the first time.
33) The client has read that the placenta produces hormones that are vital to the function of the fetus. It is evident that that the client understands the function of the placenta when she states that which hormone is primarily responsible for the maintenance of pregnancy past the 11th week? 1. Human chorionic gonadotropin (hCG) 2. Human placental lactogen (hPL) 3. Estrogen 4. Progesterone
Answer: 4 Explanation: 4. Progesterone is a hormone essential for pregnancy. After the 11th week, the placenta produces enough progesterone and estrogen to maintain pregnancy.
22) The nurse is presenting a community education session on female hormones. Which statement from a participant indicates the need for further information? 1. "Estrogen is what causes females to look female." 2. "The presence of some hormones causes other to be secreted." 3. "Progesterone is present at the end of the menstrual cycle." 4. "Prostaglandin is responsible for achieving conception."
Answer: 4 Explanation: 4. Prostaglandin is not related to conception. Prostaglandin production increases during follicular maturation and has basic regulatory functions in cells.
23) A pregnant woman is married to an intravenous drug user. She had a negative HIV screening test just after missing her first menstrual period. What would indicate that the client needs to be retested for HIV? 1. Hemoglobin of 11 g/dL and a rapid weight gain 2. Elevated blood pressure and ankle edema 3. Shortness of breath and frequent urination 4. Persistent candidiasis
Answer: 4 Explanation: 4. Signs and symptoms of infections include fever, weight loss, fatigue, persistent candidiasis, diarrhea, cough, and skin lesions (Kaposi's sarcoma and hairy leukoplakia in the mouth).
35) Which of the following is common in many non-Western cultures and is on the increase in the United States? 1. Ceremonial rituals and rites 2. Cultural assessment 3. Cultural values 4. Cosleeping
Answer: 4 Explanation: 4. Some parents advocate cosleeping or bed sharing (one or both parents sleeping with their baby or young child). Cosleeping, which is common in many non-Western cultures, is on the increase in the United States.
21) The nurse is assessing a client who is at 35 weeks' gestation. What does the nurse expect the client to report at this phase of pregnancy? 1. Nausea and vomiting 2. Maternal ambivalence 3. Emotional shifts from highs to lows 4. Stretch marks on the abdomen
Answer: 4 Explanation: 4. Striae are purplish stretch marks that may develop as the pregnancy progresses.
25) The nurse has just palpated a laboring woman's contractions. The uterus cannot be indented during a contraction. What would the intensity of these contractions best be characterized as? 1. Weak 2. Mild 3. Moderate 4. Strong
Answer: 4 Explanation: 4. Strong intensity exists when the uterine wall cannot be indented.
2) The nurse teaching a class on the reproductive system is discussing what happens at puberty. Which statement does the nurse make? 1. Boys and girls go through puberty at the same time. 2. Most girls develop breasts and start their menses at about the same time. 3. The nocturnal emissions that adolescent boys have contain a large number of sperm. 4. The onset and progress of puberty varies widely from person to person.
Answer: 4 Explanation: 4. The age at onset and progress of puberty vary widely, physical changes overlap, and the sequence of events can vary from person to person.
24) During the fourth stage of labor, the client's assessment includes a BP of 110/60, pulse 90, and the fundus is firm midline and halfway between the symphysis pubis and the umbilicus. What is the priority action of the nurse? 1. Turn the client onto her left side. 2. Place the bed in Trendelenburg position. 3. Massage the fundus. 4. Continue to monitor.
Answer: 4 Explanation: 4. The client's assessment data are normal for the fourth stage of labor, so monitoring is the only action necessary. During the fourth stage of labor, the mother experiences a moderate drop in both systolic and diastolic blood pressure, increased pulse pressure, and moderate tachycardia.
8) During a maternal assessment, the nurse determines the fetus to be in a left occiput anterior (LOA) position. Auscultation of the fetal heart rate should begin in what quadrant? 1. Right upper quadrant 2. Left upper quadrant 3. Right lower quadrant 4. Left lower quadrant
Answer: 4 Explanation: 4. The fetal heart rate (FHR) is heard most clearly at the fetal back. Thus, in a cephalic presentation, the FHR is best heard in the lower quadrant of the maternal abdomen.
29) While caring for a client in labor, the nurse notices during a vaginal exam that the fetus's head has rotated internally. What would the nurse expect the next set of cardinal movements for a fetus in a vertex presentation to be? 1. Flexion, extension, restitution, external rotation, and expulsion 2. Expulsion, external rotation, and restitution 3. Restitution, flexion, external rotation, and expulsion 4. Extension, restitution, external rotation, and expulsion
Answer: 4 Explanation: 4. The fetus changes position in the following order: descent, flexion, internal rotation, extension, restitution, external rotation, and expulsion.
26) A 21-year-old at 12 weeks' gestation with her first baby has known cardiac disease, class III, as a result of childhood rheumatic fever. During a prenatal visit, the nurse reviews the signs of cardiac decompensation with her. The nurse will know that the client understands these signs and symptoms if she states that she would notify her doctor if she had which symptom? 1. "A pulse rate increase of 10 beats per minute" 2. "Breast tenderness" 3. "Mild ankle edema" 4. "A frequent cough"
Answer: 4 Explanation: 4. The heart's signal of its decreased ability to meet the demands of pregnancy includes frequent cough (with or without hemoptysis).
33) The nurse is preparing a handout for female adolescents on the menstrual cycle. What phase of the cycle occurs if fertilization does not take place? 1. Menstrual 2. Proliferative 3. Secretory 4. Ischemic
Answer: 4 Explanation: 4. The ischemic phase occurs if fertilization does not occur.
8) A client has just been admitted for labor and delivery. She is having mild contractions lasting 30 seconds every 15 minutes. The client wants to have a medication-free birth. When discussing medication alternatives, the nurse should be sure the client understands which of the following? 1. In order to respect her wishes, no medication will be given. 2. Pain relief will allow a more enjoyable birth experience. 3. The use of medications allows the client to rest and be less fatigued. 4. Maternal pain and stress can have a more adverse effect on the fetus than would a small amount of analgesia.
Answer: 4 Explanation: 4. The nurse can explain to the couple that, although pharmacologic agents do affect the fetus, so do the pain and stress experienced by the laboring mother. If the woman's pain and anxiety are more than she can cope with, the adverse physiologic effects on the fetus may be as great as would occur with the administration of a small amount of an analgesic agent. Once the effects are explained, however, it is still the client's choice whether to receive medication.
16) The nurse explains to the client that the obstetric conjugate measurement is important because of which reason? 1. This measurement determines the tilt of the pelvis. 2. This measurement determines the shape of the inlet. 3. The fetus passes under it during birth. 4. The size of this diameter determines whether the fetus can move down into the birth canal so that engagement can occur.
Answer: 4 Explanation: 4. The obstetric conjugate extends from the middle of the sacral promontory to an area approximately 1 cm below the pubic crest. The fetus passes through the obstetric conjugate, and the size of this diameter determines whether the fetus can move down into the birth canal in order for engagement to occur.
29) The nurse is preparing a handout on the ovarian cycle to a group of middle school girls. Which information should the nurse include? 1. The hormone human chorionic gonadotropin stimulates ovulation. 2. Irregular menstrual cycles have varying lengths of the luteal phase. 3. The ovum leaves its follicle during the follicular phase. 4. There are two phases of the ovarian cycle: luteal and follicular.
Answer: 4 Explanation: 4. The ovarian cycle has two phases: the follicular phase (days 1 to 14) and the luteal phase (days 15 to 28 in a 28-day cycle).
15) Prior to receiving lumbar epidural anesthesia, the nurse would anticipate placing the laboring client in which position? 1. On her right side in the center of the bed with her back curved 2. Lying prone with a pillow under her chest 3. On her left side with the bottom leg straight and the top leg slightly flexed 4. Sitting on the edge of the bed
Answer: 4 Explanation: 4. The woman is positioned on her left or right side, at the edge of the bed with the assistance of the nurse, with her legs slightly flexed, or she is asked to sit on the edge of the bed.
19) Why is it important for the nurse to assess the bladder regularly and encourage the laboring client to void frequently? 1. A full bladder impedes oxygen flow to the fetus. 2. Frequent voiding prevents bruising of the bladder. 3. Frequent voiding encourages sphincter control. 4. A full bladder can impede fetal descent.
Answer: 4 Explanation: 4. The woman should be encouraged to void because a full bladder can interfere with fetal descent. If the woman is unable to void, catheterization may be necessary.
33) A nurse counsels a couple on sex-linked disorders. Both the man and the woman are carriers of the disorder. They ask the nurse how this disorder will affect any children they might have. What is the nurse's best response? 1. "If you have a daughter, she will not be affected." 2. "Your son will be affected because the father has the disorder." 3. "There is a 25% chance that your son will have the disorder because the mother has the disorder." 4. "There is a 50% chance that your son will be a carrier only."
Answer: 4 Explanation: 4. There is a 50% chance that a carrier mother will pass the normal gene to each of her sons, who will be unaffected.
4) The charge nurse has received the shift change report. Which client requires immediate intervention? 1. Woman at 6 cm undergoing induction of labor, strong contractions every 3 minutes 2. Woman at 4 cm whose fetus is in a longitudinal lie with a cephalic presentation 3. Woman at 10 cm and fetus at +2 station experiencing a strong expulsion urge 4. Woman at 3 cm screaming in fear because her mother died during childbirth
Answer: 4 Explanation: 4. This client is most likely fearful that she will die during labor because her mother died during childbirth. This client requires education and a great deal of support, and is therefore the top priority.
29) The client at 30 weeks' gestation with her first child is upset. She tells the prenatal clinic nurse that she is excited to become a mother, and has been thinking about what kind of parent she will be. But her mother has told her that she doesn't want to be a grandmother because she doesn't feel old enough, while her husband has said that the pregnancy doesn't feel real to him yet, and he will become excited when the baby is actually here. What is the most likely explanation for what is happening within this family? 1. Her husband will not attach with this child and will not be a good father. 2. Her mother is rejecting the role of grandparent, and will not help out. 3. The client is not progressing through the developmental tasks of pregnancy. 4. The family members are adjusting to the role change at their own paces.
Answer: 4 Explanation: 4. This is a true statement. With each pregnancy, routines and family dynamics are altered, requiring readjustment and realignment.
17) The nurse working with a client describes cellular multiplication and how the zygote moves through the fallopian tube, a movement that takes place via what process? 1. A squeezing motion 2. Pushing from another ovum that has not been fertilized 3. Hormone action 4. A very weak fluid current in the fallopian tube resulting from the beating action of ciliated epithelium
Answer: 4 Explanation: 4. This is correct. There is a very weak fluid current in the fallopian tube resulting from the beating action of the ciliated epithelium that lines the tube.
5) The nurse is orienting a new graduate nurse to the labor and birth unit. Which statement indicates that teaching has been effective? 1. "When a client arrives in labor, a urine specimen is obtained by catheter to check for protein and ketones." 2. "When a client arrives in labor, she will be positioned supine to facilitate a normal blood pressure." 3. "When a client arrives in labor, her prenatal record is reviewed for indications of domestic abuse." 4. "When a client arrives in labor, a vaginal exam is performed unless birth appears to be imminent."
Answer: 4 Explanation: 4. Unless delivery seems imminent because the client is bearing down or contractions are very close and strong, the vaginal exam is performed after the vital signs are obtained.
21) A clinic nurse is planning when to administer Rh immune globulin (RhoGAM) to an Rh-negative pregnant client. When should the first dose of RhoGAM be administered? 1. After the birth of the infant 2. 1 month postpartum 3. During labor 4. At 28 weeks' gestation
Answer: 4 Explanation: 4. When the woman is Rh negative and not sensitized and the father is Rh positive or unknown, Rh immune globulin is given prophylactically at 28 weeks' gestation.
28) A 21-year-old woman is at 12 weeks' gestation with her first baby. She has cardiac disease, class III, as a result of having had childhood rheumatic fever. Which planned activity would indicate to the nurse that the client needs further teaching? 1. "I will be sure to take a rest period every afternoon." 2. "I would like to take childbirth education classes in my last trimester." 3. "I will have to cancel our trip to Disney World." 4. "I am going to start my classes in water aerobics next week."
Answer: 4 Explanation: 4. With the slightest exertion, the client's heart rate will rise, and she will become symptomatic. Therefore, she should not establish a new exercise program.
4) A 20-year-old woman is at 28 weeks' gestation. Her prenatal history reveals past drug abuse, and urine screening indicates that she has recently used heroin. The nurse should recognize that the woman is at increased risk for which condition? 1. Erythroblastosis fetalis 2. Diabetes mellitus 3. Abruptio placentae 4. Pregnancy-induced hypertension
Answer: 4 Explanation: 4. Women who use heroin are at risk for poor nutrition, anemia, and pregnancy-induced hypertension (or preeclampsia-eclampsia).
7) The nurse is returning phone calls from clients. Which client does the nurse anticipate would not require a serum beta hCG? 1. A client with a risk of ectopic pregnancy 2. A client with spotting during pregnancy 3. A client with previous pelvic inflammatory disease 4. A client with a previous history of twins
Answer: 4 Explanation: 4. A previous history of twins is not a risk factor for ectopic pregnancy. Beta hCG testing is not indicated for this client.
24) When preparing nutritional instruction, which pregnant client would the nurse consider the highest priority? 1. 40-year-old gravida 2 2. 22-year-old primigravida 3. 35-year-old gravida 4 4. 15-year-old nulligravida
Answer: 4 Explanation: 4. An expectant adolescent must meet the nutritional needs for her own growth in addition to the nutritional needs of pregnancy.
25) Which third-trimester client would the nurse suspect might be having difficulty with psychological adjustments to her pregnancy? 1. A woman who says, "Either a boy or a girl will be fine with me" 2. A woman who puts her feet up and listens to some music for 15 minutes when she is feeling too stressed 3. A woman who was a smoker but who has quit at least for the duration of her pregnancy 4. A woman who has not investigated the kind of clothing or feeding methods the baby will need
Answer: 4 Explanation: 4. By the third trimester, the client should be planning and preparing for the baby (for example, living arrangements, clothing, feeding methods).
24) A woman is in labor. The fetus is in vertex position. When the client's membranes rupture, the nurse sees that the amniotic fluid is meconium-stained. What should the nurse do immediately? 1. Change the client's position in bed. 2. Notify the physician that birth is imminent. 3. Administer oxygen at 2 liters per minute. 4. Begin continuous fetal heart rate monitoring.
Answer: 4 Explanation: 4. Meconium-stained amniotic fluid is an abnormal fetal finding, and is an indication for continuous fetal monitoring.
18) The primigravida at 22 weeks' gestation has a fundal height palpated slightly below the umbilicus. Which of the following statements would best describe to the client why she needs to be seen by a physician today? 1. "Your baby is growing too much and getting too big." 2. "Your uterus might have an abnormal shape." 3. "The position of your baby can't be felt." 4. "Your baby might not be growing enough."
Answer: 4 Explanation: 4. The fundal height at 20-22 weeks should be about even with the umbilicus. At 22 weeks' gestation, a fundal height below the umbilicus and the size of the uterus that is inconsistent with length of gestation could indicate fetal demise.
13) The nurse is working in an outpatient clinic. Which client's indications most warrant fetal monitoring in the third trimester? 1. Gravida 4, para 3, 39 weeks, with a history of one spontaneous abortion at 8 weeks 2. Gravida 1, para 0, 40 weeks, with a history of endometriosis and a prior appendectomy 3. Gravida 3, para 2, with a history of gestational diabetes controlled by diet 4. Gravida 2, para 1, 36 weeks, with a history of history of preterm labor or cervical insufficiency
Answer: 4 Explanation: 4. The preterm client with a history of preterm labor or cervical insufficiency needs close monitoring for preterm labor onset.
6) The client at 24 weeks' gestation is experiencing painless vaginal bleeding after intercourse. The physician has ordered a transvaginal ultrasound examination. Which statements by the client indicate an understanding of why this exam has been requested? Select all that apply. 1. "This ultrasound will show the baby's gender." 2. "This ultrasound might cause the miscarriage of my baby." 3. "This ultrasound carries a risk of creating a uterine infection." 4. "This ultrasound can determine the location of my placenta." 5. "This ultrasound might detect whether the placenta is detaching prematurely."
Answer: 4, 5 Explanation: 4. Painless bleeding in the second and third trimesters can be a symptom of placenta previa. Transvaginal ultrasound will determine the placental location. 5. Painless bleeding in the second and third trimesters can be a symptom of placenta previa. Transvaginal ultrasound will determine the placental location.
1) The nurse is caring for a client who was just admitted to rule out ectopic pregnancy. Which orders are the most important for the nurse to perform? Select all that apply. 1. Assess the client's temperature. 2. Document the time of the client's last meal. 3. Obtain urine for urinalysis and culture. 4. Report complaints of dizziness or weakness. 5. Have the lab draw blood for B-hCG level every 48 hours.
Answer: 4, 5 Explanation: 4. Reporting complaints of dizziness and weakness is important, as it can indicate hypovolemia from internal bleeding. 5. Having the lab draw blood for B-hCG levels every 48 hours is important, as the level rises much more slowly in ectopic pregnancy than in normal pregnancy.
4) During the initial intrapartal assessment of a client in early labor, the nurse performs a vaginal examination. The client's partner asks why this pelvic exam needs to be done. The nurse should explain that the purpose of the vaginal exam is to obtain information about which of the following? Select all that apply. 1. Uterine contraction pattern 2. Fetal position 3. Presence of the mucous plug 4. Cervical dilation and effacement 5. Presenting part
Answer: 4, 5 Explanation: 4. The vaginal examination of a laboring client obtains information about the station of the presenting part and the dilation and effacement of the cervix. 5. The vaginal examination of a laboring client obtains information about the fetal presenting part.
8) A woman who is 40 weeks pregnant calls the labor suite to ask whether she should be evaluated. Which statements by the client indicate she is likely in labor? Select all that apply. 1. "The contractions are 5-20 minutes apart." 2. "I had pink discharge on the toilet paper." 3. "I have had cramping for the past 3-4 hours." 4. "The contractions start in my back and then go to my abdomen and are very intense." 5. "The contractions hurt more when I walk."
Answer: 4, 5 Explanation: 4. This is a sign of true labor. The contractions increase in duration and intensity and begin in the back and radiate around to the abdomen. 5. It is a sign of true labor when the client is unable to walk during the contraction.